Sei sulla pagina 1di 220

97

I.

Fill in each blank on the answer sheet with a letter (A, B, C, etc.) representing the word chosen
from the list below to make the passage a coherent one.
A. honor B. moment
C. defense D. punctured E. displaced
F. spat
G. suffocated H. crime
I. proclaimed J. infatuation
Rand Abdel-Qader, 17, told her closest friend that she was in love from the __1B__ she set

eyes on the young British soldier working alongside her in Basra, and she dreamed of a future with
him.
It was an innocent __2J__, but five months after Rand, a student of English at Basra
University, met Paul, a 22-year-old soldier posted to southern Iraq, she was dead. She was stamped
on, __3_G_ and stabbed by her father. Several brutal knife wounds __4_D_ her slender,
bruised bodyfrom her face to her feet. He had done it, he __5_I_ to the neighbors who soon
gathered round, to cleanse his __6A__.
And as Rand was put into the ground, without ceremony, her uncles __7_F_ on her covered
corpse because she had brought shame on the family. Her __8H__ was the worst they could
possibly imagine.
Rand was murdered in March. That the relationship was innocent was no __9C__. She had
been seen conversing intimately with Paul. It was enough to condemn her, because he was British,
a Christian, and the invader. The two met while he was helping to deliver relief aid to __10E
__ families in the city and she was working as a volunteer.
II.

Fill in each blank on the answer sheet with a word that best completes the sentences in the
passage.

My goal here is to illustrate how a language socialization approach can be applied to L2


classroom research to explore teaching and learning processes. The pe__1 pedagogical __al unit of
analysis or activity setting was not a task as it is typically operationalized in SLA studiesn__2_
namely _y, a goal-directed collaborative activity involving the exchange of information between
interlocutors to solve various kinds of problems. Such tasks are usually c__3 confined __ned to one
lesson or one short laboratory session. Instead, we focus on a larger activity context, a project,
comprising a s__4_ series _s of tasks performed inside and outside of class over a number of days
or weeks that culminate in one major project with a co__5_ corresponding _ding group/pair
presentation and individual term papers. The project presentation and term paper constitute an oral
and written g__6_ genre _e of research synthesis and discussion valued within not only the
academic exchange program at this university but also the wider local academic context. Project
work is often used in content-based language teaching and other academic, vo__7_ vocational _l,
and other real-world settings because students must actively build a community of inquiry
involving a__8_ authentic _ntic communication, cooperative learning, collaboration, and
problem-solving (Stoller, 2002, p. 107). It therefore provides a social setting in which peers are
expected to develop their language, li__9 literacy __y, and communication skills, as well as their
critical thinking and problem-solving skills, and also develop their identities, roles, and agency as
1-40

group members working together to ac__10_ accomplish _h meaningful projects in English. These
social, cognitive, and pragmatic aspects of project work were among the instructors top priorities.
III.

Fill in each blank on the answer sheet with ONE word that best completes the sentences in the
passages below.
Passage A:
Phenomenography takes human experience as its object of study. It investigates the ways in
which the same phenomenon is experienced in qualitatively different ways by individuals. It adopts
a research approach which is emic, __1as__ experienced by the individual, and investigates the
phenomenon in a relatively naturalistic setting. As __2 such __, it is ethnographic in orientation. In
the context of learning, phenomenographers take the object of learning, __3_that_ is, what is to be
learnt, as their point of departure for their investigations. Early phenomenographic research
addressed the question of why are some people better learners than __4 others __. They found that
what learners learnt was inseparable from their approaches to learning.
Passage B:
Decaying brains have inspired a mini-industry of brain health products__5 not __ just
supplements like coenzyme Q10, ginseng and bacopa, but computer-based fitter-brain products as
__6 well __. Nintendos US$19.99 Brain Age 2, a popular video game of simple math and memory
exercises, is __7 one __. Posit Sciences US$395 computer-based cognitive behavioral training
exercises are another. And for about US$10 a month, worried boomers can subscribe to Web sites
like Lumosity.com and Happy-Neuron.com, __8_ which _ offer a variety of cognitive training
exercises.
Alvaro Fernandez, __9_ whose _ brain fitness and consulting company, SharpBrains, has a
Web site focused on brain fitness research, estimates __10 that _ the US market for so-called
neurosoftware last year reached US$225 million.
IV. Reading comprehension: Choose the best answer for each question.
Passage A:
In the long run a government will always encroach upon freedom to the extent to which it has
the power to do so; this is almost a natural law of politics, since, whatever the intentions of the men
who exercise political power, the sheer momentum of government leads to a constant pressure upon
the liberties of the citizen. But in many countries society has responded by throwing up its own
defenses in the shape of social classes or organized corporations which, enjoying economic power
and popular support, have been able to set limits to the scope of action of the executive. Such, for
example, in England was the origin of all our libertieswon from government by the stand first of
the feudal nobility, then of churches and political parties, and latterly of trade unions, commercial
organizations, and the societies for promoting various causes. Even in European lands which were
arbitrarily ruled, the powers of the monarchy, though absolute in theory, were in their exercise
checked in a similar fashion. Indeed the fascist dictatorships of today are the first truly tyrannical
governments which western Europe has known for centuries, and they have been rendered possible
only because on coming to power they destroyed all forms of social organization which were in any
2-40

way rivals to the state.


C1.The main idea of this passage is best expressed as:
(A)Limited powers of monarchies
(B)The ideal of liberal government
(C)Safeguards of individual liberty
(D)Ruthless ways of dictators
B2.The writer maintains that there is a natural tendency for governments to _____.
(A)become fascist
(B)assume more power
(C)increase individual liberties
(D0suppress trade unions and social societies
B3.According to the passage, monarchy was first checked in England by the _____.
(A)church
(B)nobles
(C)trade unions
(D)political parties
A4.Fascist dictatorships differ from monarchies of recent times in _____.
(A)destroying peoples organizations
(B)setting limits to their scope of action
(C)exerting constant pressure on liberties
(D)getting things done by sheer momentum
Passage B:
Are we getting more than our usual share of snow in this part of the country, or less? Its hard to
say one way or the other. Only within recent years has there been enough interest in snow,
especially from the sport angle, to bring about careful recording of snow depths. Most weather
records lump snow and rain under the noncommittal head of precipitation. We do know that,
because of a general trend to higher temperatures over the last half-century or so, the winter
precipitation in this part of the world has had an increasing tendency in recent years to take the
form of rain. Whether this trend will continue this winter and in future winters is anybodys guess.
The process of sublimation that leads to the precipitation of snow is one that man can not at all
control and one that man has only a limited ability to predict.
C5.The title that best expresses the ideas of this passage is:
(A)Weather prediction
(B)Snow and winter sports
(C)A possible trend in weather
(D)What meteorological records show
D6.According to the passage, we do not know whether or not we are getting more snow than
usual because _____.
(A) winters are getting warmer
(B) weather reports are inaccurate
(C) we cannot predict weather accurately
3-40

(D) formerly snowfall and rain were not reported separately


A7.According to the passage, during the last 50 years, there has been _____.
(A)less snow and more rain
(B)greater interest in winter sports
(C)a tendency toward colder winters
(D)an increase in average annual snowfall
Passage C:
The place of our retreat was a little neighborhood consisting of farmers, who tilled their own
grounds, and were equal strangers to opulence and poverty. As they had almost all the conveniences
of life within themselves, they seldom visited towns or cities in search of superfluity. Remote from
the polite, they still retained the primeval simplicity of manners; and frugal by habit, they scarcely
knew that temperance was a virtue.
B8.The farmers described in the passage were _____.
(A)very poor
(B)temperate
(C)economical
(D)extravagant
B9.This passage mainly describes _____.
(A)a place of seclusion
(B)virtues of the farmer
(C)comforts on the farm
(D)a primeval neighborhood
A10. According to the passage, which of the following statements is true?
(A)Quiet life leads to temperance.
(B)Troubled people seek superabundance.
(C)Politeness and simplicity of manners go hand in hand.
(D)Self-sufficient farmers have no desire to visit towns and cites.

4-40


I . Vocabulary and Phrases :
1. Nobody believes the boys _____ that he didnt steal the money except for his mother.
(A) alternation (B) assertion (C) attribution (D) annotation
2. The baby quickly fell asleep last night to the _____ sound of water dripping from a leaky
faucet. (A) mordacious (B) mutable (C) monotonous (D) miscellaneous
3. The cyclist dangerously _____ one car after another trying to get through rush hour traffic.
(A) uploaded (B) dodged (C) alighted (D) jolted
4. We have solid evidence that they were trying to ______ to drive us out of the party.
(A) conspire (B) expropriate (C) allocate (D) fortify
5. Anyone discovered tampering with survey results or the collection of data will be held
_____ for their action. (A) accountable (B) accredited (C) accosted (D) abolished
6. By now, there have been very few documents that could effectively _____ the generals
insistence on war.
(A) bleach (B) expound (C) feign (D) tarnish
7. Nina is a _____. She tells us to care about the weak and the poor but she doesnt.
(A) hypothesis (B) hypocrite (C) hypnotist (D) hymnody
8. During the war, many refugees sought ______ in the U.S.
(A) sapphire (B) sanctuary (C) scripture (D) scheme
9. Being free from financial worries means taking _____, like having life insurance,
disability insurance, a three-to-six month emergency cushion and a will.
(A) procedure (B) precautions (C) prevalence (D) prestige
10. The joke was so ______ that all of us laughed until our stomach hurt.
(A) hilarious (B) speculative (C) sterilized (D) tactful
11. Lilia will not ____ her age. She doesnt want people to know how old she is.
(A) resonate (B) duplicate (C) divulge (D) emulate;
12. The employee used _____ checks to steal several thousand dollars from his coworkers
travel expense account.
(A) amenity (B) fraudulent (C) pendulum (D) ensemble
13. Because its first store was so successful, the _____ business is opening stores in ten major cities
across the nation.
(A) remunerative (B) instantaneous (C) reminiscent (D) resonant
14. Bill Gstes now spends his time on many _____ projects such as charity work.
(A) charismatic (B) altruistic (C) euphoric (D) anonymous
15. When Darren caught a cold, he _____ himself at home until he was well again.
(A) confiscated (B) incorporated (C) sequestered (D) orchestrated
16. That was an _____ performance! Annie has never sung so beautifully in her life.
(A) insentient (B) exquisite (C) oracular (D) amicable
17. The customer was extremely angry because of the broken products, so the manager apologized
to him _____ the company.
5-40

(A) on account of (B) in charge of (C) in view of (D) on behalf of


18. We felt disguised with the governors _____ mishandling of the hostage crisis; he was virtually
incompetent.
(A) grossly (B) indomitable (C) sveltely (D) emollient
19. Economical growth should never be pursued _____ environmental pollution.
(A) at the sight of (B) at the thought of (C) at the mercy of (D) at the expense of
20. Katie believes it was _____ that she met her future husband sitting on her favorite bench in her
favorite park.
(A) serendipity (B) defiance (C) composure (D) camaraderie
II. Discourse structure:
In February 2001, in the scientific magazine Nature, an article was published that was
described by then U.S. Bill Clinton as the most important, most wondrous map ever produced.
But what is this amazing map? It is the human genome sequence. ____21_____ It is the
information that you inherited from your parents! This genetic information consists of three billion
pieces of data in the form of deoxyribonucleic acid, which is usually referred to as DNA.
____22_____ What does the genome do? If you think of your body as being a complex biological
machine, then the genome is the blueprint for that machine. ____23_____ The instructions are
called genes. Your genes give you your physical characteristics such as your height, hair color, eye
color and so on.
But the genome is not just a list of all the genes in our bodies. It can also reveal how the
genes interact in order to carry out a very wide range of functions, such as causing an embryo to
turn into a baby, or fighting disease. ____24_____
You are 99.9 percent the same as the person sitting next to you and a stranger on the other
side of the world. You are even closer than that to your mother, father, sister, uncle, cousin or
grandfather. A difference of 0.1 percent may seem tiny, but it equals about three million differences
at the genetic level, which is plenty to show that every human being is a unique individual. Some of
the differences between us have no apparent effect, but others influence our appearance, our
behavior, how likely we are to suffer from particular diseases and how we respond to medical
drugs.
It is also clear that the old idea that skin color is a useful way of describing the differences
between people is completely false. The differences within racial groups are often greater than
those between people of different colors. _____25______
(A) So, understanding how the genes fit together allows us to read the complete set of instructions
for making the human being.
(B) In other words, theories of race and racial superiority have no basis in fact at the genetic level.
(C) Your genome is your genetic constitution.
(D) And DNA is found in every cell and every living thing, from bacteria to trees and people.
(E) This blueprint includes a list of the parts of the machine and instructions, for how the parts are
put together.
6-40

III. Cloze Test :


2008 is no common year; it is a leap year, with the extra day designated as February 29. A
leap year occurs every four years to help the Gregorian calendar, which now serves as the standard
calendar for civil use around the globe, synchronize the calendar year __26__ the solar year. The
adjustment is __27__ because the actual time between two yearly solar events is not a clean 365
days. It works out to be 365.24 days or so, __28__ almost one extra day every four years.
For it is not exactly one full day, the rules for leap years are __29__ more complicated than
every four years. If a year is divisible by four, its a leap year __30__ its also divisible by 100, in
which case its not a leap year. But if the year is divisible by 400, then it is a leap year, which
means that although 1800 and 1900 were not leap years, the year 2000 was.
26. (A) into
(B) with
(C) from
(D) of
27. (A) fascinating
(B) limited
(C) incredible
(D)essential
28. (A) that leads to (B) resulting in
(C) it contributes to (D) and causing
29. (A) far
(B) very
(C) so
(D) more
30. (A) until
(B) otherwise
(C) unless
(D) whenever

.There are ten errors in the following passage. Please underline and correct them.
The famous generation gap between young people and their parents that was so apparent
in the 1960s still exists today. It is said that the generation gap is a division between two
generations, generally young people and their parents. When parents and children discover they
have little in common, the gap has begun to form. Young people maintain that their parents dont
understand them, worry too much, and restrict their freedom. The parents respond that their
children dont respect them, watch too much television, and want too much freedom at an early age.
In other words, what children and parents respect are vastly different. What are the reasons for this
lack of understanding is hard to say, but there are a few theories.
In previous decades, parents have said that the generation gap developed from young people
challenging their parents old-fashioned ideals and beliefs; however, in the technological
twenty-first century, old and young generations have said that they are separated by different skills
and abilities. These challenges are true for all families, but for immigrant families the problem may
be more serious. The older generation tells to its children that they have lost all sense of tradition
and culture. What do the children learn to value in their new culture is not the same as what do their
traditional parents value.
Children of earlier generations have sworn that they will not let this happen in their families,
yet the gap has continued to exist. That a lack of communication and respect is causing their
children to drift farther and farther from them are shocking to this generation of parents. In order to
slow the formation of the generation gap, it is vital that a child listens to his or her parents. In
addition, counselors suggest that parents are available to listen to their children at all times. Many
7-40

families dont know what they will be able to heal the gap between the generations, but young
people and their parents must still try.

. Translation :
1. The European dream is under assault, as the wave of inflation sweeping the globe mixes with
this continents long-stagnant wages. Families that once enjoyed Europes vaunted quality of
life are saving their money for necessities, and cutting back on extra like movies and vacations
abroad.
2. Robots revolutionized manufacturing during the 1980s, thanks to advances in computing,
motion controls and software design. Visionaries like Dr. Richard M. Satava, who oversaw
American government-funded medical robotics research at the time, predicted that robots
would eventually be able to operate as precisely as the worlds greatest surgeons and far more
tirelessly, perhaps even in remote locations, through satellite links.

3. ,
,
4.

VI. Reading Comprehension


Habits are a funny thing. We reach for them mindlessly, setting our brains on auto-pilot and
relaxing into the unconscious comfort of familiar routine. So it seems antithetical to talk about
habits in the same context as creativity and innovation.
But brain researchers have discovered that when we consciously develop new habits, we
create parallel synaptic paths, and even entirely new brain cells, that can jump our trains of thought
onto new, innovation tracks.
No choice, but habit rules the unreflecting herd, William Wordsworth said in the 19th century.
In the ever-changing 21st century, even the word habit carries a negative connotation.
But the new habits we deliberately ingrain into ourselves create parallel pathways that can
bypass those old roads.
The first thing needed for innovation is a fascination with wonder, says Dawna Markova,
8-40

author of The Open Mind and an executive change consultant for Professional Thinking Partners.
But we are taught instead to decide. She adds, however, that to decide is to kill off all
possibilities but one. A good innovational thinker is always exploring the many other possibilities.
Ms. Ryan and Ms. Markova have found what they call three zones of existence: comfort,
stretch and stress. Comfort is the realm of existing habit. Stress occurs when a challenge is so far
beyond current experience as to be overwhelming. Its that stretch zone in the middleactivities
that feel a little awkward and unfamiliarwhere true change occurs.
Getting into the stretch zone is good for you, Ms. Ryan says in her book. It helps keep your
brain healthy. It turns out that unless we continue to learn new things, they literally begin to atrophy,
which may result in dementia, Alzheimers and other brain diseases.
Ms. Markova concludes that You cannot have innovation, unless you are willing and able to
move through the unknown and go from curiosity to wonder.
Essay questions:
1. What does William Wordsworth mean by saying No choice, but habit rules the
unreflecting herd?
2. According to the reading above, what is the relation between habits and innovation?
3. How do the zones of comfort, stretch, and stress differ from one another? How to stay in
the stretch zone?

.Teaching Proficiency
1.

Please apply the words and idioms listed below to make five questions for choice and another
five for a cloze test that you think are appropriate to assess students learning.
Questions for choice: blame drain elaborate preserve put up with
Cloze test: means academic expectation make best use of fulfill

2.

Please give a comprehensible account of how these answers in the cloze test are made. (The
bolded-faced are the answers. The explanation could be done either in English or
Chinese. )
If you continue downhill a little bit further, youll reach a pleasant park __1__ five
views both down towards the East China Sea and back uphill to the village, backed by
dramatic, craggy hill. This place is __2__ crowded; indeed most tourists dont even know
of its existence, nor the profound significance of this pretty oasis of flower beds.
Yet this is the site of the notorious Kinkaseki Concentration Camp, where, during the Second
World War, occupying Japanese forces imprisoned allied POWs __3___ during the fall of
Singapore in 1942. An estimated two-thirds of the prisoners died through ill health or in
accidents down the copper mines nearby where they were put to work as slaves.
Today, all that remains of the camp, apart from the memories, __4__ a single gate post
and section of wall in one corner of the park, and a simple, pure white memorial with an
inscription, in English _5__ to the prisoners.
9-40

(1.)
(2.)
(3.)
(4.)
(5.)
3.

(A) reassuring (B) bestowing (C) commanding (D) releasing


(A) more than (B) ever (C) all too (D) hardly
(A) were captured (B) capturing (C) who captured (D) captured
(A) is (B) being (C) are (D) to be
(A) dedicated (B) devoted (C) awarded (D) commemorated

With so many English learning materials in society, what can an English teacher do to
integrate these materials into their teaching?

10-40

BCBAA BBBBA CBABC

BDADA CDEAB

BDBAC

.There are ten errors in the following passage. Please underline and correct them.10%
The famous generation gap between young people and their parents that was so apparent
in the 1960s still exists today. It is said that the generation gap is a division between two
generations, generally young people and their parents. When parents and children discover they
have little in common, the gap has begun to form. Young people maintain that their parents dont
understand them, worry too much, and restrict their freedom. The parents respond that their
children dont respect them, watch too much television, and want too much freedom at an early age.
In other words, what children and parents respect are(1) vastly different. What are the reasons(2)
for this lack of understanding is hard to say, but there are a few theories.
In previous decades, parents have said that the generation gap developed from young people
challenging their parents old-fashioned ideals and beliefs; however, in the technological
twenty-first century, old and young generations have said that they are separated by different skills
and abilities. These challenges are true for all families, but for immigrant families the problem may
be more serious. The older generation tells to(3) its children that they have lost all sense of tradition
and culture. What do(4) the children learn to value in their new culture is not the same as what do(5)
their traditional parents value.
Children of earlier generations have sworn that they will(6) not let this happen in their
families, yet the gap has continued to exist. That a lack of communication and respect is causing
their children to drift farther and farther from them are(7) shocking to this generation of parents. In
order to slow the formation of the generation gap, it is vital that a child listens(8) to his or her
parents. In addition, counselors suggest that parents are(9) available to listen to their children at all
times. Many families dont know what(10) they will be able to heal the gap between the
generations, but young people and their parents must still try.
1. is
9.be

2.the reasons are 3.

4.

5. 6.would 7.is 8.listen

10.whether

11-40

97 -
I.
1. The pressure of population on available resources is the key to understanding history;
facts is
consequently, any historical writing that takes no congnizance of
flawed.
(A) demographic intrinsically (B) ecological marginally (C) cultural
substantively(D) psychological philosophically (E) political demonstratively
2. Intellectual
and flight from boredom have caused him to rush pell-mell into
situations that less
spirits might hesitate to approach.
(A) restlessness adventurous (B) agitation passive (C) resilience quiescent
(D) tranquility versatile (E) curiosity lethargic
attribute of human behavior, but rather a central virtue,
3. Politeness is not a
one whose very existence is increasingly being
by the faddish requirement
to speak ones mind.
(A) superficial threatened (B) pervasive undercut (C) worthless forestalled
(D) precious repudiated (E) trivial affected
4. Crosbys colleagues have never learned, at least not in time to avoid embarrassing
themselves, that her occasional
air of befuddlement
a
display of her formidable intelligence.
(A) genuine dominates (B) alert contradicts (C) acute precludes
(D) bogus presages (E) painstaking succeeds
5. Every prey to vagrant impulses that impelled him to
his talents on a host of
unworthy projects, his very
nonetheless enhanced his reputation, for the
sheer energy of his extravagance dazzled observers.
(A) undermine enthusiasm (B) isolate selectiveness (C) display affability
(D) squander dissipation (E) implicate genius
6. People frequently denigrate books about recent catastrophes as morally
attempts to profit from misfortune, but in my view our desire for such books, together with
the venerable tradition to which they belong,
them.
(A) inopportune encourages (B) fortuitous fosters (C) treacherous safeguards
(D) despicable legitimizes (E) corrupt generates
7. The demise of the rigorous academic curriculum in high school resulted, in part, from the
progressive rhetoric that
the study of subjects previously thought
as part of school learning.
(A) advocated necessary (B) enhanced indispensable (C) restricted impractical
(D) undermined popular (E) sanctioned inappropriate
caused by danger, hardship, or even cultural
8. In the absence of any
difference, most utopian communities deteriorate into
but enervating
backwaters.
(A) turmoil frantic (B) mistrust nave (C) amelioration ignorant
12-40

(D) decimation intrusive (E) stimulation placid


9. Histocompatibility antigens that attack foreign tissue in the body cannot have been
through evolution expressly to
organ transplantation; on the contrary, they have
been found to facilitate many essential biological functions.
(A) designed retain (B) produced aid (C) developed enhance
(D) selected promote (E) conserved foil
10. A
acceptance of contemporary forms of social behavior has misled a few
into believing that values in conflict with the present age are for all practical
purposes
.
(A) casual reliable (B) superficial trenchant (C) complacent superseded
(D) cautious redemptive (E) plaintive redundant
11. Though many medieval women possessed devotional books that had belonged to their
mothers, formal written evidence of women bequeathing books to their daughters is
and required
scarce, which suggests that such bequests were
no
.
(A) unselfish rationalization (B) tangential approval (C) customary
documentation (D) covert discretion (E) spurious record
12. Although their initial anger had
somewhat, they continued to
the careless worker who had broken the machine.
(A) blazed assail (B) diminished appease (C) abated berate
(D) subsided condone (E) intensified torment
13. Like many eighteenth-century scholars who lived by cultivating those in power,
Winckelmann neglected to neutralize, by some
gesture of comradeship, the
resentment his peers were bound to feel because of his
the high and
mighty.
(A) quixotic intrigue with (B) enigmatic familiarity with (C) propitiatory
involvement with (D) salutary questioning of (E) unfeigned sympathy for
society that worships efficiency, it is difficult for a sensitive and
14. In a
idealistic person to make the kinds of
decisions that alone spell success as
it is defined by such a society.
(A) bureaucratic edifying (B) rational well-intentioned (C) pragmatic
hardheaded (D) competitive evenhanded (E) modern dysfunctional
15. The presidents secretary and his chief aide adored him, and both wrote obsessively
personal memoirs about him; unfortunately, however,
does not make for
true intimacy.
(A) fatuous frankness (B) garrulous confidentiality (C) devoted idolatry
(D) candid discretion (E) rancorous criticism
about the way the building of monasteries proliferated
16. There is something
in eighteenth-century Bavaria, while in the rest of the Western world religious ardor was
and church building was consequently declining.
(A) enigmatic coalescing
(B) destructive changing
(C) immutable
13-40

dissipating(D) incongruous diminishing


(E) momentous diversifying
17. Because they had various meanings in nineteenth-century biological thought,
mechanism and vitalism ought not to be considered
terms; thus, I find
the recent insistence that the terms had single definitions to be entirely
.
(A) univocal erroneous
(B) problematic anachronistic
(C) intractable
obtuse(D) congruent suspect
(E) multifaceted vapid
II.
18. Dotting the marshy expanse of the Florida Everglades are little islands known local as
A
B
C
D
hummocks.
19. Oxacillin, cloxacillin, dicloxacillin, and nafcillin are similar to methicillin in that they are
A
B
resisting to destruction by penicillinase and can be given by mouth.
B
C
D
20. The term Neolithic was used to describe the period when humans began to develop
A
B
C
agriculture as a way of life about 11,000 ago.
D
21. The ballpoint pen, which feed ink to the writing tip from a tube, was first successfully
A
B
C
developed on a commercial scale in the mid-1940s.
C
D
22. The light often seen during a storm travels faster than the rumbling sound is; therefore, we
A
B
C
can see the lighting first.
D
23. Scorpionfish are noted for its several sheathed, venomous spins, which release poison
A
B
C
producing painful, though rarely fatal, puncture wounds.
D
24. Mannerism was an aesthetic movement in that valued highly refined grace and elegance
A
B
the beautiful manner, or style, from which Mannerism takes its name.
C
D
25. By the time some revived womens rights issues emerged again in 1960s, there has been a
A
B
C
continuous sag in the American womens rights movement.
D
26. The residual energy of the sun is so low in the outer solar system that an ocean to exist
A
14-40

would require some other source of energy to keep it from freezing solid.
B
C
D
27. Throwing wastes at random on the streets or dumping sewage into rivers and lakes are a
A
B
C
serous form of pollution.
D
28. Changes in animal behavior may be the result of biological changes, environmental
A
B
factors, or of a combination of either.
C
D
29. To change an astronomical distance measured in parsecs for its equivalent in light years,
A
B
C
one should take the number of parsecs and multiply it by 3.26.
D
30. Afterward the formation of the earth, for several hundred million years the young planet
A
B
was subjected to frequent and ferocious meteor impacts.
C
D
31. The major avenues that radiate out from the White House and the Capitol Building were
A
B
named to commemorate states of that played key roles in the nation-building process.
C
D
32. Objects that of clay are often outstandingly durable, and many have survived thousands of
A
B
years to tell us much about those who made them.
C
D
III.
The New York Times
May 13, 2008
CHENGDU, China A powerful earthquake struck Western China on Monday,
33
thousands of homes, factories and offices, and killing at least 10,000 people, the countrys worst
natural disaster in three decades.
The quake, which was estimated preliminarily to have had a magnitude of 7.9,
34
a
mountainous region outside Chengdu, capital of Sichuan Province, just after lunchtime Monday.
Its tremors were felt as far away as Vietnam and
35
another, smaller quake in the outskirts of Beijing, 900 miles away.
Landslides, power failures and fallen mobile phone towers left much of the affected area
36
from the outside world and limited information about the damage. But
37
of
concentrated devastation suggested that the death toll that could rise significantly as rescue workers
reached the most heavily damaged areas.
15-40

The quake was already Chinas biggest natural disaster since another earthquake
38
the city of Tangshan in eastern China in 1976,
39
240,000 people dead and
40
a severe challenge to the ruling Communist Party, which initially tried to
41
the
castastrophe.
This time, officials quickly
42
50,000 soldiers to help with rescue efforts, state
media said. Prime Minister Wen Jiabao flew to the scene and was shown
43
disaster
response teams from the cabin of his jet.
The prime minister was later shown on national television standing outside the damaged
44
of the Traditional Medicine Hospital in the city of Dujiangyan,
45
encouragement at people trapped in its ruins.
(A) snapshots (B) mobilized (C) posing (D) ravaged (E) edifice (F) leveled (G)
coordinating (H) toppling (I) cut off (J) set off (K) cover up (L) put off (M) shouting
(N) trapping (O) leaving
IV.
The New York Times
December 27, 2004
The location of yesterdays devastating earthquake came as no surprise. Its epicenter lay just
off the western tip of Sumatra in once of the most geologically violent regions on the planet, where
two of the plates
46
. The tremor was the fourth most powerful
quake in the past hundred years, and early estimates put the death toll at 13,000,
47
. Hundreds of thousands of people have been left homeless.
Most of the deaths were caused by tsunamis,
48
. The force of
an earthquake in the midst of a landmass is damped by the land itself,
49
. But this earthquakes epicenter lay under water,
50
, until the tsunamis it generates crash onto shore.
Eyewitnesses reported seeing the classic signs of a tsunami just before disaster struck. The
, which
water receded abruptly from land and then rose in a 40-foot wall,
51
suffered the worst damage.
Its instinctive in humans to search for the meaning of an event like this,
52
. And there will be plenty of meanings to find in the ways
53
.
But except for our obligations to help the victims in any way we can, the underlying story of this
tragedy is the overpowering, amoral mechanics of the earths surface, the movement of plates
54
. Whenever those forces punctuate human history, they do so
tragically. They demonstrate, geologically speaking,

55

(A) how ephemeral our presence is


(B) literally sweeping beaches clean from Thailand to India and Sri Lanka
(C) enormous walls of water generated by the earths sudden movement and flung outward
through the Indian Ocean and the South China Sea
(D) that grind and shift and slide against each other with profound indifference to anything
but the pressures that drive them
16-40

(E) which carries the energy of the quake hundreds of miles


(F) that humans reacted as this disaster struck and in its aftermath as the relief effort begins
(G) a number that is likely to be revised sharply upward in the days to come
(H) once shock and grief have begun to subside
(I) that make up the earths surface collide.
(J) which is why most of the damage occurs near the epicenter
V.
(A) Of the thousands of specimens of meteorites found on Earth and known to science, only about
100 are igneous; that is, they have undergone melting by volcanic action at some time since the
planets were first formed. These igneous meteorites are known as achondrites because they lack
chondrules small stony spherules found in the thousands of meteorites (called chondrites)
composed primarily of unaltered minerals that condensed from dust and gas at the origin of the
solar system. Achondrites are the only known samples of volcanic rocks originating outside the
Earth-Moon system. Most are thought to have been dislodged by interbody impact from asteroids,
with diameters of from 10 to 500 kilometers, in solar orbit between Mars and Jupiter.
Shergottites, the name given to three anomalous achondrites so far discovered on Earth,
present scientists with a genuine enigma. Shergottites crystallized from molten rock less than 101
billion years ago (some 305 billion years later than typical achondrites) and were presumably
ejected into space when an object impacted on a body similar in chemical composition to Earth.
While most meteorites appear to derive from comparatively small bodies, shergottites exhibit
properties that indicate that their source was a large planet, conceivably Mars. In order to account
for such an unlikely source, some unusual factor must be invoked, because the impact needed to
accelerate a fragment of rock to escape the gravitational field of a body even as small as the Moon
is so great that no meteorites of lunar origin have been discovered.
While some scientists speculate that shergottites derive from Io (a volcanically active moon of
Jupiter), recent measurements suggest that since Ios surface is rich in sulfur and sodium, the
chemical composition of its volcanic products would probably be unlike that of the shergottites.
Moreover, any fragments dislodged from Io by interbody impact would be unlikely to escape the
gravitational pull of Jupiter.
The only other logical source of shergottites is Mars. Space-probe photographs indicate the
existence of giant volcanoes on the Martian surface. From the small number of impact craters that
appear on Martian lava flows, one can estimate that the planet was volcanically active as recently
as a half-billion years ago and may be active today. The great objection to the Martian origin of
shergottites is the absence of lunar meteorites on Earth. An impact capable of ejecting a fragment
of the Martian surface into an Earth-intersecting orbit is even less probable than such an event on
the Moon, in view of the Moons smaller size and closer proximity to Earth. A recent study
suggests, however, that permafrost ices below the surface of Mars may have altered the effects of
impact on it. If the ices had been rapidly vaporized by an impacting object, the expanding gases
might have helped the ejected fragments reach escape velocity. Finally, analyses performed by
space probes show a remarkable chemical similarity between Martian soil and the shergottites.
17-40

56. The passage implies which of the following about shergottites?


I. They are products of volcanic activity.
II. They derive from a planet larger than Earth.
III. They come from a planetary body with a chemical composition similar to that of Io.
(A) I only
(B) II only
(C) I and II only (D) II and III only
(E) I, II, and III
57. According to the passage, a meteorite discovered on Earth is unlikely to have come from a
large planet for which of the following reasons?
(A) There are fewer large planets in the solar system than there are asteroids.
(B) Most large planets have been volcanically inactive for more than a billion years.
(C) The gravitational pull of a large planet would probably prohibit fragments from
escaping its orbit.
(D) There are no chondrites occurring naturally on Earth and probably none on other large
planets.
(E) Interbody impact is much rarer on large than on small planets because of the density
of the atmosphere on large planets.
58. The passage suggests that the age of shergottites is probably
(A) still entirely undetermined
(B) less than that of most other achondrites
(C) about 3.5 billion years
(D) the same as that of typical achondrites
(E) greater than that of the Earth
59. According to the passage, the presence of chondrules in a meteorite indicates that the
meterorite
(A) has probably come from Mars
(B) is older than the solar system itself
(C) has not been melted since the solar system formed
(D) is certainly less than 4 billion years old
(E) is a small fragment of an asteroid
60. The passage provides information to answer which of the following questions?
(A) What is the precise age of the solar system?
(B) How did shergottites get their name?
(C) What are the chemical properties shared by shergottites and Martian soils?
(D) How volcanically active is the planet Jupiter?
(E) What is a major feature of the Martian surface?
61. It can be inferred from the passage that each of the following is a consideration in
determining whether a particular planet is a possible source of shergottites that have been
discovered on Earth EXCEPT the
(A) planets size
(B) planets distance from Earth
(C) strength of the planets field of gravity
(D) proximity of the planet to its moons
18-40

(E) chemical composition of the planets surface


62. It can be inferred from the passage that most meteorites found on Earth contain which of
the following?
(A) Crystals
(B) Chondrules
(C) Metals
(D) Sodium
(E) Sulfur
(B) Classical physics defines the vacuum as a state of absence: a vacuum is said to exist in a
region of space if there is nothing in it. In the quantum field theories that describe the physics of
elementary particles, the vacuum becomes somewhat more complicated. Even in empty space,
particles can appear spontaneously as a result of fluctuations of the vacuum. For example, an
electron and a positron, or antielectron, can be created out of the void. Particles created in this
way have only a fleeting existence; they are annihilated almost as soon as they appear, and their
presence can never be detected directly. They are called virtual particles in order to distinguish
them from real particles, whose lifetimes are not constrained in the same way, and which can be
detected. Thus it is still possible to define the vacuum as a space that has no real particles in it.
One might expect that the vacuum would always be the state of lowest possible energy for a
given region of space. If an area is initially empty and a real particle is put into it, the total energy,
it seems, should be raised by at least the energy equivalent of the mass of the added particle. A
surprising result of some recent theoretical investigations is that this assumption is not invariably
true. There are conditions under which the introduction of a real particle of finite mass into an
empty region of space can reduce the total energy. If the reduction in energy is great enough, an
electron and a positron will be spontaneously created. Under these conditions the electron and
positron are not a result of vacuum fluctuations but are real particles, which exist indefinitely and
can be detected. In other words, under these conditions the vacuum is an unstable state and can
decay into a state of lower energy; i.e., one in which real particles are created.
The essential condition for the decay of the vacuum is the presence of an intense electric field.
As a result of the decay of the vacuum, the space permeated by such a field can be said to acquire
an electric charge, and it can be called a charged vacuum. The particles that materialize in the
space make the charge manifest. An electric field of sufficient intensity to create a charged
vacuum is likely to be found in only one place: in the immediate vicinity of a superheavy atomic
nucleus, one with about twice as many protons as the heaviest natural nuclei known. A nucleus
that large cannot be stable, but it might be possible to assemble one next to a vacuum for long
enough to observe the decay of the vacuum. Experiments attempting to achieve this are now
under way.
63. Which of the following titles best describes the passage as a whole?
(A) The Vacuum: Its Fluctuations and Decay
(B) The Vacuum: Its Creation and Instability
(C) The Vacuum: A State of Absence
(D) Particles That Materialize in the Vacuum
(E) Classical Physics and the Vacuum
64. According to the passage, the assumption that the introduction of a real particle into a
19-40

vacuum raises the total energy of that region of space has been cast into doubt by which of
the following?
(A) Findings from laboratory experiments
(B) Findings from observational field experiments
(C) Accidental observations made during other experiments
(D) Discovery of several erroneous propositions in accepted theories
(E) Predictions based on theoretical work
65. It can be inferred from the passage that scientists are currently making efforts to observe
which of the following events?
(A) The decay of a vacuum in the presence of virtual particles
(B) The decay of a vacuum next to a superheavy atomic nucleus
(C) The creation of a superheavy atomic nucleus next to an intense electric field
(D) The creation of a virtual electron and a virtual positron as a result of fluctuations of a
vacuum
(E) The creation of a charged vacuum in which only real electrons can be created in the
vacuums region of space
66. Physicists recent investigations of the decay of the vacuum, as described in the passage,
most closely resemble which of the following hypothetical events in other disciplines?
(A) On the basis of data gathered in a carefully controlled laboratory experiment, a
chemist predicts and then demonstrates the physical properties of a newly synthesized
polymer.
(B) On the basis of manipulations of macroeconomic theory, and economist predicts that,
contrary to accepted economic theory, inflation and unemployment will both decline
under conditions of rapid economic growth.
(C) On the basis of a rereading of the texts of Jane Austens novels, a literary critic
suggests that, contrary to accepted literary interpretations, Austens plots were actually
metaphors for political events in early nineteenth-century England.
(D) On the basis of data gathered in carefully planned observations of several species of
birds, a biologist proposes a modification in the accepted theory of interspecies
competition.
(E) On the basis of a study of observations incidentally recorded in ethnographers
descriptions of non-Western societies, an anthropologist proposes a new theory of
kinship relations.
67. According to the passage, the author considers the reduction of energy in an empty region
of space to which a real particle has been added to be
(A) a well-known process
(B) a frequent occurrence
(C) a fleeting aberration
(D) an unimportant event
(E) an unexpected outcome
68. According to the passage, virtual particles differ from real particles in which of the
following ways?
I. Virtual particles have extremely short lifetimes.
II. Virtual particles are created in an intense electric field.
20-40

III. Virtual particles cannot be detected directly.


(A) I only
(B) II only
(C) III only
(D) I and II only
(E) I and III only
69. The authors assertions concerning the conditions that lead to the decay of the
vacuum would be most weakened if which of the following occurred?
(A) Scientists created an electric field next to a vacuum, but found that the electric
field was not intense enough to create a charged vacuum.
(B) Scientists assembled a superheavy atomic nucleus next to a vacuum, but found
that no virtual particles were created in the vacuums region of space.
(C) Scientists assembled a superheavy atomic nucleus next to a vacuum, but found
that they could not then detect any real particles in the vacuums region of
space.
(D) Scientists introduced a virtual electron and a virtual positron into a vacuums
region of space, but found that the vacuum did not then fluctuate.
(F) Scientists introduced a real election and a real positron into a vacuums region of
space, but found that the total energy of the space increased by the energy equivalent
of the mass of the particles.
(C) Zooplankton, tiny animals adapted to an existence in the ocean, have evolved clever
mechanisms for obtaining their food, miniscule phytoplankton (plant plankton). A very
specialized feeding adaptation in zooplankton is that of the tadpolelike appendicularian who lives
in a walnut-sized (or smaller) balloon of mucus equipped with filters that capture and concentrate
phytoplankton. The balloon, a transparent structure that varies in design according to the type of
appendicularian inhabiting it, also protects the animal and helps to keep it afloat. Water
containing phytoplankton is pumped by the appendicularians muscular tail into the balloons
incurrent filters, passes through the feeding filter where the appendicularian sucks the food into its
mouth, and then goes through an exit passage. Found in all the oceans of the world, including the
Arctic Ocean, appendicularians tend to remain near the waters surface where the density of
phytoplankton is greatest.
70. It can be inferred from the passage that which of the following is true of
appendicularians?
(A) They are exclusively carnivorous.
(B) They have more than one method of obtaining food.
(C) They can tolerate frigid water.
(D) They can disguise themselves by secreting mucus.
(E) They are more sensitive to light than are other zooplankton.
71 .The author is primarily concerned with
(A) explaining how appendicularians obtain food
(B) examining the flotation methods of appendicularians
(C) mapping the distribution of appendicularians around the world
(D) describing how appendicularians differ from other zooplankton
21-40

(E) comparing the various types of balloons formed by appendicularians


72. According to the passage, all of the following are descriptive of appendicularians
EXCEPT
(A) tailed
(B) vegetarian (C) small-sized (D) single-celled
(E)ocean-dwelling
73. The passage suggests that appendicularians tend to remain in surface waters
because they
(A) prefer the warmer water near the surface
(B) are unable to secrete mucus at the lower levels of the ocean
(C) use the contrast of light and shadow at the surface to hide from predators
(D) live in balloons that cannot withstand the water pressure deeper in the ocean
(E) eat food that grows more profusely near the surface
(D) The transplantation of organs from one individual to another normally involves two major
problems: (1) organ rejection is likely unless the transplantation antigens of both individuals are
nearly identical, and (2) the introduction of any unmatched transplantation antigens induces the
development by the recipient of donor-specific lymphocytes that will produce violent rejection of
further transplantations from that donor. However, we have found that among many strains of rats
these normal rules of transplantation are not obeyed by liver transplants. Not only are liver
transplants never rejected, but they even induce a state of donor-specific unresponsiveness in which
subsequent transplants of other organs, such as skin, from that donor are accepted permanently.
Our hypothesis is that (1) many strains of rats simply cannot mount a sufficiently vigorous
destructive immune-response (using lymphocytes) to outstrip the livers relatively great capacity to
protect itself from immune-response damage and that (2) the systemic unresponsiveness observed
is due to concentration of the recipients donor-specific lymphocytes at the site of the liver
transplant.
74. The primary purpose of the passage is to treat the accepted generalizations about
organ transplantation in which of the following ways?
(A) Explicate their main features
(B) Suggest an alternative to them
(C) Examine their virtues and limitations
(D) Criticize the major evidence used to support them
(E) Present findings that qualify them
75. It can be inferred from the passage that the author believes that an important
difference among strains of rats is the
(A) size of their livers
(B) constitution of their skin
(C) strength of their immune-response reactions
(D) sensitivity of their antigens
(E) adaptability of their lymphocytes
22-40

76. According to the hypothesis of the author, after a successful liver transplant, the
reason that rats do not reject further transplants of other organs from the same
donor is that the
(A) transplantation antigens of the donor and the recipient become matched
(B) lymphocytes of the recipient are weakened by the activity of the transplanted
liver
(C) subsequently transplanted organ is able to repair the damage caused by the
recipients immune-response reaction
(D) transplanted liver continues to be the primary locus for the recipients
immune-response reaction
(E) recipient is unable to manufacture the lymphocytes necessary for the
immune-response reaction
77. Which of the following new findings about strains of rats that do not normally
reject liver transplants, if true, would support the authors hypothesis?
I. Stomach transplants are accepted by the recipients in all cases.
II. Increasing the strength of the recipients immune-response reaction can induce
liver-transplant rejection.
III. Organs from any other donor can be transplanted without rejection after liver
transplantation.
IV. Preventing lymphocytes from being concentrated at the liver transplant
produces acceptance of skin transplants.
(A) II only
(B) I and III only
(C) II and IV only
(D) I, II, and III only
(E) I, III, and IV only

23-40

97
I. Multiple Choice
1. It is pretty late, and it is high time we __________ the party.
(A) leave

(B) left

(C) is leaving

(D) will leave

2. The woman is __________ a bankruptcy because she is unable to make both ends meet.
(A) on the verge of (B) on behalf of

(C) in the light of

(D) in amount of

3. __________ arrived at the airport than the plane began to take off.
(A) No sooner had she

(B) She had hardly

(C) Scarcely she had

(D) As soon as had she

4. __________ for the heavy rain, we would have had a nice party.
(A) Has it not been (B) Had it not been (C) Were it not (D) Was it not
5. When I entered the room, I found my grandmother __________ on the sofa.
(A) sit

(B) sat

(C) seated

(D) seating

6. Johns girlfriend insisted on __________ the bill though John had little cash with him.
(A) being paid

(B) paying

(C) having paid

(D) his paying

7. Mr. Brown bought that piece of land near the lake with a view to __________ a summer villa
there.
(A) build

(B) having built

(C) building (D) be built

8. To our surprise, one of the passengers on the plane escaped __________ in the disastrous air
crash.
(A) to be hurt

(B) to have been hurt (C) being hurt

(D) from hurting

9. I intended __________ my sick uncle yesterday afternoon, but the thundershower stopped me.
(A) to call on

(B) calling on

(C) to have called up (D) to have called on

10. When the queens illegal action was discovered, the emperor ordered that she __________ to
death.
(A) be putting

(B) be put

(C) was putting

(D) was put

11. If there were no sunlight, what __________?


(A) did you think will happen
(C) do you think would be happened

(B) do you think would happen


(D) did you think will be happened

12. That was a close call; you __________ hit by the scooter.
24-40

(A) could have been (B) can have been (C) could be (D) can be
13. This camera will cost at least ten thousand dollars.
(A) not more than

(B) not less than

(C) no more than (D) no less than

14. During the past decade, population in the residential area __________.
(A) largely raised (B) greatly added (C) has much added (D) had greatly increased
15. Many visitors to the busy city are not accustomed to the fast pace __________ people move.
(A) that

(B) which

(C) on which

(D) at which

25-40

97
I.Vocabulary & Phrases:
1. Very often, the formation of a barren desert is considered to be caused by a lack of
rainfall. Nevertheless, low ______ alone does not necessarily make an area a desert.
(A) prescription (B) predicament (C) predilection (D) precipitation
2. High fructose corn syrup _____ the need to use more expensive cane sugar, but it is
more fattening.
(A) circumscribes (B) circumvents (C) circumnavigates (D) circumcises
3. A healthy diet will help build an immune system that is stronger and more _____.
(A) rudimentary (B) conspicuous (C) prodigious (D) resilient
4. In the 1980s, Osborne was the first company to do something that was _____ for the
time: making a computer that had a built-in screen.
(A) frenzied (B) consecutive (C) expedient (D) audacious
5. You are ______ warned by your doctor not to eat lobster, which may cause severe
allergic rash all over your body.
(A) considerably (B) specifically (C) contradictorily (D) incredibly
6. Richard Nixon resigned from the presidency to escape the _____ of an impeachment.
(A) indignity (B) candor (C) retrospect (D) elation
7. A _____ of old radio and TV shows can now be purchased over the World Wide Web for
the public to savor the beautiful old days.
(A) myriad (B) paradigm (C) consensus (D) longevity
8. A ban on using trans fats for restaurants in New York would _____ throughout the
country.
(A) reverberate (B) reverse (C) relapse (D) replenish
9. The power grid enables electricity generated in one location to be distributed to distant
areas, but it also allows local damage from storms or _____ to affect the entire system.
(A) vindication (B) proliferation (C) vandalism (D) residue
10. An examination paper in arithmetic can be marked objectively, but a literary essay can
be marked only _______ on the personal impression of the examiner.
(A) submissively (B) subjectively (C) sublimely (D) substantially
11. If you are writing a long research paper, it is a good idea to put a brief _____ at the
beginning that mentions what you are going to discuss.
(A) synthesis (B) hypnosis (C) syringe (D) synopsis
12. The Burmese _______ refused international relief aid offer for the cyclone aftermath
and was denounced for depriving the victims of basic human rights.
(A) regime (B) regimen (C) regiment (D) region
13. Barack Obama remarked, The document they produced was eventually signed but
_____ unfinished. It was stained by this nation's original sin of slavery, a question that
divided the colonies...
26-40

(A) optimally (B) eternally (C) ultimately (D) absolutely


14. The general public were outrageous and decided to boycott the ______ negative
impact on children from cockamamie TV programs.
(A) ubiquitous (B) unsolicited (C) dismal (D) hazy
15. Some people fear that global warming will _____ human life from our planet.
(A) slander (B) annihilate (C) forego (D) renounce
16. Steve Jobs was fired from Apple because he had a _____ with his administrative
companion and the board of directors sided with the other.
(A) falling out (B) looking up (C) coming down (D) breaking up
17. Everyone unconsciously, or rather instinctively, uses body language to ____ their
messages ____.
(A) checkout (B) figureout (C) getacross (D) passout
18. How to _____ the increasing amount of trash in the city has been a big problem to the
city government.
(A) get away with (B) keep up with (C) come up with (D) cope with
19. In the midst of dejection and confusion, something began to ____ himthat he still
loved what he did, so he decided to start over.
(A) happen to (B) flash across (C) dawn on (D) strike on
20. Death is very likely the best invention of life. Its lifes change agent; it clears out the old
to ____ the new.
(A) make up for (B) make way for (C) break up with (D) break down
21. The truffle was prepared with such care that even those who did not have __________
French cuisine found the meal to be delicious.
(A) a preconception of (B) an endorsement of (C) a contention with (D) a penchant for
22. What seemed to be a _________ alternative to fossil fuel turned out to be more
damaging to the environment.
(A) fervent (B) feasible (C) fertile (D) fatal
23. The presidential candidate apologized _________ for her inappropriate remarks on
racial issues, so her support rate went up five points.
(A) capriciously (B) obstinately (C) truculently (D) profusely
24. During his lectures, Professor Campbell speaks as eloquently and effectively as he
writes, thus earning from his students the moniker king of __________ .
(A) debate (B) rhetoric (C) logic (D) metaphor
25. Kathys shy demeanor, unfortunately, was often misinterpreted by those who did not
know her; indeed, strangers typically construed her __________ behavior as coyness.
(A) courteous (B) amiable (C) tenacious (D) demure
26. The miners health had ___________ while he was in custody, and he passed away
the day before the trial.
(A) revived (B) mitigated (C) bolstered (D) deteriorate
II. Cloze Test:
27-40

Like just about every ambitious engineering student at Chinas Tsinghua University in
the early 1980s, Li Sheng had his heart set __27__ the high-tech, high-profile electronics
fieldup until the day he bombed on an electronics exam. But his uncharacteristic
classroom __28__ led Li to a field that could play an even larger role in Chinas future:
energy production. I think the choice was a very fortunate one in the end, said Li __29__
studied thermal engineering and in 2000 became a full professor at TsinghusChinas
MITat the remarkably young age of 35. Energy is incredibly important for a growing
society like China.
But energy means carbon, and Chinas booming economy puts it on a path to become
the worlds No. 1 greenhouse-gas emitter as early as 2020. Li knows that China needs
clean energy __30__ the developed world needs China to clean up, which is why he joined
the Tsinghua-BP Clean Energy Research and Education Center as director when it
opened in July 2003. The centers most promising project is a new technology called
polygeneration, __31__ coal is converted into a cleaner gaseous fuel that can both
generate electricity and be processed into a petroleum substitute. Polygeneration could cut
the carbon emissions China generates by burning its copious coal reserves and reduce its
dependence on oil imports. __32__ his team continues to refine the technologyits still
more expensive than direct cola combustionLi is lobbying the government to construct a
$600 million demonstration plant and he is optimistic he will see it __33__. China is
motivated to develop this technology, Li says. And the rest of the world is hoping it
__34__.
27. (A) in (B) on (C) at (D) to
28. (A) mumble (B) crumble (C) stumble (D) scramble
29. (A) that (B) who (C) , that (D) , who
30. (A) as badly as (B) as soon as (C) as possible as (D) as long as
31. (A) thus (B) by which (C) how (D) where
32. (A) Where (B) Whenever (C) While (D) notwithstanding
33. (A) found (B) building (C) establish (D) constructed
34. (A) does (B) is (C) has (D) did
Each of the psychoactive drugs that you have read so far has a specific chemical
makeup that is known __35__ scientists. __36__, designer drugs are new chemical
combinations that are created to imitate the effects of controlled drugs. Designer drugs are
included in the __37__ drug-control categories, and states also control them.
The most common types of designer drugs imitate stimulants, narcotics, and
hallucinogens. __38__, the designer drug called Ecstasy, or XTC, imitates the effects of
amphetamines and LSD The effects of these new drugs are often more powerful and less
predictable than __39__ of the drugs they imitate. __40__ presents enormous risks to
users. Many cases of permanent brain damage and paralysis have been reported.
35. (A) as (B) for (C) to (D) by
36. (A) In contrast (B) As a result (C) In addition (D) In a different light
28-40

37. (A) provincial (B) federal (C) favorable (D) provisional


38. (A) Take for example (B) Make an example (C) As one example (D) Give an example
39. (A) this (B) that (C) those (D) these
40. (A) This (B) That (C) Which (D) What
III. Reading Comprehension:
If you read a thumbnail biography, in an encyclopedia or on the website, youre liable
to be told that Carl Linnaeus was the father of taxonomythat is, of biological
classificationor that he created the Latin binominal system of naming species, still used
today. Those statements are roughly accurate, but they dont convey what made the man
so important to biology during his era and afterward. You might read that he coined the
name Homo sapiens for our own species and placed us, daringly, within a category of
mammals that included monkeys and apes. Thats true, too, but somewhat misleading.
Linnaeus was no full-blown evolutionist. On the contrary, he heartily embraced the
prevailing creationist view of biological origins, which stipulated that studying nature
reveals evidence for the creative powers and mysterious orderliness of God He wasnt
such a pious man, though, that he sought nothing but godliness in the material world
Heres what makes him a hero for our time: he treasured the diversity of nature for its
theological edification, and he hungered to embrace every possible bit of it within his own
mind He believed that humankind should discover, name, count, understand, and
appreciate every kind of creature on earth.
To assemble all that knowledge, two things were required: tireless and acute
observation, and a system.
th

In spring of 1732, just before his 25 birthday, Linnaeus set off on an expedition. Over
the next five months he traveled some 3,000 miles, by horseback and foot and boat,
making collections and taking notes as he went. He was interested in everythingbirds,
insects, fish, geology, the customs and technology of the Sami peoplebut especially in
plants. He made drawings in his journal, some of which were crude sketches, some of
which (again, those of plants) were delicate and lovingly precise. Eventually, he produced
a book, Flora Lapponica, describing the botanical data he had gathered.
41. Which of the following about Carl Linnaeus is TRUE?
(A) He was skeptical of the creative powers and mysterious orderliness of God
(B) He was a full-blown enthusiast of Darwins Theory of Evolution.
(C) He put human beings within the same category with apes and monkeys.
(D) He was so pious that he sought nothing but godliness in the material world
42. Why did the author write this passage since there are already a lot about Carl Linnaeus
in biographies, in an encyclopedia or on the website?
(A) He meant to clarify some misunderstandings and contradictions in these
sources of information about Carl Linnaeuss achievements.
(B) He reckoned Thumbnail biographies, encyclopedias, or websites only give
29-40

factual information, which is insufficient to illustrate how much effort Carl


Linnaeus put into his work.
(C) He meant to use Carl Linnaeus as an example for modern scientists to
follow in doing the groundwork for scientific research.
(D) He wanted to put emphasis on the importance of the integration of
scientific research and theological edification in accomplishing great
discoveries.
43. What made Carl Linnaeus so important during his era and for our time?
(A) He contributed wholeheartedly to the prevailing creationist view of biological origins,
which stipulated that studying nature reveals evidence for the creative powers and
mysterious orderliness of God
(B) He is the father of taxonomythat is, of biological classification and he traveled
some 3,000 miles to gather and assemble all the knowledge of every living
creature in nature
(C) He coined the name Homo sapiens for our own species and placed us, daringly,
within a category of mammals that included monkeys and apes.
(D) His tireless and acute observation of the diversity of nature to construct a system
to name and appreciate it for theological edification.
In March, 2001, Fidel Castro signed an agreement with South Africas President
Thabo Mbeki to promote the development and distribution of the cheaper alternatives to
the AIDS drugs marketed by U.S. pharmaceutical corporations, which are usually
overpriced beyond the reach of most AIDS patients in the developing world Castro also
promised to put Cubas pharmaceutical industry to work on helping countries such as
South Africa and Brazil manufacture generic drug treatments patented by U.S. and
European pharmaceutical companies.
Health ministers from developing countries such as India, Indonesia and Iran also
backed South Africas legal battle against 39 drug companies over the right to sidestep
patents by importing generic drugs to fight the AIDS crisis. Challenging AIDS drug patents
is fast emerging as the defining issue of a new Third World solidarity.
Intellectual property has been known to arouse considerable skepticism in the
developing worldparticularly when Western corporations set out to patent traditional
remedies used in those countries for centuries. When it is against a humanitarian crisis
threatening the lives of millions of people, the sanctity of intellectual property is far from
assured While the U.S. may have managed to cope with industrial-scale piracy of CDs and
movies in the developing world, life-saving medicine is another matter. For example,
companies in India and Brazil have copied AIDS treatments and distributed them in the
developing world at a fraction of the cost charged in the West for the brand-name product.
Whatever the outcome of the resulting disputes in the courts and at the World Trade
Organization, the moral weight of life-saving necessity may have already won the case in
the court of public opinion.
30-40

44. What is the main idea of this article?


(A) Explaining why Fidel Castro supported intellectual property and advocated generic
AIDS drugs.
(B) Pointing out why the Western industrialized countries have failed to stop alternative
AIDS drugs.
(C) Providing the reasons why AIDS patients in developing countries are not being
taken care of.
(D) Juxtaposing two opposing attitudes about AIDS patent drugs and the opinions of
the general public
45. Which of the following countries may NOT be a member of the new Third World
solidarity?
(A) Sweden (B) India (C) Indonesia (D) Cuba
46. What is the tone of the author?
(A) pessimistic (B) nonchalant (C) passionate (D) concerned
47. What does the author imply in the last sentence of the third paragraph?
(A) The general public is sick of avaricious drug companies that make profit off of the
vulnerable.
(B) Most people consider it more important to help those AIDS patients in developing
countries.
(C) The World Trade Organization will adjudicate the disputes over generic AIDS
drugs.
(D) Developing countries have won the case of patented AIDS drugs made in Brazil
and India
48. What is the closest meaning of sidestep?
(A) evade (B) tread (C) succumb (D) demand
Occupations encourage gender differences among workers in many ways, and one of
the most pervasive is internal stratification. That is to say, men and women in the same
occupation often perform different tasks and functions. Even in those occupations that
appear sexually integrated, the aggregate statistics often mask extreme internal
segregation. Although
the number of female bakers increased from 25 percent in 1970 to 41 percent in 1980,
for example, the majority of female bakers are found in highly automated baking industries,
while their male counterparts are located in less automated bakeries. The same
phenomenon has been detected among pharmacists, financial managers, and bus
driversall groups where the influx of women workers suggests a diminution of sex
segregation.
Another strategy used to maintain gender differences in integrated occupations is the
use of behavioral and etiquette rules. When women enter male-dominated occupations,
certain rules are often introduced to govern their dress and demeanor. In office settings, for
instance, dress codeseither formal or implicitare not unusual; female employees may
31-40

be required to wear dresses, nylons, and high-heeled shoes to enhance their femininity. So
it is for female marines and male nurses, both of whom are required to dress differently
from their male and female counterparts. Male nurses never wear the traditional nursing
cap; female marines never sport the standard Marine Corps garrison cap.
Informal practices also play a role in constituting femininity in female marines and
masculinity in male nurses. As members of visible minority groups, they stand out at work
and receive far more than their fair share of attention. This phenomenon was first
documented by Rosabeth Moss Kanter, who found that women in corporations, simply by
virtue of their numerical rarity, were noticed and scrutinized more than their male
counterparts. This added pressure may actually result in different job performance from
men and women in nontraditional occupations and worsen gender differences. Kanters
corporate women, for example, became more secretive, less independent, and less
oppositional in response to their greater visibilityall traits that have traditionally been
associated with femininity.
Another informal technique that enhances gender differences is practiced by
supervisors who evaluate men and women differently. The very qualities that are highly
praised in one sex are sometimes denigrated in the other. Thus, a man is ambitious, a
woman pushy; a woman is sensitive, a man, wimpy.
But it would be a mistake to claim that all gender differences are forced on people. In
addition to the external pressures I have just described, male nurses and female marines
actively construct their own gender by redefining their activities in terms of traditional
masculine and feminine traits. For example, women in the Marine Corps insist that their
femininity is intact even as they march cadence in camouflage units. Likewise, male nurses
contend that their masculinity is not at all threatened while they care for and nurture their
patients.
49. The author is mainly concerned about
(A) explaining how femininity and masculinity can be reconstructed for specific careers.
(B) examining jobs that, at first glance, seem to be non-conventional for men and
women.
(C) exploring the reasons why gender differences cannot be ignored in any occupation.
(D) discussing the pervasiveness of gender segregation in the work place despite the
appearance of integration.
50. The author suggests which of the following about internal stratification?
(A) Although women now work in industries once dominated by men, they find it
comparatively more difficult to be promoted to managerial positions.
(B) As men and women rarely engage in the same activities on the job, certain
specialties can be feminine-identified and others masculine-identified.
(C) Even when men and women are given the same tasks to perform, women continue
to receive less pay than their male counterparts.
(D) Since men and women are segregated in the workplace, men tend not to value the
work carried out by women, but not the other way around.
32-40

51. Which of the following best describes the organization of the first paragraph?
(A) A hypothesis is provided, evidence to support it is given, and its implications are
discussed.
(B) An observation is made, specific situations that are applicable are cited, and a
generalization is derived
(C) A generalization is made, a clarification is put forth, and specific examples are
offered in support.
(D) A specific case is presented, its particulars are analyzed, and a conclusion is drawn
from it.
52. The author specifically mentions all of the following as methods to maintain gender
differences in the workplace EXCEPT
(A) pressure from coworkers to behave in a conventionally masculine or feminine.
(B) the internal pressure men and women feel to be traditionally masculine or feminine.
(C) the designation of dress codes so that the physical differences between men and
women are highlighted.
(D) a managers use of particular words for men and particular words for women
although describing the same quality.
53. What is the closest meaning of wimpy?
(A) weak (B) sophisticated (C) calculating (D) dominating
Smooth muscles in arteriole walls can influence blood pressure by changing the
resistance to blood flow out of the arteries and into arterioles. The smooth muscles in the
arteriole walls also regulate the distribution of blood to the capillaries of the various organs.
At any given time, only about 5-10 percent of the bodys capillaries have blood flowing
through them. However, each tissue has many capillaries, so every part of the body is
supplied with blood at all times. Capillaries in a few organs, such as the brain, heart,
kidneys, and liver, usually carry a full load of blood, but in many other sites, the blood
supply varies as blood is diverted from one destination to another, depending on need.
In addition to the smooth muscles that can constrict or dilate an arteriole leading into a
capillary bed, a second mechanism regulates the distribution of blood. A capillary called a
thoroughfare channel, through which blood streams directly from arteriole to venule. This
channel is always open. Capillaries branching off from thoroughfare channels form the bulk
of the capillary bed. Passage of blood into these branching capillary is regulated by rings of
smooth muscle called precapillary sphincters. Blood flows through a capillary bed when its
precapillary sphincters are relaxed. It bypasses the bed when the sphincters are
contracted. After a meal, for instance, precapillary sphincters in the wall of the digestive
tract let a larger quantity of blood pass through the capillary beds than when food is not
being digested. During strenuous exercise, many of the capillaries in the digestive tract are
closed off, and blood is supplied more generously to skeletal muscles.
The contraction of the smooth muscles in both these mechanisms is under the
influence of nerves and hormones.
33-40

54. What is the passage mainly about?


(A) How blood circulation is determined by nerves and hormones.
(B) The difference between the blood supply for the organs and that for other parts of
the body.
(C) How smooth muscle controls the distribution of blood.
(D) How blood is transported from the blood vessels to the organs in our body.
55. What two mechanisms control the distribution of blood to the capillary beds of the
body?
(A) Constriction or dilation of an arteriole and relaxation of precapillary sphincters.
(B) Constriction or dilation of an venule and contraction of precapillary sphincters.
(C) Dilation or constriction of an venule and relaxation of precapillary sphincters.
(D) Dilation or constriction of an venule and contraction of thoroughfare channels.
56. In which of the following situation is blood distributed to the capillary beds of the body?
(A) During strenuous exercise. (B) When a person is very hungry.
(C) When food is being digested. (D) When the precapillary sphincters are contracted.
57. This passage is most likely to be found in ______.
(A) New York Times. (B) The Economist.
(C) Fortune. (D) Scientific American.
You all know what a wave isat least until you have to write a paragraph defining one,
illustrating once again that physics is as much language as mathematics. In the science it
is often quite difficult to generalize or explain precisely what is meant by our observations
using the deep and common pool of words from the well of our linguistic heritage.
We need a paradigm for a wave that encompasses the many varieties of waves that
occur in nature. If you think of graceful ocean swells, or the ripples generated when you
drop a pebble into a calm pond, it is apparent that water waves are an oscillating
disturbance that moves, we say propagates, across the water surface. When such waves
make contact with you or a canoe floating in the water, it is evident that they give or
transfer energy and momentum to you or the canoe. A wave involves energy and
momentum transport at a characteristic speed that has little direct relationship to the
speeds of the entrained particles. In a water wave well away from the beach, the dominant
motion of an individual water particle is up and down, perpendicular to the direction the
water wave is moving. Such waves are much more complex as they break near a beach!
A propagating wave disturbance may involve any manner of physical quantity. A
mechanical wave is a propagating disturbance in and of a material (such as a solid, liquid,
or gas) from its equilibrium configuration. The term mechanical wave indicates its close
association with matter. These waves involve oscillatory motions of the atoms and/or
molecules that make up the material, without requiring net transport of the material itself
over long distances in the direction of motion of the wave. Such mechanical waves
represent propagating oscillations from equilibrium, be they departures from equilibrium
positions (e. g. water waves and acoustic waves), or pressure or density oscillation (also
34-40

acoustic waves). An ocean wave predominantly moves water up and down with little or no
average motion of the water along the direction of travel of the wave. A huge locomotive
engine generates mechanical waves (vibrations and sound waves) in the steel rails that
propagate speedily along their length; although the particles in the rail move slightly, the
rail itself does not progressively move as do the waves.
Other types of waves, capable of propagating even through a vacuum, involve
propagating oscillations of fields of various kinds: gravitational fields (gravity waves), or
electric and magnetic fields (electromagnetic wavesi.e., light). Such waves are not
mechanical waves.
58. Which of the following is the closest in meaning to the word propagates in the
second paragraph?
(A) oscillates (B) progresses (C) ripples (D) moves
59. The purpose of the above passage is mainly to _____.
(A) explain how tsunamis occur (B) demonstrate the linguistic characteristics of a wave
(C) generalize what a wave is meant (D) describe the movement of a wave
60. Which of the following is TRUE?
(A) Waves move by transferring energy, momentum and mass at a characteristic
speed.
(B) Water waves and acoustic waves are mechanical waves.
(C) Mechanical waves involve transport of the material over long distances in the
direction of motion of the wave.
(D) Since there is a deep pool of vocabulary, its not hard to generalize precisely what
is meant by our observations.
IV. Writing
1. What do you think are the major problems of English learning and teaching in Taiwan?
What ideas and measures do you plan to adopt to cope with these problems when
teaching?
2. What is critical thinking? How do you incorporate it into EFL teaching?
3. When there are students of various English proficiency levels in one class, how do you
teach English to meet each students needs?
4. How do you improve students speaking in a class of forty or more students?
5. What are good sources of outside reading materials that you could use in class? Why
should outside reading materials be used? What are the constructive ways of teaching
these?
6.Write an original passage with at least SEVEN phrases out of the thirteen
phrases in the PHRASE-BANK section and create a test for with the
phrases you choose (the phrases have been numbered for you).
Please attach the answer key to each blank just below the passage which you create for

Part of the grade here is given based on the number of the phrases you use; that is, the
35-40

more given phrases you use, the higher grade will probably result
PHRASE-BANK:
A run into B stand out C let go of D let on E. do without F. lay off G. set out
H. set ones heart on I.figure out J.give it up K.show up L.dress up M.stop off

97

1
D
11
D
21
D

2
B
12
A
22
B

3
D
13
C
23
D

4
D
14
A
24
B

5
B
15
B
25
D

31
B
41
D
51
C

32
C
42
B
52
A

33
D
43
D
53
A

34
A
44
D
54
C

35
C
45
A
55
A

6
A
16
A
26

36
A
46
C
56
C

36-40

7
A
17
C
27
B

8
A
18
D
28
C

9
C
19
C
29
D

10
B
20
B
30
A

37
B
47
B
57
D

38
C
48
A
58
D

39
C
49
D
59
C

40
A
50
B
60
B

37-40

38-40

39-40

97

97

50 2 100

I. Vocabulary, Phrases and Grammar: Choose the most appropriate answer.

1. Thousands of bodies are still buried under the _________.


rabble
 rubble
 bubble
 dabble
fact that whatever exists must have a
(B)
on which to stand, a
(C)
from which to grow, and a
2. It is a
(A)
on which to turn. The words in (A) (B) (C) (D) in the above sentence are _________ respectively.
(D)
root, hinge, patent, basis
 hinge, patent, basis, root  patent, basis, root, hinge  basis, root, hinge, patent
3. They went ahead, public opposition _________.
nought
 spite
 yet
 notwithstanding
4. _________, derived from Greek words literally meaning fear of the marketplace, involves the fear of open or public spaces.
Claustrophobia
 Hydrophobia
 Agoraphobia
 Xenophobia
5. The results of the experiment were _________ not only because the results challenged old assumptions but also because they called
the _________ methodology into question.
predictable; contemporary  inconclusive; traditional
 provocative; prevailing
 interesting; projected
6. Most anesthetics put the patients to _________.
delight
 pain
 sleep
 awakening
7. Most people in the world hope to see peace prevail and expect to live in _________.
enmity
 dimity
 enemy
 amity
8. A weak voice is a serious _________ to a stage actors career.
sense
 discretion
 detriment
 optimum
9. In high school, Greg was _________(e)d because of his political views, but in college he found many people who shared his opinions.
reiterate
 mandate
 recourse
 ostracize
10. Not to mention _________, even food and clothing are not to be got without the toil of the hands and the sweat of the brows.
rich and honor
 the rich and the honor
 the riches and honor
 riches and honor
11. The policemen riding _________ broke up the demonstration.
on the back of horse
 on horseback
 on the horseback
 horseback
12. The court annulled the marriage because it was _________.
legitimate
 illegal
 fertile
 pleasant
13. The New York telephone directory lists hundreds of different _________s, including action groups for Korean-American Voters,
Fairness to Africa, and A Smoke-Free City.
commensurate
 coalition
 chide
 scenario
14. Jenny didnt like the apartment with the old-fashioned tub and radiators. She preferred a more _________ place.
contemporary
 poignant
 traumatic
 extrovert
15. In Tokyo, space is so limited that most apartments and homes are very small. The _________ of some bedrooms are barely larger than
a closets.
dimensions
 distort
 sequence
 lament
16. Two loads of woolen cloth were _________(e)d to the factory last Monday.
attest
 mobile
 orient
 dispatch
17. The teacher and the other students became irritated when Sue kept asking questions that werent _________ to the class discussion.
peevish
 plight
 germane
 symmetrical
18. A lot of memories are _________(e)d in this photograph album.
gobble
 spurt
 enshrine
 butt
19. When children get into a fight, it is sometimes best not to _________, but to let them work it out themselves.
intervene
 inept
 lament
 revoke
20. He is writing to the Principal _________ the nomination of this years medal.
regarding of
 regard to
 as regards
 as regarding
21. Three _________ sugar made her coffee tasteless.
spoonful of
 spoonfuls of
 spoons
 spoon
22. Thomas Carlyle, a famous _________, declared that the British would rather lose India than the writings of Shakespeare.
man in letters
 man of letter
 man in letter
 man of letters

23. The mansion _________ six million dollars.


is worth
 is worth of
 is worthy
 is worthwhile
24. The teacher said that the child _________ confidence.
was lacked
 lacked
 lacked in
 lacked of
25. I decided to retrace my steps, and there at the bike rack _________ my tote bag along with my bicycle.
X
 was
 were
 to be
26. The guests here are _________ German and Japanese, but _________ we get a few Americans, too.
mostly; never
 most; never
 mostly; sometimes
 most; sometimes
27. When philosophers debate what it is _________ makes human unique among animals, they often point to language.
that
 who
 which
 whom
28. _________ was made of minute particles called corpuscles was believed by scientists.
Light
 That light
 As light
 Whereas light
29. He is 70, but most people expect him, health _________, to seek a second term of office in 2008.
permitting
 permitted
 permits
 to permit
30. During the early period of ocean navigation, _________ any need for sophisticated instruments and techniques.
so that hardly
 when there hardly was
 hardly was
 there was hardly
31. To ignore the past or the future is _______ folly, at worst wickedness.
at least
 at times
 at most
 at best
32. He said, Ill go, _________.
rains or shines
 rain or shine
 raining or shining
 as rain as shine
33. Geysers are found near rivers and lakes, where water drains through the soil _________.
surface below the deep
 deep below the surface
 the deep below surface
 the deep surface below
34. Lydia was apprehensive about going to an Indian restaurant because she _________.
enjoyed all Indian foods
 was afraid the food might be too spicy for her
 was curious about new foods
 didnt have to work that day
35. My mother considered baked custard a panacea. According to her, it _________.
was the worst-tasting thing in the world
 cured anything from flu to a broken heart
 should be saved for special occasions
 tasted good, but was not good for us
In questions 36-41 each sentence has four underlined words or phrases, marked , , , and . Identify the one
underlined word or phrase that must be changed in order for the sentence to be correct.

II. Error Correction:

36. The teacher suggested that she be election as the model student in the class.

 

37. Most decisions concerning practical matters need to be made on the base of incomplete information.

38. She informed to her boss that she needed a larger office.

39. Researchers are investigating a series of indicators that could help themselves predict earthquakes.

40. Fungi are essential in the process of decay, which returns ingredients to the soil, enhances soil fertility, and decompose animal debris.

41. In 1974, farmers were digging a well by the tomb in Xian, China, and unexpectedly found thousand of terra cotta soldiers.

III. Reading comprehension: Please select one best answer to each of the following questions based on the passages.
Reading #1 (Questions 42-45)
In the past century, there has been a technological explosion, largely in the domain of transportation and information, that makes the
interactions of a print-dominated world seem as hard-won and as easily erased as the print revolution made earlier forms of cultural traffic
appear. For with the advent of the steamship, the automobile, the airplane, the camera, the telephone, and the computer, we have entered
into an altogether new condition of neighborliness, even with those most distant from ourselves. Marshall McLuhan, among others, sought
to theorize about this world as a global village, but theories such as McLuhans appear to have overestimated the communitarian
implications of the new media order. We are now aware that with media, each time we are tempted to speak of the global village, we must
be reminded that media create communities with no sense of place. The world we live in now seems rhizomic, even schizophrenic,
calling for theories of rootlessness, alienation, and psychological distance between individuals and groups on the one hand, and fantasies
(or nightmares) of electronic propinquity on the other. Here we are close to the central problematic issue of cultural processes in todays
world.
42. According to the author, the technological explosion that took place a century ago can be equated with _________.
cultural revolution
 print revolution
 military revolution
 religious reform
43. In the passage, an altogether new condition of neighborliness refers to _________.
a condition of sociability
 a condition of borderline  a condition of renewal
 a condition of hostility
44. The word propinquity means _________.
mystery
 addiction
 threat
 accessibility
45. What can be inferred from the passage?
The author totally agrees with Marshall McLuhans theorization of global world order.
 The author partially agrees with Marshall McLuhans theorization of global world order.
 The author totally disagrees with Marshall McLuhans theorization of global world order.
 The author disregards Marshall McLuhans theorization of global world order.
Reading #2 (Questions 46-50)
is Jonathan Swifts most universal satire. Although it is full of allusions to recent and contemporary historical events,
it is as valid today as it was in 1726, for its objects are mans moral nature and the defective political, economic, and social institutions
which, because of his imperfections, he creates. Swift adopts an ancient satirical device: the imaginary voyage. Lemuel Gulliver, the
narrator, is a ships surgeon, a reasonably well-educated man, kindly, resourceful, cheerful, inquiring, patriotic, truthful, and rather
unimaginative. He is, in short, a reasonably decent example of humanity, with whom we identify ourselves readily enough. He undertakes
four voyages, all of which end disastrously among several remote nations of the world.
When
first appeared, everyone read itchildren for the story, politicians for the satire of current affairsand ever
since it has retained a hold on readers of every kind. Almost unique in world literature, it is simple enough for a child, and complex enough
to carry an adult beyond his depth. Swifts art works on many levels. First of all, there is the sheer playfulness of the narrative. Through
Gullivers eyes, we gaze on marvel after marvel: a tiny girl who threads an invisible needle with invisible silk, or a white mare who threads
a needle between pastern and hoof. The travels, like a fairy story, transport us to imaginary worlds that function with a perfect, fantastic
logic different from our own; Swift exercises our sense of vision. But beyond that, he exercises our perceptions of meaning. In
, things are seldom what they seem; irony, probing or corrosive, underlies almost every word.
Gullivers Travels

Gullivers Travels

Gullivers

Travels

46. The word defective means _________.


perfect
 complex
 menacing
 flawed
47. Which of the following features is not a characteristic of Lemuel Gulliver?
Illiterate.
 Inquisitive.
 Realistic.
 Knowledgeable.
48. Which of the following statements is true?
depicts the everyday practice well-known to its readers.

is now considered outdated in terms of the materials and issues it addresses.

appears to be accessible and intriguing to adults and children alike.

remained unnoticed when it first appeared.
49. The word satirical means _________.
comical
 mocking
 mad
 melancholic
50. What can be inferred from the passage?
Gulliver undertakes four eventful voyages and dies tragically.
 Readers will find it hard to identify themselves with the narrator protagonist.
 Because man is imperfect, the political or social institutions he establishes are likewise unsatisfactory.
 The novelist intended to arouse readers antipathy toward imagination.
Gullivers Travels
Gullivers Travels
Gullivers Travels

Gullivers Travels

97

1.

2.

3.

4.

5.

6.

7.

8.

9.

10.

11.

12.

13.

14.

15.

16.

17.

18.

19.

20.

21.

22.

23.

24.

25.

26.

27.

28.

29.

30.

31.

32.

33.

34.

35.

36.

37.

38.

39.

40.

41.

42.

43.

44.

45.

46.

47.

48.

49.

50.

97

50. Advances in technology occur at such a fast pace that


dictionaries have difficulty incorporating the __________

40 1.5 60
I. Vocabulary
41. Christina said she would _________ the want ads every
day until she found the job for which she was qualified.
(A) apprehend
(B) stipulate
(C) scrutinize
(D) ventilate
42. My brother is as __________ as a mule. He never
listens to people, even when theyre giving really good
advice.
(A) obstinate
(B) gullible
(C) extrovert
(D) diffident
43. Dr. Wang is considered one of the most outstanding
researchers and __________ academic authors in our
country. He has published numerous papers on
international journals.
(A) industrial
(B) prolific
(C) hideous
(D) frantic
44. The decision is based on the __________ that all
information must be freely available for all.
(A) stimulation
(B) resumption
(C) consumption
(D) presumption
45. In 1766 colonial leader George Washington began the
__________ process of turning farmers into a disciplined
combat force for the Revolutionary War.
(A) ardent
(B) arduous
(C) apathetic
(D) architectural
46. A string of doctors agreed she was suffering from Lyme
disease and prescribed antibiotics, but that did nothing to
__________ her pain.
(A) alleviate
(B) engross
(C) excel
(D) ascribe
47. The live performance was nothing but a ________ the
lights wouldnt work, one actor forgot his lines and
another fell off the stage.
(A) trap
(B) debauchery
(C) fiasco
(D) paradox
48. Being a teacher, you are supposed to use your sense of
humor to __________ a difficult situation or teach an
important lesson.
(A) congeal
(B) depute
(C) comprehend
(D) defuse
49. Human guts are full of bacteria. These __________ bugs
help digestion, and also stop their disease-causing
counterparts from invading.
(A) stilted
(B) liberal
(C) synchronous
(D) intestinal

that emerge as names for new inventions.


(A) neologisms
(C) euphemisms

(B) clichs
(D) compensations

51. In painting, it is important to always place your strongest


color first; otherwise the overlapping color will _____
the color underneath.
(A) penetrate

(B) replicate

(C) obliterate
(D) fluctuate
52. Although he claimed that he was innocent, the witnesss
____________ proved that he was guilty.
(A) contention
(C) context

(B) testimony
(D) verdict

53. The __________ among the worlds scientists is that


global warming will have a big impact on our
environment.
(A) tendency

(B) essence

(C) distinction
(D) consensus
54. The sales clerk asked the customer to __________ his
identity by showing her a drivers license with a photo.
(A) verify
(C) nullify

(B) specify
(D) qualify

55. Animals seem to sense when an earthquake is


__________. Their behavior changes just as the disaster
is about to occur.
(A) imminent

(B) deplorable

(C) invincible
(D) discrete
56. The plants and the animals that were buried underground
began to ____________.
(A) depose
(C) decompose

(B) compose
(D) dispose

57. The natural resources will soon be ____________ if


people still waste them.
(A) deleted

(B) destroyed

(C) depressed

(D) depleted

58. She was so fed up with her ____________ office work


that she decided to take up the job as a tour guide for a
travel agency.
(A) sedentary
(C) reproductive

(B) synchronic
(D) interactive

59. Those people who are successful tend to have some


common ____________. For example, they usually have
unbending will power and they dont give up easily.
(A) distributes

(B) contributes

(C) attributes

(D) tributes

5 3

60. The government will ____________ strict regulations


concerning the handling of the radioactive waste after the
protest from the public.
(A) impose
(B) import
(C) impark
(D) impair
61. Avian flu is a ____________ disease. It passes from
birds to animals and people.
(A) contagious
(B) contemplative
(C) continuous
(D) conflicting
62. A(n) ____________ is a doctor who treats patients with
cancer.
(A) cardiologist
(B) urologist
(C) oncologist
(D) psychologist
63. Each summer, an ____________ of tourism makes the
city boisterous.
(A) influx
(B) input
(C) indulgence
(D) influence
II. Cloze
(A)
The mixed population of Toni Morrisons hometown,
Lorain, consisting of Southern blacks, immigrant
Europeans, and Mexicans, provided fertile ground for
racial friction and violence. When she was two, her
family home was set on fire with the family inside. This
incident intensified her fathers __64__ deeply distrustful
attitude toward whites. These early influences, her
familys reverence for black culture and storytelling
__65__ experiences creating a basic mistrust of white
culture, were evidently catalysts for the literature
Morrison __66__ author as an adult. As Morrison
herself observes, there are few white characters in her
novels. Her stories do not describe relationships between
blacks and whites. The fictional universe she creates
seeks __67__ to create awareness and respect for the
black experience by focusing on relationships __68__
the black community.
64. (A) so
(B) such
(C) still
(D) already
65. (A) combining with
(B) combined with
(C) combining to
(D) combined to
66. (A) will
(B) may
(C) would
(D) might
67. (A) still
(B) even
(C) never
(D) instead
68. (A) of
(B) within
(C) among
(D) between
(B)

Stem cell research is regarded as one of the most


outstanding biological advances in modern times.
Proved to have the remarkable potential to develop into
many different cell types in the human body, stem cells
can act as a repair system for the body. __69__ the
body is affected with diseases and the original body cells
die off, the stem cells play their parts. That is to say,

not only do stem cells divide without limit to __70__


other cells that had disappeared due to illnesses, they are
also either to remain a stem cell or become another type
of cell with a more specialized function, such as a
muscle cell, a red blood cell, or a brain cell. There are
three major categories of mammalian stem
cells-embryonic stem cells, adult stem cells, and cord
blood stem cells. Among these three types, it is
believed that embryonic stem cells rather than adult
ones are __71__ because they offer great potential to
replace tissues damaged by disease. For instance, a
British scientific journal recently released stem cell
studies declaring UK scientists has successfully extracted
stem cells out of rat or mouse embryos. The epiblast
stem cells, __72__ they have been named, are taken
from the rat or mouse embryos at a slightly later stage of
its development than previous mouse embryonic stem
cells. Most important of all, the epiblast stem cells can
serve as the missing link between mouse and human
embryonic stem cells, because they are more similar to
human embryonic stem cells than to mouse embryonic
stem cells. Stimulated by these findings, stem cell
experts are now devoting themselves __73__ the stem
cells aid research in cures for disease ranging from heart
disease to burns.
69. (A) Before (B) Once
(C) Whether (D) Although
70. (A) measure (B) outstare (C) replenish (D) trudge
71. (A) the most useful
(B) more useful
(C) useless
(D) the least useful
72. (A) because (B) as
(C) when
(D) since
73. (A) to use (B) to using (C) to having (D) to have
III. Reading Comprehension
(A)
Moralists have long argued that a major reason for
our burgeoning violent crime rate is that allegedly, fewer
parents are spanking their offspring. Paradoxically,
virtually every study ever made into the backgrounds of
violent criminals has shown that they are far more likely
than law-abiding citizens to have been subjected to a
great many beatings and other forms of cruel physical
punishments by their parents and other adult guardians.
These same data have also shown that besides a great
deal of physical punishment, violent criminals in almost
all cases have never received any genuine warmth or
respect as human beings from their families. Those in
positions of authority had always been cruel, callous and
indifferent to them. It was hence very easy for them to
turn around and react toward society in kind.
Children who are often spanked tend to be
conspicuously quieter, less articulate and more sullen

5 4

than those who grow up under milder, more democratic


forms of discipline. In addition, harshly disciplined
offspring tend to display a large amount of negativity in
their approach to people and to life. Such traits as
negativity and sullenness are a good deal more common
among boys in our culture than among girls. Since
almost all studies on discipline in the home have shown
that boys tend to be the target of more physical
punishment and psychic humiliation than girls, this is not
surprising. And it may partially account for the greater
openness of girls to classroom learning, particularly
during the years of middle childhood. It may also be one
of the factors responsible for the diminished ability of
males in this society to express their feelings and
emotions. The ability to articulate, the attitudes toward
life and personalities all seem to be influenced by ways
of discipline.
74. What do the moralists believe?
(A) The crime rate is decreasing.
(B) Parents should not punish children physically.
(C) Violent criminal behaviors result from physical punishment
at childhood.
(D) Violent criminal behaviors result from lack of physical
punishment at childhood.
75. What does research conclude in relation to violent
criminals?
(A) They are inherently more violent than other people.
(B) They commit crimes as a reaction to the unkind
attitude of other people.
(C) They do not necessarily receive more severe beating
from their parents than other people.
(D) Their family members refuse to show their love and
warmth after the violent behaviors.
76. What are the characteristics of children who are punished
by their parents?
(A) eloquent
(B) taciturn
(C) eminent
(D) reverend
77. According to the author, what is true for children,
particularly during their middle childhood?
(A) Boys receive less attention in classroom.
(B) Boys receive equal amount of punishment as girls.
(C) Boys become more open-minded than girls.
(D) Boys become less willing to share their thoughts and
emotions.
(B) Humans have struggled against weeds since the
beginning of agriculture. Marring our gardens is one of
the milder effects of weeds any plants that thrive where
they are unwanted. They clog waterways, destroy
wildlife habitats, and impede farming. Their spread

eliminates grazing areas and accounts for one-third of all


crop loss. They compete for sunlight, nutrients, and
water with useful plants.
The global need for weed control had been answered
mainly by the chemical industry. Its herbicides are
effective and sometimes necessary, but some pose
serious problems, particularly if misused. Toxic
compounds threaten animal and public health when they
accumulate in food plants, groundwater, and drinking
water. They also harm workers who apply them.
In recent years, the chemical industry has introduced
several herbicides that are more ecologically sound.
Yet new chemicals alone cannot solve the worlds weed
problems. Hence, scientists are exploring the innate
weed-killing powers of living organisms, primarily
insects and microorganisms.
The biological agents now in use are environmentally
benign and are harmless to humans. They can be chosen
for their ability to attack selected targets and leave crops
and other plants untouched. In contrast, some of the most
effective chemicals kill virtually all the plants they come
in contact with, sparing only those that are naturally
resistant or have been genetically modified for resistance.
Furthermore, a number of biological agents can be
administered only once, after which no added
applications are needed. Chemicals typically must be
used several times per growing season.
78. What topic does this passage primarily deal with?
(A) The bad influences of unwanted plants on agriculture.
(B) The potential dangers of toxic compounds.
(C) Advantages of biological agents over chemical ones.
(D) A proposal to ban the application of all herbicides.
79. Which of the following statements about the use of
chemical agents as herbicides would the author most
likely agree?
(A) It should be completely banned.
(B) It becomes more dangerous.
(C) It is safe but inefficient.
(D) It is frequently required.
80. We can infer from the passage that __________.
(A) chemical herbicides only cause harm to unwanted
weeds
(B) new herbicides that are ecologically sound can solve
the weed problem
(C) biological agents should be applied several times per
season
(D) weeds take in sunlight, nutrients, and water that
useful crops need

5 5

97

41

42

43

44

45

46

47

48

49

50

51

52

53

54

55

56

57

58

59

60

61

62

63

64

65

66

67

68

69

70

71

72

73

74

75

76

77

78

79

80

97

10

11

12

13

14

15

10

10

10

Part A
1
Part B
1
Part C

97

85
1
2
1-70
71-85

(20%)
D 1. All the offices in the building were ______ with lights last night. Nobody knew what happened.
(A) absorbent
(B) abrupt
(C) abiding
(D) ablaze
B 2. Professor Brown is on ______ . You wont be able to take his course this semester.
(A) sabotage
(B) sabbatical
(C) savagery
(D) safari
A 3. Sam used to be a hot-tempered young man, but he is ______ now.
(A) mellow
(B) vicious
(C) irritable
(D) brutal
D 4. According to this book, plants and animals ____ gradually to adapt to their environments.
(A) devolve
(B) revolve
(C) involve
(D) evolve
C 5. The government needs to fix the farmers ____ system to ensure a great harvest.
(A) orientation
(B) purification
(C) irrigation
(D) migration
C 6. Drinking water is the best way to ___ your thirst.
(A)
quash
(B) quest
(C) quench
(D) query
C 7. Kevin always ______ about how unfairly his manager treats him.
(A) eulogizes
(B) extols
(C) mutters
(D) compliments
D 8. She doesnt like to go out with _____ men, because they usually consider themselves more
important than others.
(A) antagonistic
(B) ambitious
(C) aggressive
(D) arrogant
with phones and faxes every day.
C 9. A secretary usually lives a busy life and is likely to be
(A) equipped
(B) acquainted
(C) bombarded
(D) outfitted
B 10. Their boss offered a bonus as an _____ for them to work harder.
(A) obligation
(B) incentive
(C) essential
(D) imperative
D 11. It is a
in their culture for a bride to see the sun on the wedding day.
(A) treaty
(B) proverb
(C) will
(D) taboo
A 12 The manager prefers to ____ duties to give his staff wider experience.
(A) rotate
(B) ambush
(C) waylay
(D) intercept
A 13. Miss Wang always ______________ a sense of genuine interest in her students. No wonder she is
a popular teacher in school.
(A) conveys
(B) converts
(C) condemns
(D) condenses
D 14. The countdown was
, and the rocket ship went off on schedule.
(A) comical
(B) expectant
(C) indivisible
(D) flawless
1 8

C 15. To paint well, every young artist dreams that he can be ______ to the great Picasso for a few
years.
(A) dispensed
(B) bestowed
(C) apprenticed
(D) allocated
B 16. To prevent their enemies further attacks, they established a road block on the ____ between
the two countries.
(A) metropolis

(B) frontier

(C) burrow

(D) hub

D 17. It is surprising that ABC Bank _____ up NY Bank and became the biggest bank in the area.
(A) grabbed
(B) gasped
(C) grubbed
(D) gobbled
A 18. Never lend your money to someone who is _____ for borrowing and failing to return.
(A) notorious
(B) candid
(C) obliging
(D) keen
B 19. To prevent others from copying your invention, you should apply for a _____.
(A) bequest
(B) patent
(C) shell
(D) fleck
A 20. Keeping the fruit in the refrigerator can help you _____ their freshness.
(A) prolong
(B) abridge
(C) curtail
(D) envisage
(10%)
D 21. Will the visitors have an attitude? If yes, what should we do?
(A) look bored
(B) look surprised
(C) act in a friendly way
(D) act in an unfriendly way
A 22. That man has never been so bummed out in his life.
(A) depressed
(B) happy
D

(C) rich
(D) poor
23. The movie star took a lot of heat for going to the pub last night.
(A) carried a heater
(B) got hurt
(C) became popular
(D) was criticized
24. The task got on my nerves, so I couldnt but get it done.
(A) pleased me
(B) made me afraid
(C) relaxed me
(D) irritated me
25. They always stay at cross purposes. Thats why it is not easy to make them work together.
(A) without goals
(B) full of ideas
(C) interested in different things (D) restless
26. Her speech is going to knock you all off the pews.
(A) amaze you
(B) make you worry
(C) make you fall to the floor
(D) disappoint you
27. Mr. Brown loved to flaunt his intellectual prowess in front of his friends.
(A) carried a heater
(B)showed off
(C) became popular
(D) got hot
28. Typical nine-year-old girls are full of spunk.
(A) very sad
(B) very self-assertive
(C) very quiet
(D) very noisy
2 8

B 29. The old mans plan to build a castle by the beach is a pipe dream.
(A) a practical plan
(B) an impossible hope
(B) a realistic goal
(D) a new invention
C 30. That company is now owned by a shrewd tycoon.
(A) a poor person with a lot of business experience
(B) a rich person with no business experience
(C) a clever person who has become rich and successful in business
(D) a stupid person who has become rich and successful in business
(10%)
D 31. We could have enjoyed ourselves at the party if the weather ____________ fine.
(A) is
(B) was
(C) will be
(D) had been
B 32. ________ I you, I wouldnt make friends with a dishonest person like Jason.
(A) Was
(B) Were
(C) Had been
(D) Have been
C 33. That acne ___ by daily consumption of the new medicine gives patients a new hope.
(A) controlled
(B) will control
(C) has been controlled (D) had controlled
C 34. Everyone knows ____ in the frozen wastes on that island takes special equipment.
(A) it is survival
(B) survive
(C) to survive
(D) that survive
A 35. In my opinion, noise pollution receives less attention than ____ water pollution.
(A) does
(B) it does
(C) is
(D) it is
B 36. ____ is quite common in people who have low blood pressure.
(A) Faint
(B) Fainting
(C) The Faint
(D) Be Fainting
D 37. Crossing the valley, _____ temperatures reach above 100 degree Fahrenheit, is a challenging task
for the climbers.
(A) that
(B) whose
(C) which
(D) where
A 38. Today teachers are seen more and more in their role _____ facilitators.
(A) as
(B) like
(C) as if
(D) alike
B 39. Dr. Murphy, _____ from the south in a craft, attempted to prove his theory of relevance.
(A) setting sail
(B) set sail
(C) has set sail
(D) he set sail
D 40. More advances have been made in technology in the last decade ____ in all the rest of human
history.
(A) as well as
(B) and
(C) as
(D) than
(30%)
Qs 41~50
In the ninth inning against the Devil Rays, Yankees pitching coach Ron Guidry went to the 41
and told Wang Chien-ming that it was time for him to
42 what he could do. The Yankees won
because Wang always does his best
43
he is under pressure. He became one of the best young
pitchers in Major League Baseball 44
the age of 26. Wang decided to sign a contract with the
Yankees 45
Atlanta because he wanted to play with Roger Clemens, 46
was his idol. Wang is a
3 8

man of 47 . He never asks much from others and he has a sense of 48 . Thats why his
teammates like to
49 with him. Now he is a
40 to people in Taiwan. His most devoted fans
watch every game in which Wang is pitching.
B 41. (A) course
(B) field
(C) court
A 42. (A) prove
D 43. (A) because

(B) refuse
(B) since

C 44. (A) for


(B) on
B 45. (A) nothing but (B) instead of

(C) toss
(C) if
(C) at
(C) along with

(D) rink
(D) scout
(D) when
(D) in
(D) as well as

D 46. (A) what


A 47. (A) simplicity

(B) whom
(B) means

(C) that
(C) publicity

(D) who
(D) fun

B 48. (A) guilt

(B) humor

(C) direction

(D) security

D 49. (A) put up


C 50. (A) speaker

(B) find fault


(B) player

Qs 51~60
The musical Les Miserables,

51

(C) compare
(C) hero

(D) make friends


(D) pitcher

on Victor Hugos story about poverty and injustice in the 19th

century France, has had a positive 52 on people all over the world.
The musical Les Miserables has been a big hit all around the world. It has not only captured peoples
inspired them to overcome problems in their own lives. The story is 54
of
imaginations,
53
hope and optimism in the midst of poverty and injustice. It tells of how a criminal 55
Jean Valjean
is given a 56
chance to become an honest man. Valjean 57
become honest, and he makes many
sacrifices to help others. The musical version of the story 58
in France, and then was translated
and revised for the London stage. From there it moved to 22
59 countries. Although critics
originally hated it, the public loved it, and many people wrote letters to the director about
60 the
musical changed their lives.
B 51. (A) which based (B) based
(C) is based
(D) that is based
C 52. (A) affect
(B) attitude
(C) influence
(D) result
C 53. (A) while
(B) thus
(C) but
(D) which
A 54. (A) that
(B) which
(C) some
(D) one
C 55. (A) who called (B) called as
(C) named
(D) naming
B 56. (A) last
(B) second
(C) more
(D) next
A 57. (A) does
(B) may
(C) will
(D) should
B 58. (A) playing
(B) played
(C) performing
(D) performed
B 59. (A) the other
(B) other
(C) others
(D) another
D 60. (A) that
(B) if
(C) what
(D) how
Qs 61~70
Sound recording technology was first developed in the 1960s. Since then, illegal 61 of the
sounds of lawful recordings, or sound recording piracy, 62
life difficult for the music industry.
Owing to the poor quality 63
from the difficult task of pirating on ordinary tapes, it was quite 64
that piracy did not really hurt the music industry much.
4 8

It was not until the 1980s that the advance of digital technology 65 easy duplication and high
quality of recordings, increasing the rate of piracy. 66 , what has really made piracy intolerable and
out of control is the 67 of the Internet and the MP3 format. With ready access to 68 music files
over the Internet, a pirate can compress audio CDs into small files that can then be distributed, without
having to pay any additional costs 69 the phone bill.
inconceivable threat 70 the music industry.

Thats how piracy has been posing an

C 61. (A) purchase


D 62. (A) is making

(B) manufacture
(B) making

(C) reproduction
(C) made

(D) consumption
(D) has made

A 63. (A) resulting


B 64. (A) a dilemma

(B) to result
(B) a relief

(C) resulted
(C) a wound

(D) results
(D) an annoyance

A 65. (A) open the door to(B) got even with

(C) put an end to

(D) stood in the way of

D 66. (A) Moreover


C 67. (A) legitimacy

(B) Conversely
(B) authority

(C) Therefore
(C) availability

(D) However
(D) productivity

B 68. (A) download


C 69. (A) more than

(B) downloading
(B) less than

(C) be downloaded (D) being downloaded


(C) other than
(D) rather than

A 70. (A) to

(B) on

(C) for

(D) in

(30%)
Qs71~75
Hollywoods superstars think highly of their privacy, and the last thing they would like to see is for
their grand houses to become tourist spots. However, there are some exceptions: the homes of Michael
Jackson and Elvis Presley.
Neverland Ranch in California is the place Michael Jackson called home from 1988 to 2005. Since
it covers about 11 sq km, Jackson built a zoo and a theme park on his ranch. The island in the famous
childrens story Peter Pan gave him the idea for the name. When Jackson was living there, he often
invited seriously ill children to visit the ranch: here they didnt have to follow any rules. Its said that
these children could play and eat whatever they liked. Unfortunately, the ranch was closed by the
government because of some bad news about its owner.
Each year, about 640,000 people pay a visit to Graceland in Tennessee, the house where singer
Elvis Presley had lived. After Presley had been dead for five years, his ex-wife turned the house into a
museum. People have to pay about US$10 to see the entire mansion. The huge white building has 23
rooms, and there are 40 guides to show visitors around them. One of the rooms has three TV sets side by
side. Presley got the idea from a trip to the White house. In the back yard is the famous singers grave,
where people often put flowers and teddy bears. It is said that Elviss ghost still lives in the mansion.
Though we can no longer visit Michael Jacksons Neverland, we still can go to Graceland in order
to get some idea of how superstars live. Maybe this will make it easier for us to leave the stars alone, for
they surely need some privacy.

5 8

D 71. What are Hollywoods superstars least likely to do?


(A) Protect their privacy.
(B) Build grand mansions.
(C) Invite their friends to their homes.
(D) Let tourists come inside their houses.
B 72. How did Jackson name his ranch?
(A) Peter Pan told him the name.
(B) He got it from a story.
(C) He knew that he could never live there.
(D) The ranch is on an island having this name.
A 73. Why did Elvis put three TV sets side by side?
(A) He liked the way one of the White House rooms is designed.
(B) He could watch three channels at the same time.
(C) The room was big enough for three TVs.
(D) Visitors can choose the channel they want to watch.
D 74. Which is not true about Jacksons ranch?
(A) It is large enough to have a zoo and a theme park in it.
(B) Michael Jackson lived there from 1988 to 2005.
(C) Children who were invited there could do whatever they wanted.
(D) Jackson himself closed the ranch for some reason.
C 75. Which is true about Elviss mansion?
(A) Graceland was opened to the public fifteen years after Elvis died.
(B) Millions of people visit Graceland every year.
(C) Elvis is buried in the back yard of Graceland.
(D) People put flowers and teddy bears in Presleys room.
Qs76~80
The origins of most nursery rhymes can be found in folk tales or historical events. The rhymed lines
of these childrens poems are not only joyful to read; they also have some history behind them. Take the
well-known verse Three Blind Mice as an example.
Three blind mice, three blind mice,
See how they run, see how they run.
They all ran after the farmers wife,
Who cut off their tails with a carving knife.
Did you ever see such a thing in your life,
As three blind mice?
These lines are not just about three blind mice and a farmers wife. The poem is based on English
history. The farmers wife refers to the daughter of King Henry VIII, Queen Mary I, whose husband
6 8

was King Philip of Spain. Mary was a staunch Catholic and she cruelly killed many Protestants, for they
held a religious belief different from hers. Therefore she was nicknamed Bloody Mary. The three blind
mice were actually three noblemen who stuck to their Protestant faith and were accused of plotting
against the Queen. Queen Mary did not have their heads cut off, but she blinded them. And in the end, she
also had them burnt at the stake!
C 76. What is the main idea of the above passage?
(A) The way nursery rhymes were written.
(B) The story of English history.
(C) The origin of the poem Three Blind Mice
(D) How Queen Mary killed people.
A 77. What cant we infer from the passage?
(A) The farmer refers to King Henry.
(B) The mice refer to the noblemen.
(C) Queen Mary killed the noblemen.
(D) The noblemen were not blind at first.
D 78. What does the word staunch refer to?
(A) Being cruel to others.
(B) Being blind by nature.
(C) Having much money.
(D) Having a strong belief.
B 79. Why was Queen Mary called Bloody Mary?
(A) She blinded many Protestants.
(B) She put many Protestants to death.
(C) She killed many farmers.
(D) She cut off the tails of many mice.
D 80. Why were the three noblemen blinded?
(A) They killed King Philip.
(B) They were Catholics.
(C) They were burnt at the stake.
(D) They plotted against the queen.
Qs 81~85
When drawing human figures, children often make the head too large for the rest of the body. A
recent study offers some insights into this common disproportion in childrens illustrations. As part of the
study, researchers asked children between 4 and 7 years old to make several drawings of men. When they
drew front views of male figures, the size of the heads was markedly enlarged. However, when the
children drew rear views of men, the size of the heads was not so exaggerated. The researchers suggest
that children draw bigger heads when they know they must leave room for facial details. Therefore, the
old head size in childrens illustrations is a form of planning ahead and not an indication of a poor sense
of scale.
7 8

A 81.The main subject of the passage is ______.


(A) what the results of an experiment revealed
(B) how children learn to draw
(C) how researchers can gather data from works of art
(D) what can be done to correct a poor sense of scale
C 82. It can be inferred that, during the research project, the children drew ______.
(A) pictures of men from different angles
(B) figures without facial expressions
(C) sketches of both men and women
(D) only the front view of men
B 83. The word scale in the last line is close in meaning to ______.
(A) measurement
(B) proportion
(C) balance
(D) property
A 84. The findings of the experiment described in the passage would probably be of LEAST interest to
which of the following groups?
(A) teachers of art to children
(B) commercial artists
(C) experts in child development
(D) parents of young children
C 85. The passage provides information to support which of the following conclusions?
(A) Children under the age of 7 do not generally have a good sense of scale.
(B) With training, young children can be taught to avoid disproportion in their art.
(C) Children enlarge the size of the head because they sense it is the most important part of the body.
(D) Children plan ahead when they are drawing pictures.

8 8

97

2. Professor Brown is on ______ . You wont


be able to take his course this semester.
B
(A) sabotage
(B) sabbatical
(C) savagery
(D) safari

33

33. That acne ___ by daily consumption of


the new medicine gives patients a new hope.
(A) controlled
C
(B) will control
(C) has been controlled
(D) had controlled

39

39. Dr. Murphy, _____ from the south in a


craft, attempted to prove his theory of
B relevance.
(A) setting sail
(B) set sail
(C) has set sail
(D) he set sail

54

65

82

B
D

B
A

54. (A) that (B) which (C) some (D) one


A or D
54 of hope and optimism in D
A The story is
the midst of poverty and injustice.
65. (A) open the door to (B) got even with
(C) put an end to (D) stood in the way of
A It was not until the 1980s that the advance of
digital technology 65 easy duplication and
high quality of recordings, increasing the rate
of piracy.
82. It can be inferred that, during the
research project, the children drew ______.
(A) pictures of men from different angles
C
(B) figures without facial expressions
(A) sketches of both men and women
(D) only the front view of men

84

84. The findings of the experiment described


in the passage would probably be of
LEAST interest to which of the following
groups?
A
(A) teachers of art to children
(B) commercial artists
(C) experts in child development
(D) parents of young children

85

85. The passage provides information to


support which of the following conclusions?
(A) Children under the age of 7 do not
generally have a good sense of scale.
(B) With training, young children can be
taught to avoid disproportion in their art.
C
(C) Children enlarge the size of the head
because they sense it is the most
important part of the body.
(D) Children plan ahead when they are
drawing pictures.

I.

II.

Error-Picking and Correction: 20%


1. No matter you like it or not, staying at the top is impossible, for the view from the top tells us that the only way is
A
B
C
D
down.
2. Pictures are not just an aspect of method but they are an essential part of the overall experiences we must help our
A
B
C
students to cope.
D
3. Such is the versatility of the SV-3900 as it can be used equally successfully to record music or as a scientific research
A
B
C
D
tool.
4. Since language is used in context, it is followed that it should be learned in context, and preferably, in an authentic
A
B
C
D
context.
5. I figured they wouldnt mind that I sat in a corner for a couple of hours as long as I nibbled an onion bagel and nursed
A
B
C
D
a cup of tea.
6. As our day winding to a close, we realize not all travel is about seeing famous sites; many times, traveling is about the
A
B
C
people you chance to meet.
D
7. In front of us, wandered the wrong way through three lanes of stalled cars and trucks was a depressed elephant, its
A
B
trunk held high to avoid breathing exhaust fumes.
C
D
8. Many remember Vicki Van Meter as the brave sixth-grader beamed from the cockpit of her single-engine Cessna
A
B
C
172the kid who in 1993 became the youngest girl to fly across the Atlantic to Scotland.
D
9. In his writing, Robert Fulghum expresses his thoughts on the ordinary things that have happened to him in the course
A
of his varied life like father, neighbor, working cowboy, folksinger and minister, among others.
B
C
D
10. It is probably true that many activities appear to be communicative are little more than empty drills, in the sense that
A
B
C
D
no one really cares about the meaning.
Cloze: Please create a cloze test of five questions with four items for each based on the following article.
Underline the word(s) as the blanks you want. 20%
The previous growing season, this lush coastal field near Rome was filled with rows of delicate durum wheat,
used to make high-quality pasta. Today it overflows with rapeseed, a tall, gnarled weedlike plant with coarse yellow
flowers that has become a promising crop for European farmers: rapeseed can be turned into biofuel. Motivated by
generous subsidies to develop alternative energy sources, Europes farmers are beginning to grow crops that can be
turned into fuels meant to produce fewer emissions than gas or oil. They are chasing their counterparts in the Americas
1 / 2

who have been raising crops for biofuel for more than five years.
This is a much-needed boost to our economy, our farms, said Marcello Pini, 50, a farmer, standing in front of
rapeseed he planted for the first time. Of course, we hope it helps the environment, too. In March, the European
Commission approved a directive that included a binding target requiring member countries to use 10 percent biofuel
for transport by 2020the most ambitious and specific goal in the world.
III.

Reading Comprehension: Please create five reading comprehension questions with four items in each based on the
following article. 20%
The first thing you notice about Six Senses Hideaway Yao Noi, is the overgrown vegetation that threatens to
obscure the path to your accommodation. In fact, you could almost believe that the 54 butler-serviced pool villas, lying
amid thick and unkempt greenery, were somehow left over from the former rubber plantation that the resort is built
onand thats what the designers want you to think, of course. In reality, this is a studied, artfully landscaped wilderness,
created to let you feel as though you really are in the middle of a Thai jungle. Why else would you be forking out $800 a
night?
Yao Noi island is 45-minute speedboat ride from the tourist haven of Phuket. Theres little vehicular traffic, a
single 7-Eleven is the only place to shop and villagers still eye visitors curiously. All of this heightens the enjoyable
isolation at Six Senses. Its not every day you get to meditate in a mangrove swamp, or strike a yoga pose while watching
the sunrise from limestone cliffs. If you must have the trappings of civilization, you can sample 60 varieties of homemade
ice cream and sorbets at Living Room, an all-day restaurant overlooking PhangNga Bays famous Hong Islands; drink to
the same view at the bar (which opens at a liberal 10:30 a.m.); or book a massage at the spa. Nightlife is nonexistent on
the island, so youre holed up at the resort after dark. This translates to raiding your villas wine fridge (it comes stocked
with 23 bottles) and skinny-dipping in the privacy of your pool. What more could you ask for?

IV.

Writing: After reading the following poem, please write an article no more than 300 words to describe the
relationship between you and either of your parents and how it has influenced you. 40%

Sundays too my father got up early


and put his clothes on in the blueblack cold,
then with cracked hands that ached
from labor in the weekday weather made
banked fires blaze. No one ever thanked him.
Id wake and hear the cold, splintering, breaking.
When the rooms were warm, hed call,
and slowly I would rise and dress,
fearing the chronic angers of that house,
Speaking indifferently to him,
who has driven out the cold
and polished my good shoes as well.
What did I know, what did I know
of loves austere and lonely offices?

2 / 2

I.

II.

Error-Picking and Correction: 20%


1

(A) whether

(D)cope with

(B)that

(B)follows

(B)if

(A)winds

(A)wandering

(B)beaming

(C)as

10

(B)appearing or which appear

Cloze: Please create a cloze test of five questions with four items for each based on the following article.
Underline the word(s) as the blanks you want. 20%

III.

Reading Comprehension: Please create five reading comprehension questions with four items in each based on the
following article. 20%

IV.

Writing: After reading the following poem, please write an article no more than 300 words to describe the
relationship between you and either of your parents and how it has influenced you. 40%

1 / 1

96

I. Vocabulary, idioms and phrases: 16% (1% for each)


1.

On the ______ of restructuring family life, we cling ever more ardently to this antiquated and
ill-conceived provider-homemaker design.
(A) orgy
(B) bust
(C) knot
(D) cusp
2. Many ______ forms of information can be linked together in the database.
(A) decadent
(B) dissident
(C) disparate
(D) desperate
3. A light rum and tonic in hand, he abandoned his post for the kitchen to ______ food.
(A) mingle with
(B) forage for
(C) refer to
(D) patch up
4. The star was ______ at the airport by a crowd of photographers and reporters, which scared her to
death.
(A) mobbed
(B) hailed
(C) feted
(D) spurred
5. A ______ of greeting emails flooded my mailbox as Christmas approached.
(A) blizzard
(B) creed
(C) lapse
(D) plague
6. That so much attention has been paid to the sexual scandal of the official renders it ______ absurd .
(A) scrupulously (B) monumentally (C) inferentially
(D) inextricably
7. Everyone crowded round, ______ on him and readily taking up his offer of free drinks.
(A) taunting
(B) beaming
(C) fawning
(D) frowning
8. Kevin liked to walk around the house in nothing but a pair of jeans, showing off his muscular ______.
(A) venue
(B) statue
(C) niche
(D) torso
9. The suggestion may sound ______ but the simple fact is it works.
(A) corny
(B) genial
(C) candid
(D) vigorous
10. Before they took action, the bank robbers were shunted into a tunnel and ______ by club-wielding
policemen.
(A) ambushed
(B) scrounged
(C) assuaged
(D) imposed
11. The movie's special effects are absolutely ______ , which hardly add anything positive to the movie.
(A) armored
(B) pathetic
(C) frantic
(D) impish
12. The young man put his own life ______ to rescue the drowning girl.
(A) on probation (B) in peril
(C) at bay
(D) on alert
13. They chose Alex for the last leg of the relay race because he was good at track events, especially at
______.
(A) satire
(B) caveat
(C) sprint
(D) diameter
14. The two companies finally came to a consensus that was ______ agreed on.
(A) rightfully
(B) exclusively
(C) reciprocally
(D) outrageously
15. Since Rita keeps early hours and exercises regularly, she is ______.
(A) in good hands (B) off the hook
(C) in the pink
(D) on the alert
16. Tom is real a ______. He said he was in love with me, but in fact he was seeing other girls.
1

(A) dirty dog

(B) black sheep

(C) small potato

(D) good egg

II. Headings: Please choose the proper heading for each paragraph of the following
article. 14% (2% for each)
(A) A New Culture

(G) Communities Are Established

(B) Crops Flourish

(H) Natives Resist Icelanders

(C) Early Stories May Be True

(I ) Trade with Scandinavia

(D) Greater Dependence on the Sea

(J ) Invaders from the West

(E) Life on the Land

(K) Difficult Times

(F) A Pioneer Attracts Followers

(L) Ties with North America Strengthen


The First Greenlanders

C
About a millennium ago, legends tell us, a Viking named Leif Eriksson sailed to the shores of North
America, arriving hundreds of years ahead of Christopher Columbus. Even though archaeologists have yet
to uncover any physical evidence of Eriksson's visit, the presumption that a Viking band traveled that far
has gained credibility in recent years. Excavations in Greenland indicate that Vikings flourished there for
hundreds of years, trading with the European continent and probably Native American tribes, before
disappearing.
1. ______
A central figure in this story was Eriksson's father, Erik the Red, who grew up in Iceland. In 980 A.D.,
Erik the Red headed farther west when he was banished from Iceland--for murder. He set sail for land that
was visible west of Iceland. Three years later, he returned to Iceland and convinced hundreds of others to
join him in settling this new country. Some 25 boats set out for what Erik the Red had dubbed Greenland.
Only 14 ships survived the seas, but about 450 new colonists set foot ashore.
2. ______
The land they saw before them was bare, uninhabited, and inhospitable, but Erik the Red's
advertisements were not entirely false. A thin green carpet of arctic heath promised support for grazing
farm animals. Farms sprang up quickly and, later, churches. One colony, simply called the Eastern
Settlement, sat in the toe of Greenland; the Western Settlement lay close to what is now Nuuk, Greenland's
capital.
Settlement a Challenge
Settling Greenland posed a formidable challenge. There were no trees large enough to produce timber
for shelter or fuel. The only wood was small brush and driftwood. The Vikings settled inland, on fjords
resembling those of their homeland. There they built homes of driftwood, stone, and sod. For adequate
insulation, the walls of some buildings were made six to 10 feet thick.
3. ______
Shelter, food, and clothing were, of course, essential to survival. The summer was too short to farm
grain crops, so settlers probably went without beer or bread. Although they farmed domesticated animals
imported from Europe--goats, sheep, cattle--the settlers ate them sparingly, relying instead on secondary
products, such as milk and cheese. In the early days, the Greenlanders' lives differed little from those of
their compatriots in Scandinavia. They netted fish and hunted seal and caribou. They wove clothing from
wool and linen, sometimes adding the fur of the arctic hare.
4. _______
2

For about two centuries, Greenland's Vikings had the country to themselves. Yet life was by no means
easy, and they relied on a fragile trade with Scandinavia to survive. In exchange for iron, timber, and grain
from Europe, they traded pelts of bear and arctic fox as well as narwhale tusks and rope made of walrus
hide. Whalebone, too, was traded to Europeans for use in stiffening clothes. According to one account, the
Greenlanders even traded live polar bears.
5. _______
At some point during the fourteenth century, Greenland's climate grew colder. With the climate
change, glaciers began creeping over the land, bringing with them a runoff of sand, silt, and gravel. That
runoff slowly robbed the settlers of valuable pastureland. To make matters worse, the Black Death hit
Iceland, killing some 30 percent. Although there is still no evidence the sickness reached Greenland,
archaeologists believe it left its mark by curtailing the flourishing trade.
6. _______
The Greenlanders adapted. Recent evidence shows that their diet shifted from land-based foods to
marine products. Like their kin in Norway, the Vikings in Greenland had always exploited marine life but,
by the close of the fourteenth century, the proportion of their food taken from the sea had risen to 80
percent.
7. _______
Between 1100 and 1200 A.D., as the colder weather arrived, so did the Thules. These Native
Americans, migrants from the area surrounding the Bering Strait began trickling eastward from Ellesmere
Island, just northwest of Greenland. It's likely that an uneasy trade between the Vikings and Thules sprang
up and that, as living conditions grew harsher for the Vikings, the better-adapted Thules thrived.

III. Blank-filling: Fill in the blanks with the most appropriate words and phrases.
10% (1% for each)
Two years ago, Tim Lenczowski dreaded walking from the parking lot into his office. Weighing 335
pounds, Lenczowski suffered constantly from pain in his knees and ankles. Everyday activities such as
walking and even traveling on an airplane had become difficult.
At the age of 39, he was 1. d
with a heart condition and hypertension. He knew it was time to
make a change and his doctor agreed. The extra weight not only took a toll on his physical health, but also
was chipping 2. ________ at his self-esteem. Lenczowski also spoke of his experience of feeling like a
3. h
when he worked as a fundraiser for a nonprofit health organization. "People would see me
then ... they didn't respect me," remembers Lenczowski. "How could I ask for money to support [the
foundation] without practicing what I 4. p
?"
Lenczowski struggled with weight for most of his life. He tried just about "every diet 5. i
" and
though he lost weight on some, he would always gain it back. A sedentary lifestyle and fast-food diet had
caught up with him and as his 40th birthday approached, he'd become fed up and realized he didn't want to
6. l
________ the next half of his life as a fat person.
Not knowing where to start, Lenczowski started walking because it was low impact. He walked a
marathon and though it took him nine hours to complete, Lenczowski says he made some great friends in
the process. After the race, the same friends asked him to try kickboxing. "The thought of going to a gym
was intimidating enough, but kickboxing?" Lenczowski recalls. "My friends kept on me 7. ______ I caved
and reluctantly decided to try it."
3

The first time he went to the class, Lenczowski says he sat in the parking lot for a while trying to
8. m
________ enough courage to walk into the gym. But once inside, he says everyone welcomed
him. In eight months of kickboxing, fat burning classes and watching his diet he'd lost 60 pounds.
Lenczowski says he was ecstatic and his friends began pushing him to take the next step to get in shape -boot camp. "It's the hardest thing I've done," Lenczowski says. With the support of his friends and by
pushing and challenging himself each month to move from the back of the class to the front, he started to
see results. Eight months later he had lost a total of 120 pounds. To date, Lenczowski has run five full
marathons and 12 half-marathons, accomplishments he would have thought impossible five years ago.
Achieving various fitness goals was a tremendous 9. b
to Lenczowski's self-confidence. The
physical accomplishments began to have a positive affect on other aspects of his life as he realized he
could do anything he set his mind to. Since losing weight, he's become more outgoing, happier, moved
closer to a park, changed jobs and become a fitness instructor. He recently bought his own boot camp
franchise.
The dramatic weight loss and healthier lifestyle has also improved Lenczowski's physical health. He's no
longer on heart medication and his doctor has greatly reduced his blood pressure medicines.
Lenczowski says the biggest key to his success was his 10. n
of support from friends, doctors
and fitness instructors. As he approaches his goal weight, he's set a new goal of passing his experience on
to others who want to lose weight.

I. Vocabulary, Idioms and Phrases: 16% (1% for each)


1~5 DCBAA
6~10 BCDAA
11~15 BBCCC
16 A
II. Headings: 14% (2% for each)
1~5 FGEIK 6-7 DJ
III. Blank-filling: 10% (1% for each)
1. diagnosed 2. away

3. hypocrite

6. live out

8. muster up 9. boost

7. until

4. preached

5. imaginable
10. network

1 / 2

97

You have 90 minutes to finish the following questions. Good luck!


I.

Choose the Correct Answer (30%)


C 1.The maple syrup is obtained from the sap that ________ from the trees in early spring.
(A) effaces (B) enjoins (C) exudes (D) emblazons
A 2.Janes account of the event was a lengthy ________, unrelieved by any light moments.
(A)jeremiad (B) caste (C) aureole (D) libido
C 3.Tommy felt that it was beneath his dignity to ________ his hands in such menial labor.
(A)toady (B) haggle (C) sully (D) impugn
B 4.Jasons ________ temperament was apparent to all who heard him rant about his difficulties.
(A)mendacious (B) bilious (C) libelous (D) callous
D 5.Clovers parents thought that she was ________ safely in the private school in the US and decided to leave for
Taiwan.
(A)consecrated (B) apprised (C) genuflected (D) ensconced
D 6.We frequently judge people by the company with whom they ________.
(A)congeal (B) abrade (C) abdicate (D) consort
A 7.Although this plastic has many ________ qualities such as transparency, it is unbreakable.
(A)vitreous (B) sartorial (C) lissome (D) dulcet
C 8.Henry is ________ to any suggestions which comes from those he looks up to.
(A)disconsolate (B) devious (C) amenable (D)astral
B 9.Her work had fallen into a state of ________; no one bothered to read them.
(A)apogee (B) oblivion (C) nadir (D) rime
C 10.He could never be a stuffy ________; his classes were always lively and filled with humor.
(A)compendium (B) imbecility (C) pedagogue (D) abnegation
A 11.Amanda decided to ________ and accept a position to work in an overseas institution.
(A)take the plunge (B) make a dent (C) push the envelope (D) beat the rap
C 12.The CEO ________ the rumors of the companys impending sale.

(A)ratcheted up (B) took a hit (C) tamped down (D) weighed in on


13.Come to class early tomorrow because the teacher has something special ________ for us.
(A) off on a tangent (B) left on the sidelines
(C) on the lookout
(D) in store for
B 14.The cookie factory ________ millions of cookies every week.
(A) plays out (B) churns out (C) mires in (D) bathes in
B 15.Patrick is a gifted tennis player and he plays tennis ________ some professionals.
(A) in the wake of (B) on par with
(C) by virtue of
(D) in full swing as
II.
Choose the wrong sentence in each question and correct them (10 %)
D 1.
(A) He is keeping quiet lest he disturb his father.
(B) Immediately the room went dark, the children began to cry.
(C) That he married Louise was a bolt from the blue.
(D) This grand prize goes to you think whomever can solve this thorny problem.
A 2.
(A) It is essential that my position is clearly understood.
(B) Thomas is no less intelligent a man than Robert.
(C) I want this wrapped up in cloth.
(D) Many a little make s a mickle.

2 / 2

C 3.
(A) For a student to succeed takes hard work.
(B) Their performance leaves nothing to be desired.
(C) We had done nothing but sent for a doctor.
(D) The student demonstrators returned to the campus , assured of the political reform at the earliest possible
time.
C 4.
(A) We felt our boat driven by a powerful current.
(B) Nowhere was the missing key to be found.
(C) More than one witness were able to identify the suspect.
(D) Seismologists are unable to predict the time another earthquake will occur.
B 5.
(A) If he were to become young again, he would compete in decathlon.
(B) But for the rainstorm yesterday, they could have a pleasant weekend.
(C) I shall be only too pleased to accept your invitation.
(D) However much you might give him, he was never satisfied.
III.
1. Fill in the blanks with the words provided. Change the forms of the words if necessary. (20 %)
(A) suppress (B) bind (C) cognitive (D) embroider (E) inhibit
(F) rule (G) admire (H) profession (I) imprint (J) attest
During the Tang Dynasty, girls as young as five had bandages wrapped around their feet to ____1____
the growth to no longer than six inches. This barbaric process became popular all over China because people
believed that the smaller the feet, the more beautiful the girl. Later in their lives, women who had endured
foot binding suffered from terrible infections or were unable to walk even short distances. Women with
____2____ feet wore lotus shoes. These delicate shoes were ____3____ by the woman and ____4____ to the
smallness of her feet. Many men became fascinated with these small-footed women. As a result, small feet
and lotus shoes became a fetish for some men.
Fetishes have been around forever. However, it wasnt until the late 1800s that it was identified and
named. In 1877, French psychologist Alfred Binet introduced the word fetish to the world meaning
____5____ of an inanimate object. Since then, people who have serious fetishes for strange things have been
thought of as mentally ill. Psychologists have varying theories about why some people develop fetishes, but
most believe that
6 , an event during childhood that has stayed in the mind, is to blame.
To treat people with severe fetishes, doctors use
7 therapy and psychoanalysis. This former
focuses on
8
the patients present-day desires, while the latter tries to figure out what caused the
problem during childhood. Prescription drugs that limit the amount of testosterone in males and estrogen in
females also help. These medications enable patients to ignore their fetish and regain control of their lives.
Fetishes are normal as long as they dont cross the line. When it ____9____ a persons life and puts
other in danger,
10 help is the answer.
2. Please change the above passage into a cloze test with 5 questions. (10%)
IV. Please briefly explain the following teaching methodology (15%)
i. Audio-lingual method
ii. Natural approach
iii. Communicative language teaching
V.

Use at least one of the above teaching methodology to create a 50-minute


teaching activity based on the issue of global warming. (15%)


I. Choice: Choose one word or phrase which would best keep the meaning of the
original sentence if it were substituted for the underlined word. 20%
( B )1. Unorganized guessing will probably not raise your score as significantly as
choosing one letter as a guess answer for the entire examination.
(A)Cryptic
(B) Haphazard
(C)Leisure
(D)Subsequent
( A )2.The law officer in many early Western settlements had to maintain order by
means of his gun.
(A) marshal
(B) mayor
(C)major
(D)military
( B )3.Electrical energy may be separated into two components specified as
positive and negative.
(A)germinated
(B)designated
(D)contaminated

(C)accumulated

( B )4.Professor Baker explained in detail several theories of evolution which had


historically preceded that of Charles Darwin.
(A)found fault with
(B)expounded upon
(C)presided over

(D)took after

( C )5.The thief was apprehended, but the person who aided him escaped.
(A)arrogance

(B)avarice

(C)accomplice

(D)asset

( D )6.A clever politician, he took advantage of every speaking engagement to


campaign for the next election.
(A)rash
(D)shrewd

(B)intrepid

(C)crude

( C )7.It will be necessary for the doctor to widen the pupils of your eyes with some
drops in order to examine them.
(A)brandish
(D)soothe

(B)flatter

(C)dilate

( A )8.Since we left our umbrellas at home, we got very wet in the sudden
downpour.
(A) drenched
(B) parched
(C)audible
(D)autonomous
( D )9.A cut in the budget put 10 percent of the employees jobs in danger.
(A)clemency
(B)decency
(C)eulogy
(D)jeopardy
( D )10.It was necessary to divide the movie into three parts in order to shoo it on
television.
(A)abate
(B)assert
(C)transact
(D)segment


Part One: Vocabulary
1. Students were bored by the lecture, but as soon as the teacher showed the
movie, they all __________.
(A) retrieved (B) reclined (C) resolved (D) revived
2. Lily won the first place in the speech contest and her confidence in speaking English was
__________.
(A) blustered (B) bolstered (C) bantered (D) bartered
3. If you do not pay your insurance bills, your ____________ will be denied.
(A) coverage (B) coven
(C) covering (D) covertures
4. In the summer time, ____________ of mosquitoes come out after sundown.
(A) myriads (B) amounts (C) quantities (D) volumes
5. Last year, East Africa was ____________with swarms of locusts that destroyed the
crops.
(A) pleated (B) plinked (C) plumed (D) plagued
6. Human beings are ____________ because we eat both plants and animals.
(A) omnivorous (B) capricious (C) unpredictable (D) inconsistent
7. My mom never makes large purchases without checking for the best price. This
was an example of her ____________.
(A) judgment (B) conviction (C) prudence (D) recklessness
8. The maid did not intentionally drop the vase. It was ____________.
(A) deliberate ( B) inadvertent (C) voluntary (D) premeditating
9. Sean behaved in a(n) __________ manner even though he was quite shy.
(A) gregarious (B) aloof (C) conceited (D) bashful
10. The comedian tried all he could to make the ____________ audience laugh, but they
remained quiet.
(A) conversable (B) expansive (C) restless (D) taciturn
11. The surgeon made an ____________ into the patients breast and the removed the
tumor.
(A) incursion (B) indention (C) incision (D) infliction
12. When we first learn about the customs of other culture, we may have a(n)
____________ to them because we are not used to them.
(A) aversion (B) diversion (C) inversion (D) reversion
13. Lillians psychiatric condition was __________ by the stress of her work.
(A) seduced (B) deduced (C) trickled (D) triggered
14. I do not like my ___________ life style. I want to get more exercise.
(A) routine (B) frazzling (C) sedentary (D) interactive
45

15. Judys parents sent her to Vienna, Austria. They think the environment there can
__________ her music talents.
(A) culture (B) nurture (C) nudge (D) nuzzle
16. The old woman was too weak to walk steadily. She ___________ all the way to the
market.
(A) stumbled (B) strolled (C) startled (D) stigmatized
17. Jim learned many things from his colleague Vincent when he got his first job in IBM.
Vincent was Jims __________.
(A) minister (B) master (C) monitor (D) mentor
18. His laughter was __________. After people heard him laugh, they all began to laugh.
(A) innocuous (B) incongruous (C) invincible (D) infectious
19. After the operation, the doctor told the patients family that the following 24 hours were
a(n) __________ period. Whether the patient could survive or not depended on his
condition during that period.
(A) crucial (B) inciting (C) momentous (D) substantial
20. I felt __________ when people told me that the party was a great success.
(A) flocked (B) fleeted (C) flirted (D) flattered
21. My comments were totally ___________ by my colleagues. They had no idea of what I
was talking about.
(A) miscounted (B)misconstrued (C) misreckoned (D) miscued
22. Every body in the department liked Lisa very much so they elected her the chair of the
department ___________.
(A) unwittingly (B) unanimously (C) unyieldingly (D) unrestrainedly
Part Two: Cloze
(A) Are some people generally better than others at coping with stress? Recent research
there is a certain kind of person who has a
suggests that the answer is yes
23
relatively stress- resistant personality. Suzanne Kobasa (1982) has found that people who
cope well with stress tend to
24
to what they are doing (rather than alienated), to
feel in control (rather than powerless), and to welcome
25
amounts of change and
challenge. In studies of people
26
stressful situations, Kobasa and her associates
have found that those with stress-resistant personalities 27 , those who are high in
commitment, control, and challenge experience fewer physical illnesses than those whose
personalities are less hardy.
23. (A) that
(B) when
(C) because
(D) often
24. (A) be committing (B) be committed (C) commit
(D) committing
25. (A) manipulating
(B) maximal
(C) minimal
(D) moderate
26. (A) face
(B)faced
(C) facing
(D) being facing
27. (A) in fact
(B) for example
(C) in general
(D) that is
46

(B) The farm wifes sense of duty to her family and the land was forged by wars. It was
chiseled and
28
easterners and immigrants broke the sod. It was battered by the
Great Depression. Families who survived
29
searched for meaning in their lives
and financial security.
It may have originated with Martha Washington,
30 commitment to her husband
and the 4,000-acre farm at Mount Vernon was as strong as her patriotism to the cause of the
Revolutionary War. Historians describe Martha as
31
with her husband in the days
of management: George rose at 4 a.m., and filled diaries with accounts of his farming; she
gave household orders before breakfast,
32
time for gardening, the care of her
children and training of slaves and servants. Symptomatic of the central tragedy of this
nation, Marthas dowry included 150 slaves.

28. (A) divided by


29. (A) what
30. (A) her
31. (A) an equal
32. (A) secure

(B) defined as
(B) still
(B) what
(B) a rival
(B) securing

(C) described as (D) delimited by


(C) this
(D) that
(C) which
(D) whose
(C) a model
(D) a support
(C) secured
(D) being securing

Part Three: Reading Comprehension


(A) It is been said that the famous pinstriped uniform of the New York Yankees baseball
team adds a certain something to a ballplayers game, an aura of intimidation that will help
him succeed. Others may dispute that idea, but it raised an interesting question: Can an
athletes uniform affect success? A study of the mens judo competition at the 2004
Olympics in Athens suggested yes.
In high-level judo matches, to differentiate between the contestants, one competitor
wears a white uniform, called judogi, while the other wears blue. The study found that
those in blue won more matches than those in white and suggested, among other reasons,
that the brighter blue had an intimidating effect, much as with some animals, red coloration
is thought to be intimidating.
But a new study by researchers in the University of Glasgow and the University of
Amsterdam disputes those findings. Reanalyzing the match data, they reported that other
factors came into play. The analysis showed that higher-ranked contestants wore blue more
often and that higher-ranked contestants were more likely to win. Athletes in blue often
ended up with more recovery time between matches because of the tournament format.
When all factors are considered, the researchers say, there was no bias toward blue.

47

33. What does the first study about the judo competition suggest?
(A) The team that wears red is more likely to win.
(B) The team that wears blue has high morale.
(C) Uniforms contribute to the winning of the game.
(D) Uniforms play a dominant role in winning a game.
34. What is true about the new study?
(A) It collected new data to testify the hypothesis.
(B) It was focused on the winning possibilities of all sports.
(C) It suggests that uniform plays a dominant role in winning the game.
(D) It suggests that the game format is to the advantage of the more skilled athletes.
35. What can be inferred from the passage?
(A) Those who are more skilled can choose what to wear.
(B) Those who are more skilled can choose to wear blue.
(C) The wining of a game is a result of more complex factors.
(D) The Yankees intend to intimidate other teams by wearing the pinstriped uniform.
36. What does the word aura in line 2 mean?
(A) characteristic
(B) atmosphere
(C) light
(D) sound
(B) In part, gender roles are shaped by the power of reward and punishment the key
processes in operant conditioning. Parents, teachers, peers and others often reinforce,
usually with tacit approval, gender appropriate behavior and respond negatively to
gender inappropriate behavior. If you are a man, you might recall getting hurt as a young
boy and being told that men dont cry. If you succeeded in inhibiting your crying, you
might have earned an approving smile or even something tangible like an ice cream cone.
The reinforcement probably strengthened your tendency to act like a man and suppress
emotional displays. If you are a woman, chances are your crying wasnt discouraged as
gender-inappropriate.
Studies suggest that parents may use punishment more than reward in socializing
gender roles. Many parents take gender-appropriate behavior for granted and dont go out
of their way to reward it. But they may react negatively to gender-inappropriate behavior.
Thus, a ten-year-old boy who enjoys playing with dollhouses may elicit strong disapproval
from his parents.
48

37. What is the passage about?


(A) Gender roles as stereotypes
(B) Gender appropriate behavior
(C) Gender inappropriate behavior
(D) Factor that shapes the gender role

38. Based on the passage, what is true about gender roles?


(A) Parents more often than not explicitly teach children to follow the norm.
(B) Young boys are often encouraged to learn to control their emotional display.
(C) Gender roles are solely decided by the concepts of people around children.
(D) Those children who get ice cream and candies are more likely to conform to the
norm.
39. What can be inferred from the passage?
(A) Reward is the primary way because of its motivating power.
(B) Elders prefer to use punishment because it is more effective.
(C) A young boy will likely get punished if he enjoys playing with a Barbie.
(D) A young girl will be silenced by her parents when she cries because of getting hurt.
40. What does the word tangible in line 6 mean?
(A) touchable
(B) attractive
(C) minimal

(D) seductive

49

97

10

11

12

13

14

15

16

17

18

19

20

21

22

23

24

25

26

27

28

29

30

31

32

33

34

35

36

37

38

39

40

50

97

I. READING (1-14=14%, 15-38=48% )


PASSAGE 1 (See the end of the test paper)
(Questions 1-6)
Do the following statements agree with the information given in the passage on myopia?
GIVEN on your answer sheet.
1.
2.
3.
4.
5.
6.

Write TRUE, FALSE, NOT

Irrefutable proof has been given for the causes and development of myopia.
Nearsightedness occurs when objects in the distance become out of focus.
Too much eye growth after birth results in long eyes.
Nearsighted people are not more likely to develop retinal problems.
If you have myopic brothers or sisters, it increases the chances of you developing myopia.
Myopia may be caused by a combination of genetic and environmental factors.

(Questions 7-14)
Complete the following summary with words taken from Reading Passage 1.
Write NO MORE THAN TWO WORDS OR A NUMBER for each answer.

Some researchers have suggested that myopia is 7 Another factor in support of


the genetic case is that the 8 has adverse effects on the development of the eye.
On the other side of the argument, 9 can also lead to myopia. Night lights are
the other factor. Tests were conducted for these on three different 10.
However, some evidence has been put forth questioning the 11 of the
environmental case. Questions include: do myopic parents tend to turn on night
lights? How about research into different ethnic groups? Why does it take so long
for the damage to manifest itself and how12are parents memories? These are
some alternatives in the treatment of .13 However, the favored option does
have some 14.

PASSAGE 2 (See the end of the test paper)

(Questions 15-21)

The passage on the bee behavior has seven sections A-G.


Choose the correct heading for each section from the list of headings below.
Write the correct number i-x in boxes 15-21 on your answer sheet.
List of Headings
Bee behavior is a mystery
i
ii Communicating direction when outside a hive
iii How bees carry food on their bodies
iv Von Frisch discovers that bees communicate
How bees communicate direction when inside a hive
v
vi The special position of bee language
vii Expressing distance by means of dance
viii The purpose of the two simple dances
ix The discovery that bees have a special scent
Von Frisch discovers three types of dance
x

97
15.
16.
17.
18.
19.
20.
21.

Section A: __________
Section B: __________
Section C: __________
Section D: __________
Section E: __________
Section F: __________
Section G: __________

(Questions 22-24)
The writer mentions THREE kinds of bee dance identified by von Frisch.
List the name the writer gives to each dance. Use ONE WORD ONLY for each answer.
22. ___________________
23. ___________________
24. ___________________
(Questions 25-27)
Complete the sentences below with words taken from the passage.
Write NO MORE THAN THREE WORDS for each answer.
25. Von Frisch discovered the difference between dance types by changing the position of ______________
26. The dance outside the hive points in the direction of _______
27. The angle of the dance from the vertical shows the angle of the food from _________
PASSAGE 3 (See the end of the test paper) Questions 28-31
28. Which THREE possible reasons for American dominance of the film industry are given in the text?
A
B
C
D
E
F

plenty of capital to purchase what it didnt have


making films dealing with serious issues
being first to produce a feature film
well-written narratives
the effect of the First World War
excellent special effects

(Questions 29-31)
Answer the questions below using NO MORE THAN THREE WORDS from the passage for each answer.
29. Which TWO types of film were not generally made in major studios?
30. Which type of film did America develop in both short and feature films?
31. Which type of film started to become profitable in the silent period?
(Questions 32-38)
Match each statement with the correct country below. Write A-J in boxes 32-38.
You may use any letter more than once.
32. It helped other countries develop their own film industry.
33. It was the biggest producer of films.
34. It was the first to develop the feature film.
35. It was responsible for creating stars.
36. It made the most money from avant-garde films.
37. It made movies based more on its own culture than outside influences.
38. It had a great influence on silent movies, despite its size.

97
List of Countries
A

France

F Japan

Germany

G Soviet Union

USA

H Italy

Denmark

Britain

Sweden

China

I. READING
Passage 1 ()

Passage 2 ()

FALSE

NOT GIVEN

15

vi

TRUE

16

iv

FALSE

17

TRUE

18

viii

NOT GIVEN

19

vii

hereditary // inherited

20

ii

nervous system

21

close work // near work

22

10

subjects

23

11

validity

24

12

accurate

25

the feeding dish

13

myopia // nearsightedness

26

the food (source)

14

side effects // dangers

27

the sun

round, sickle, waggle

Passage 3 ()
28

ADF

31

avant-garde (films)

34

37

29

cartoons, serials

32

35

38

30 (slapstick)comedy//slapstick 33

36

97

45 2 100

60 %
15 %
(

to lipstick and clothing.


) 1. Petroleum is also used for producing goods ranging from plastics and laundry
(A) determent
(B) detention
(C) detergent
(D) determinant
.
) 2. As the demand for food remains high and less food is being supplied, food prices around the world are
(A) soaking
(B) soaring
(C) sobering
(D) sobbing
) 3. With its modern design
bright and roomy stations, the new MRT system will carry Kaohsiung into a brave new
era of travel.
(A) fetching
(B) feigning
(C) fermenting
(D) featuring
) 4. In Tibet, the first incident occurred this March when around 400 monks left their
and marched to protest Chinas
restrictions on their religion.
(A) monody
(B) monocracy
(C) monarchy
(D) monastery
) 5. At Newcastle University in England, scientists have created
by implanting human DNA into cow eggs.
(A) embryos
(B) emeralds
(C) emigrants
(D) emperors
) 6. The government has taken action to combat online crime such as
advertising and credit card fraud.
(A) perceptive
(B) conceptive
(C) deceptive
(D) interceptive
the verse from the first line to the last without stop.
) 7. After the encouragement of the teacher, the student began to
(A) rave about
(B) turn over
(C) reel off
(D) make up
) 8. Bowling used to be a popular sport in Taiwan, but it hasnt really continued to
in the past few years.
(A) hold up
(B) set on
(C) put up
(D) catch on
) 9. From this spot, you can have a breathtaking
of the whole canyon.
(A) congestion
(B) sublime
(C) inquisition
(D) panorama
) 10. Because of the
of the instructions, she had no idea what to do next.
(A) erection
(B) perplexity
(C) ailment
(D) contemplated
) 11.
resistant is a simple idea: Bacteria that might be expected to be wiped out by a drug are instead unaffected by it.
(A) Genetic
(B) Pathogenic
(C) Antibiotic
(D) Subterranean
) 12. The new computer game Wii provides us with an
way of exercising. People now may play sports in their living
rooms, which was unimaginable before.
(A) innovative
(B) urgent
(C) aggressive
(D) assessorial
of those who flout them too brazenly.
) 13. Saudi Arabias young men and women can be merciless in their
(A) assimilation
(B) condemnation
(C) implication
(D) confirmation
) 14. In the
, the outermost layer of skin, is about as thick as a sheet of paper over most of the skin.
(A) epidermis
(B) cochlear
(C) cortex
(D) cohort
) 15. The leaves of the white mulberry provide food for silkworms,
silk fabrics are woven.
(A) whose cocoons
(B) from whose cocoons
(C) whose cocoons are from
(D) whose cocoons

15 %

16 between men and women results in poorer health for children and greater 17 for the family, 18 to a new study. The
UN agency UNICEF found that in places where women are 19 from family decisions, children are more likely to suffer from
20 . There would be 13 million 21 malnourished children in South Asia if women had an equal say in the family, UNICEF said.
UNICEF surveyed family decision-making in 30 countries around the world. Their chief finding is that equality between men and
women is vital to 22 poverty and improving health, especially that of children, in developing countries. The conclusions are contained
in the agency's latest report. This report 23 to a greater 24 of opportunities for girls and women in education and work which
contributes to 25 and poverty. 26 men control the household, less money is spent on health care and food for the family, which
27 in poorer health for the children.
An increase in 28 and income-earning opportunities for women would increase their 29 power, the report said. For
example, the agency found that 30 has the greater share of household income and assets decides whether those resources will be used
for family needs.
(
D
) 16. (A) Unequal
(B) Inequal
(C) Unequality
(D) Inequality
(
B
) 17. (A) poor
(B) poverty
(C) poorness
(D) impoverish
(
C
) 18. (A) due
(B) regarding
(C) according
(D) with regard
(
A
) 19. (A) excluded
(B) exclude
(C) exclusion
(D) excludes
(
B
) 20. (A) malnourish
(B) malnutrition
(C) ill-nutrition
(D) ill-nourished
(
D
) 21. (A) more
(B) few
(C) less
(D) fewer
(
C
) 22. (A) reduce
(B) increase
(C) reducing
(D) increasing
(
A
) 23. (A) points
(B) indicates
(C) suggests
(D) shows
(
C
) 24. (A) lacks
(B) lacking
(C) lack
(D) lacky
(
D
) 25. (A) power
(B) empower
(C) empowerment
(D) disempowerment
(
A
) 26. (A) Where
(B) Which
(C) Whom
(D) What
(
D
) 27. (A) lead
(B) result
(C) leads
(D) results
(
C
) 28. (A) employ
(B) employee
(C) employment
(D) employed
(
B
) 29. (A) house
(B) household
(C) householder
(D) homemaker
(
C
) 30. (A) whatever
(B) whichever
(C) whoever
(D) wherever

10 %

Throughout North America and parts of Europe, beekeepers are noticing a disturbing trend.
31
This past winter, beekeepers
lost half of their colonies in a mystery scientists are struggling to solve.
32
However, the current situation, called Colony Collapse Disorder (CCD) is nothing like past die-offs. Losses are occurring
mostly because worker bees are unable to return to the hive, which is highly unusual behavior.
33
Worker bees collect pollen from flowering plants to feed the colonys young. In this process, called pollination, the bees fertilize the
plants, making it possible for them to reproduce. With one-third of American food crops dependent on bees for reproduction, CCD is
threatening the nations food supply.
34
Because bees also pollinate alfalfa, hay, and seeds, farm animals that consume these plants
could starve.
35
If scientists cannot figure out how to stop it, many people are afraid that the North American and European diets will soon be
reduced to grains and water.
(A) These losses have been rapid, have occurred in large numbers, and no one knows whats causing them.
(B) And its not just fruit and vegetables, but meat, too.
(C) Bees, which play a crucial role in farming, are rapidly dying out.
(D) Historically, bee colony die-offs are not all that uncommon.
(E) So far, researchers have been unable to determine whether CCD is resulting from disease, stress, chemicals, or other factors.
31.C

32.D

33.A

34.B

35.E

20 %

There are two reading passages. Please choose the best answer for each question

Passage A
Esperanto is what is called a planned, or artificial, language. It was created more than a century ago by Polish eye doctor Ludwik Lazar
Zamenhof. Zamenhof believed that a common language would help to alleviate some of the misunderstandings among cultures.
In Zamenhofs first attempt at a universal language, he trued to create a language that was as uncomplicated as possible. This first
language included words such as ab, ac, ba, eb, and ce. This did not result in a workable language in that these monosyllabic words, though
short, were not easy to understand or to retain.
Next, Zamenhof tried a different way of constructing a simplified language. He made the words in his language sound like words that
people already knew, but he simplified the grammar tremendously. One example of how he simplified the language can be seen in the
suffixes: all nouns in this language end in o, as in the noun amiko, which means friend, and all adjectives end in a, as in the adjective bela,
which means pretty. Another example of the simplified language can be seen in the prefix mal-, which makes a word opposite in meaning;
the word malamiko therefore means enemy, and the word malbela therefore means ugly in Zamenhofs language.
In 1887, Zamenhof wrote a description of this language and published it. He used a penname, Dr. Esperanto, when signing the book. He
selected the name Esperanto because this word means a person who hopes in his language. Esperanto clubs began popping up throughout
Europe, and by 1905 Esperanto had spread from Europe to America and Asia.
In 1905, the First World Congress of Esperanto took place in France, with approximately 700 attendees from 20 different countries.
Congresses were held annually for nine years, and 4,000 attendees were registered for the Tenth World Esperanto Congress scheduled for
1914, when World War I erupted and forced its cancellation.
Esperanto has had its ups and downs in the period since World War I. Today, years after it was introduced, it is estimated that perhaps a
quarter of a million people are fluent in it. Current advocates would like to see its use grow considerably and are taking steps to make this
happen.
(
B
) 36. The topic of the passage is
(A) how language can be improved
(B) one mans efforts to create a universal language
(C) a language developed in the last few years
(D) using language to communicate internationally
(
A
) 37. According to the passage, Zamenhof wanted to create a universal language
(A) to resolve culture differences
(B) to build a name for himself
(C) to create one world culture
(D) to provide more complex language
(
D
) 38. It can be inferred from the passage that the Esperanto word malespera means
(A) hopeful
(B) hope
(C) hopelessness
(D) hopeless
(
B
) 39. According to the passage, what happened to the Tenth World Esperanto Congress?
(A) It had attendees from 20 countries.
(B) It never took place.
(C) It was scheduled for 1915.
(D) It had 4,000 attendees.
(
C
) 40. The paragraph following the passage most likely discuss
(A) another of Zamenhofs accomplishments
(B) the disadvantages of using an artificial language
(C) how current supporters of Esperanto are encouraging its growth
(D) attempts to reconvene the World Congress of Esperanto in the 1920s

Passage B
Narcolepsy is disease characterized by malfunctioning sleep mechanics. It can consist of a sudden and uncontrollable bout of sleep
during daylight hours and disturbed sleep during nighttime hours. It occurs more often in men than in women, and it commonly makes its
appearance during adolescence or young adulthood. At least a half million Americans are believed to be affected by narcolepsy.
Narcolepsy can take a number of forms during daylight hours. One common symptom of the disease during daytime hours is a sudden
attack of REM (rapid-eye movement) sleep during normal waking hours. This occurs in some people hundreds of times in a single day,
while others only have rare occurrences. During a sleep attack, narcoleptics may experience automatic behavior; even though asleep, they
may continue automatically performing the activity they were involved in prior to falling asleep. Others experience cataplexy during
daytime hours; cataplexy involves a sudden loss of muscle tone that may cause the head to droop or the knees to wobble in minor attacks or
a total collapse in more serious attacks. Cataplexy seems to occur most often in conjunction with intense emotion or excitement.
During sleep hours, narcolepsy can also manifest itself in a variety of ways. During the transitional phase that precedes the onset of
sleep, it is common for hallucinations to occur. These hallucinations, known as hypnagogic phenomena, consist of realistic perceptions of
sights and sounds during the semi-conscious state between wakefulness and sleep. Narcoleptics may also suffer from night wakening during
sleep, resulting in extremely fragmented and restless sleep. Then, upon waking, a narcoleptic may experience sleep paralysis, the inability to
move, perhaps for several minutes, immediately after waking.
(
C
) 41. Which of the following would be the most appropriate title for this passage?
(A) Hallucinations during Sleep
(B) A Good Night Sleep
(C) An Unusual Sleep Disturbance
(D) A Cure for Narcolepsy
(
B
) 42. Which of the following would be most likely to occur during daily activities?
(A) Sleep paralysis
(B) Hallucinations
(C) Night wakening
(D) Automatic behavior
(
C
) 43. Which of the following would involve a complete collapse?
(A) Automatic behavior
(B) REM sleep
(C) Cataplexy
(D) Hallucinations
(
D
) 44. At which of the following ages would a person be most likely to develop narcolepsy?
(A) 55
(B) 40
(C) 35
(D) 20
(
A
) 45. When would hypnagogic phenomena most likely occur?
(A) Just after going to bed
(B) Soon after waking
(C) After getting up
(D) In the middle of the night

97

45 2 100

60 %
15 %
(

) 1. Petroleum is also used for producing goods ranging from plastics and laundry
to lipstick and clothing.
(A) determent
(B) detention
(C) detergent
(D) determinant
) 2. As the demand for food remains high and less food is being supplied, food prices around the world are
.
(A) soaking
(B) soaring
(C) sobering
(D) sobbing
) 3. With its modern design
bright and roomy stations, the new MRT system will carry Kaohsiung into a brave new
era of travel.
(A) fetching
(B) feigning
(C) fermenting
(D) featuring
) 4. In Tibet, the first incident occurred this March when around 400 monks left their
and marched to protest Chinas
restrictions on their religion.
(A) monody
(B) monocracy
(C) monarchy
(D) monastery
) 5. At Newcastle University in England, scientists have created
by implanting human DNA into cow eggs.
(A) embryos
(B) emeralds
(C) emigrants
(D) emperors
) 6. The government has taken action to combat online crime such as
advertising and credit card fraud.
(A) perceptive
(B) conceptive
(C) deceptive
(D) interceptive
) 7. After the encouragement of the teacher, the student began to
the verse from the first line to the last without stop.
(A) rave about
(B) turn over
(C) reel off
(D) make up
) 8. Bowling used to be a popular sport in Taiwan, but it hasnt really continued to
in the past few years.
(A) hold up
(B) set on
(C) put up
(D) catch on
) 9. From this spot, you can have a breathtaking
of the whole canyon.
(A) congestion
(B) sublime
(C) inquisition
(D) panorama
) 10. Because of the
of the instructions, she had no idea what to do next.
(A) erection
(B) perplexity
(C) ailment
(D) contemplated
) 11.
resistant is a simple idea: Bacteria that might be expected to be wiped out by a drug are instead unaffected by it.
(A) Genetic
(B) Pathogenic
(C) Antibiotic
(D) Subterranean
) 12. The new computer game Wii provides us with an
way of exercising. People now may play sports in their living
rooms, which was unimaginable before.
(A) innovative
(B) urgent
(C) aggressive
(D) assessorial
) 13. Saudi Arabias young men and women can be merciless in their
of those who flout them too brazenly.
(A) assimilation
(B) condemnation
(C) implication
(D) confirmation
) 14. In the
, the outermost layer of skin, is about as thick as a sheet of paper over most of the skin.
(A) epidermis
(B) cochlear
(C) cortex
(D) cohort
) 15. The leaves of the white mulberry provide food for silkworms,
silk fabrics are woven.
(A) whose cocoons
(B) from whose cocoons
(C) whose cocoons are from
(D) whose cocoons

15 %

16 between men and women results in poorer health for children and greater 17 for the family, 18 to a new study. The
UN agency UNICEF found that in places where women are 19 from family decisions, children are more likely to suffer from
20 . There would be 13 million 21 malnourished children in South Asia if women had an equal say in the family, UNICEF said.
UNICEF surveyed family decision-making in 30 countries around the world. Their chief finding is that equality between men and
women is vital to 22 poverty and improving health, especially that of children, in developing countries. The conclusions are contained
in the agency's latest report. This report 23 to a greater 24 of opportunities for girls and women in education and work which
contributes to 25 and poverty. 26 men control the household, less money is spent on health care and food for the family, which
27 in poorer health for the children.
An increase in 28 and income-earning opportunities for women would increase their 29 power, the report said. For
example, the agency found that 30 has the greater share of household income and assets decides whether those resources will be used
for family needs.
(
D
) 16. (A) Unequal
(B) Inequal
(C) Unequality
(D) Inequality
(
B
) 17. (A) poor
(B) poverty
(C) poorness
(D) impoverish
(
C
) 18. (A) due
(B) regarding
(C) according
(D) with regard
(
A
) 19. (A) excluded
(B) exclude
(C) exclusion
(D) excludes
(
B
) 20. (A) malnourish
(B) malnutrition
(C) ill-nutrition
(D) ill-nourished
(
D
) 21. (A) more
(B) few
(C) less
(D) fewer
(
C
) 22. (A) reduce
(B) increase
(C) reducing
(D) increasing
(
A
) 23. (A) points
(B) indicates
(C) suggests
(D) shows
(
C
) 24. (A) lacks
(B) lacking
(C) lack
(D) lacky
(
D
) 25. (A) power
(B) empower
(C) empowerment
(D) disempowerment
(
A
) 26. (A) Where
(B) Which
(C) Whom
(D) What
(
D
) 27. (A) lead
(B) result
(C) leads
(D) results
(
C
) 28. (A) employ
(B) employee
(C) employment
(D) employed
(
B
) 29. (A) house
(B) household
(C) householder
(D) homemaker
(
C
) 30. (A) whatever
(B) whichever
(C) whoever
(D) wherever

10 %

Throughout North America and parts of Europe, beekeepers are noticing a disturbing trend.
31
This past winter, beekeepers
lost half of their colonies in a mystery scientists are struggling to solve.
32
However, the current situation, called Colony Collapse Disorder (CCD) is nothing like past die-offs. Losses are occurring
mostly because worker bees are unable to return to the hive, which is highly unusual behavior.
33
Worker bees collect pollen from flowering plants to feed the colonys young. In this process, called pollination, the bees fertilize the
plants, making it possible for them to reproduce. With one-third of American food crops dependent on bees for reproduction, CCD is
threatening the nations food supply.
34
Because bees also pollinate alfalfa, hay, and seeds, farm animals that consume these plants
could starve.
35
If scientists cannot figure out how to stop it, many people are afraid that the North American and European diets will soon be
reduced to grains and water.
(A) These losses have been rapid, have occurred in large numbers, and no one knows whats causing them.
(B) And its not just fruit and vegetables, but meat, too.
(C) Bees, which play a crucial role in farming, are rapidly dying out.
(D) Historically, bee colony die-offs are not all that uncommon.
(E) So far, researchers have been unable to determine whether CCD is resulting from disease, stress, chemicals, or other factors.
31.C

32.D

33.A

34.B

35.E

20 %

There are two reading passages. Please choose the best answer for each question

Passage A
Esperanto is what is called a planned, or artificial, language. It was created more than a century ago by Polish eye doctor Ludwik Lazar
Zamenhof. Zamenhof believed that a common language would help to alleviate some of the misunderstandings among cultures.
In Zamenhofs first attempt at a universal language, he trued to create a language that was as uncomplicated as possible. This first
language included words such as ab, ac, ba, eb, and ce. This did not result in a workable language in that these monosyllabic words, though
short, were not easy to understand or to retain.
Next, Zamenhof tried a different way of constructing a simplified language. He made the words in his language sound like words that
people already knew, but he simplified the grammar tremendously. One example of how he simplified the language can be seen in the
suffixes: all nouns in this language end in o, as in the noun amiko, which means friend, and all adjectives end in a, as in the adjective bela,
which means pretty. Another example of the simplified language can be seen in the prefix mal-, which makes a word opposite in meaning;
the word malamiko therefore means enemy, and the word malbela therefore means ugly in Zamenhofs language.
In 1887, Zamenhof wrote a description of this language and published it. He used a penname, Dr. Esperanto, when signing the book. He
selected the name Esperanto because this word means a person who hopes in his language. Esperanto clubs began popping up throughout
Europe, and by 1905 Esperanto had spread from Europe to America and Asia.
In 1905, the First World Congress of Esperanto took place in France, with approximately 700 attendees from 20 different countries.
Congresses were held annually for nine years, and 4,000 attendees were registered for the Tenth World Esperanto Congress scheduled for
1914, when World War I erupted and forced its cancellation.
Esperanto has had its ups and downs in the period since World War I. Today, years after it was introduced, it is estimated that perhaps a
quarter of a million people are fluent in it. Current advocates would like to see its use grow considerably and are taking steps to make this
happen.
(
B
) 36. The topic of the passage is
(A) how language can be improved
(B) one mans efforts to create a universal language
(C) a language developed in the last few years
(D) using language to communicate internationally
(
A
) 37. According to the passage, Zamenhof wanted to create a universal language
(A) to resolve culture differences
(B) to build a name for himself
(C) to create one world culture
(D) to provide more complex language
(
D
) 38. It can be inferred from the passage that the Esperanto word malespera means
(A) hopeful
(B) hope
(C) hopelessness
(D) hopeless
(
B
) 39. According to the passage, what happened to the Tenth World Esperanto Congress?
(A) It had attendees from 20 countries.
(B) It never took place.
(C) It was scheduled for 1915.
(D) It had 4,000 attendees.
(
C
) 40. The paragraph following the passage most likely discuss
(A) another of Zamenhofs accomplishments
(B) the disadvantages of using an artificial language
(C) how current supporters of Esperanto are encouraging its growth
(D) attempts to reconvene the World Congress of Esperanto in the 1920s

Passage B
Narcolepsy is disease characterized by malfunctioning sleep mechanics. It can consist of a sudden and uncontrollable bout of sleep
during daylight hours and disturbed sleep during nighttime hours. It occurs more often in men than in women, and it commonly makes its
appearance during adolescence or young adulthood. At least a half million Americans are believed to be affected by narcolepsy.
Narcolepsy can take a number of forms during daylight hours. One common symptom of the disease during daytime hours is a sudden
attack of REM (rapid-eye movement) sleep during normal waking hours. This occurs in some people hundreds of times in a single day,
while others only have rare occurrences. During a sleep attack, narcoleptics may experience automatic behavior; even though asleep, they
may continue automatically performing the activity they were involved in prior to falling asleep. Others experience cataplexy during
daytime hours; cataplexy involves a sudden loss of muscle tone that may cause the head to droop or the knees to wobble in minor attacks or
a total collapse in more serious attacks. Cataplexy seems to occur most often in conjunction with intense emotion or excitement.
During sleep hours, narcolepsy can also manifest itself in a variety of ways. During the transitional phase that precedes the onset of
sleep, it is common for hallucinations to occur. These hallucinations, known as hypnagogic phenomena, consist of realistic perceptions of
sights and sounds during the semi-conscious state between wakefulness and sleep. Narcoleptics may also suffer from night wakening during
sleep, resulting in extremely fragmented and restless sleep. Then, upon waking, a narcoleptic may experience sleep paralysis, the inability to
move, perhaps for several minutes, immediately after waking.
(
C
) 41. Which of the following would be the most appropriate title for this passage?
(A) Hallucinations during Sleep
(B) A Good Night Sleep
(C) An Unusual Sleep Disturbance
(D) A Cure for Narcolepsy
(
B
) 42. Which of the following would be most likely to occur during daily activities?
(A) Sleep paralysis
(B) Hallucinations
(C) Night wakening
(D) Automatic behavior
(
C
) 43. Which of the following would involve a complete collapse?
(A) Automatic behavior
(B) REM sleep
(C) Cataplexy
(D) Hallucinations
(
D
) 44. At which of the following ages would a person be most likely to develop narcolepsy?
(A) 55
(B) 40
(C) 35
(D) 20
(
A
) 45. When would hypnagogic phenomena most likely occur?
(A) Just after going to bed
(B) Soon after waking
(C) After getting up
(D) In the middle of the night


60
200
: 100%
I. Idioms and Phrases 10%
1. We have decided to _____ the play for another week because so many people are coming to see it.
(A) hold over
(B) bring out
(C) read over
(D) sit out
2. Three robbers ________ in an old building. They refused to turn themselves into to the police. They even exchanged fire with
the police.
(A) held out
(B) pulled through
(C) dropped off
(D) messed around
3. His paper is poorly organized; I cannot__________.
(A) give him the benefit of the doubt
(B) step into his shoes
(C) see the wood for the trees
(D) separate the wheat from the chaff
4. Kevin is an excellent tour guide because he knows all the ________ of the itinerary.
(A) odds and ends
(B) ins and outs
(C) pins and needles
(D) ps and qs
5. His success is ______ because he fools around almost every day.
(A) on the scene
(B) beyond doubt
(C) on the contrary
(D) out of the question
6. If you break a window, do not ______; admit that you did it.
(A) eat dirt
(B) hit the jackpot
(C) pass the buck
(D) let the grass grow under your feet
7. Im not going to waste my time with you. I have ______.
(A) a white elephant
(B) a goose egg
(C) a dirty dog
(D) other fish to fry
8. Zhang Lian Wei started his career as a caddie, ______ and now has become the first Chinese golfer to play at one of the worlds
premier tournaments, the U.S. Masters.
(A) opted out
(B) showed his hand
(C) buttered up
(D) rose up the rank
9. The economy got ______ of inflation thanks to higher interest rates.
(A) over the bump
(B) in hand
(C) in the flesh
(D) to a hair
10. I ______. There is no way the police could have bumped into us accidentally.
(A) do a double take
(B) smell a rat
(C) make a mark
(D) hold my horses
II. Vocabulary Test 30%
1. Donation is not a _________ for the famine problems in that poverty-stricken country; instead, farming technology should be
introduced to help the people depend on themselves.
(A) panacea
(B) solace
(C) pseudonym
(D) celerity
2. Maria is so __________ to advertising that she buys whatever product promoted to her.
(A) reclusive
(B) fervid
(C) illicit
(D) susceptible
3. Leave your worries behind. This newly-designed machine will _________ us from the tedious work.
(A) concord
(B) emancipate
(C) feign
(D) familiarize
4. Not being able to endure the autocracy, a group of people _________ to overthrow the government.
(A) expunge
(B) conspire
(C) conserve
(D) exalt
5. Children growing up in caring and loving environment are liable to develop __________ personality.
(A) prolific
(B) quiescent
(C) insolent
(D) genial
6. The candidates _________ speech has brought the audience nothing but ennui.
(A) prolix
(B) aberrant
(C) erroneous
(D) indigent
7. To achieve fame and fortune, Sam is _________ to anyone in authority.
(A) arbitrary
(B) sullen
(C) obsequious
(D) loquacious
8. The wealthy entrepreneur is so _______ in donating money to charity that he wins much fame and respect.
(A) nostalgic
(B) nautical
(C) lavish
(D) snobbish
9. Mr. Andon is a _______ of antique furniture. Anyone who is interested in collecting antiques may ask for his advice.
(A) dilettante
(B) partisan
(C) prodigal
(D) connoisseur
10. After much effort, the mother finally _______ her naughty kid into going to bed in time.
(A) retaliated
(B) saturated
(C) coaxed
(D) shrieked
-1-


11. Adolescents are a

source of new words, creating dozens, even hundreds, of slang expressions that find their
way into mainstream dictionaries within a few years.
(A) supercilious
(B) surreptitious
(C) spurious
(D) superfluous
12. A good spokesman or representative should be astute enough to
some provocative questions that will wield
far-reaching consequences in public reactions.
(A) pander
(B) perforate
(C) pillage
(D) parry
13. My fathers tedious words are nothing but preaching, leaving me
and drifting away.
(A) obliterated
(B) jaded
(C) frittered
(D) careened
14. A background check in case of
and immorality is highly recommended for anyone who is going to be an
education practitioner or work with young people.
(A) depravity
(B) imbecility
(C) serendipity
(D) ubiquity
15. The Apache are a
society, where husbands typically move into wives dwellings and women take the
leadership role in family affairs.
(A) matricidal
(B) matriculated
(C) matrilineal
(D) matrimonial
III. Cloze 20%
(A) Desert plant populations have evolved sophisticated physiological and behavioral traits that aid survival in arid conditions.
Some send out long, unusually deep taproots; others utilize shallow 1 widespread roots, which allow them to absorb large,
intermittent flows of water. Certain plants protect their
2 to water. The creosote bush produces a potent root toxin that 3
the growth of competing root systems. Daytime closure of stomata exemplifiers a further genetic adaptation; guard cells work to
4
daytime water loss, later allowing the stomata to open when conditions are more favorable to gas exchange with the
environment.
Certain adaptations reflect the principle that a large surface area facilitates water and gas exchange. Most plants have small
leaves, modified leaves (spines), or no leaves 5 . The main food-producing organ is not the leaf but the stem, which is often
green and non-wood.
1. (A) and
(B) with
(C) but
(D) while
2. (A) process
(B) progress
(C) stress
(D) access
3. (A) inhibits
(B) inhabits
(C) protects
(D) remains
4. (A) maximize
(B) minimize
(C) magnetize
(D) emphasize
5. (A) instead
(B) at all
(C) for all
(D) indeed
(B) When were forced to make a choice between two unpleasant outcomes, the U.S. novelist Joseph Heller gave us a short and
wonderful description for this
6 : a catch-22. In his novel Catch-22, there is a fictional air force rule (Catch-22) that wartime air
force pilots must face: if a pilot continues flying dangerous combat missions without asking to be relieved of this life-threatening duty,
he must be insane. But if he asks his superiors for permission not to fly these missions, he would be sane, and therefore must carry on
flying.
The word catch, means a hidden problem or something within a situation that is undesirable and must be overcome. For
example, you could say you got a great new job with a high salary, but theres a catch: the job is on the night
7 . This means that
there is the bad (the catch) to go along with the good (a high-paying job).
A catch-22 is different,
8 , because there is no positive outcome. Heres is an example of a catch-22: Not exercising is bad
for your health, but if you exercise in a city with a lot of air pollution, thats bad for your health, too. So, you can see that no matter
what you choose and do in a catch-22 situation, you lose.
on the organization behind the war, and made fun of the ridiculous ideas and crazy
Hellers novel was an important
9
language used by
10
in an effort to control peoples thoughts and actions. The fictional catch-22 exemplified Hellers thoughts
on the war, and he must be pleased to see the phrase catch-22 come into common usage in fewer than 10 years after the book was
published.
6. (A) premiere
(B) holocaust
(C) dilemma
(D) gimmick
7. (A) term
(B) shift
(C) routine
(D) condition
8. (A) otherwise
(B) though
(C) contrarily
(D) consequently
9. (A) satire
(B) animosity
(C) pendant
(D) sitcom
10. (A) authorization
(B) intermediary
(C) regime
(D) bureaucracies
(C) In the late 1950s, Mao Zedong announced that the more people, the stronger we are. Unfortunately, Mao was
-2-


completely wrong. As the Chinese population 11
over the next few decades, the nation grew weaker and weaker. In 1979,
Deng Xiaoping started a birth planning initiative to curb the crisis. This project
12
the one-child policy which still exists
today.
The one-child policy 13 urban Han Chinese couples 13 a single offspring. This has helped not only slow down
population growth, but also reduce the extent of problems
14 from overpopulation. Such problems include public services,
such as health, education, and policing, being stretched thin. They also include environmental strains from over-farming and human
waste disposal. Unfortunately, the one-child policy has
15 () a new problem: an aging population.
Chinas population is aging at the fastest rate ever recorded. Health care costs are,
16
, rising like crazy. But the real
problem is that there are fewer people of working age to support retired elderly dependents. Many couples from single-child
families find themselves supporting their four parents
17 their own child. This situation is known as 4:2:1 phenomenon.
To reduce the burden on the working population, health officials are developing long-term care systems for the elderly. 18
have also been made to improve retired peoples access to public pension and encourage businesses to set up private ones.
19 ,
couples from single-child families, and those who have a masters degree, can now have two children if they wish. However, few
such couples wish to have even one child, 20 mind two.
Thus, the age imbalance in China is expected to grow and worsen
in the future.
11. (A) swept
(B) swelled
(C) swiped
(D) swooned
12. (A) reverted to
(B) exuded through
(C) traced back
(D) evolved into
13. (A) restores/to
(B) restricts/to
(C) relieves/of
(D) relates/with
14. (A) rising
(B) raising
(C) arising
(D) arousing
15. (A) given rise to
(B) brought about
(C) led to
(D) taken place
16. (A) as a result
(B) therefore
(C) for example
(D) now and then
17. (A) alone
(B) as well as
(C) beside
(D) additional
18. (A) Temptations
(B) Attentions
(C) Temperaments
(D) Attempts
19. (A) Therefore
(B) Furthermore
(C) Afterward
(D) However
20. (A) none
(B) not
(C) neither
(D) never
IV. Grammar and Rhetorics 20%
(For Questions 6-10, choose the best expression that with the closest meaning with the sentence underlined 20%)
1. I think no one can have what he wants all the time, _____?
(A) dont I
(B) do I
(C) cant he
(D) can he
2. Who is the handsome man _____ drives a luxurious car?
(A) who
(B) that
(C) which
(D) , who
3. Let the car ______ at once.
(A) wash
(B) being washed
(C) to be washed
(D) be washed
4. There is none ______ admires his talents and courage.
(A) as
(B) that not
(C) but
(D) but not
5. Playing the piano and singing simultaneously ______ difficult.
(A) is
(B) are
(C) are of
(D) were
6.The coffee is as tasty if not tastier than that tea.
(A) as tasty as if not tastier than that tea
(B) as tasty, if not tastier than, that tea
(C) as tasty if not, tastier than that tea
(D) as, tasty if not tastier than that tea
7. His sister, who is Secretary of the firm, has arrived.
(A) His sister is Secretary of the firm; she has arrived.
(B) His sister is Secretary of the firm and his has arrived.
(C) His sister is Secretary of the firm. She has arrived.
(D) His sister, Secretary of the firm, has arrived.
8. Pollution is injurious to health; it is also bad for emotional wee-being and upsets the ecological balance.
(A) Pollution is injurious to health, to emotional well-being, and upsets the ecological balance.
(B) Pollution is injurious to health, it is also bad for emotional well-being and to ecological balance.
(C) Pollution is injurious to health, to emotional well-being, and to ecological balance.
(D) Pollution injures health; it is also injurious to emotional well-being, and to ecological balance.
-3-


9. She wore a beautiful, and it is obvious that , expensive dress.
(A) a beautiful and expensive, it is obvious that, dress
(B) a beautiful and obviously expensive dress
(C) a, and it is obvious that beautiful and expensive dress
(D) an obviously beautiful and expensive dress
10. Walking along the track, a train suddenly appeared.
(A) Walking along the track, a train was suddenly seen.
(B) Walking along the track, suddenly seen by me a train.
(C) Walking along the track, it suddenly appears a train.
(D) Walking along the track, I suddenly was a train.
V. Discourse Structure 10%
Milton Friedman, who died in Nov. 2006, was one of the worlds most influential economists. In many ways, his ideas have
reshaped our very civilization. Friedman believed strongly in free markets, and a limited role of the government in the economy.
1 .
Friedman was awarded the 1976 Nobel Prize in Economics for his contribution to the field of money theory. In his theories,
the amount of money that is poured into the national economy can affect inflation and even be used to control it.
2 . Imagine
the price of everything rising steadily every year but your money being worth the same amount!
3 . To convey this message, Friedman wrote for popular magazines and even appeared in a TV series called Free to Choose
(you can watch this in its entirety on the Internet).
4 .
Friedmans political views mirrored his economic ones.
5 . Friedman said his proudest accomplishment was his major
role in the abolishment of the military draft. Since 1973, the armed forces of the United States have been filled solely with
volunteer soldiers. As Friedman predicted, the army is much more effective due to the change.
At the age of 94, after a long life of revolutionizing the field of economics, Milton Friedman passed away. His ideas, though,
are as alive as ever, and helping to make the world a better place.
(A) For example, he supported policies that imposed restrictions on governmental interference in peoples lives
(B) Most importantly, he believed in these not because they make the rich richer, but because they make all of society freer and
more prosperous
(C) By the 1980s, many governments, in particular those in England, America, and Canada, began to embrace Friedmans ideas
(D) You may not think the constant rise of prices is every important, but this problem was once a scourge on peoples lives
(E) Outside of controlling the supply of money, however, Friedman believed the government should minimize its guidance of the
economy
VI. Reading Comprehension 10%
I became a fan of Joseph Hellers writing while I was a student in high school in the 1970s. His most famous book,
Catch-22, was practically an anthem for myself and my friends. We had dissected it, sitting in the park outside school, reciting
certain key passages aloud and proclaiming to anyone who would listen that this was quite possibly the best book ever written.
Nearly twenty years later I am not sure that we were wrong.
Heller created a modern-day anti-hero who was a soldier trying to stay sane I the midst of a war in which he no longer believed.
This spoke to my generation, growing up as we did during the turmoil of Vietnam, and --- however you felt about the issue --- his
ideas were considered important.
I had spent many hours imagining what the man who had created the savage wit and brilliant imagery of that book would be like
I person. I was soon to find out. To this day, I have no idea how it was arranged, but somehow an invitation to speak at my high
school was extended and duly accepted.
On the day, I made sure to be near the gate of the school to see him arrive. I was looking for a limousine, or perhaps an
entourage of reporters surrounding the man whose dust-jacket picture I had scrutinized so often. But suddenly, there he was,
completely alone, walking hesitantly toward the school like just a normal person. He walked by me, and I was amazed to see that he
was wearing rather tattered sneakers, down at the heel.
When he began speaking in the auditorium, I was dumbfounded, for he had a very heavy speech impediment.
That cant be him, I whispered loudly to a friend. He sounded like a dork.
My notions of a brilliant man at that time did not extend to a speech impediment --- or any handicap whatsoever. Ordinary
people were handicapped, but not men of brilliance. There was, in fact, a fair amount of whispering going on I the auditorium.
-4-


And then somehow, we began to listen to what he was saying. He was completely brilliant. He seemed to know just what we
were thinking and articulated feelings that I had only barely known that I had. He spoke for forty minutes and held us all spell-bound.
I would not have left my seat even if I could.
As I listened, I began to feel awaken in me the possibility of being more than I had supposed that I could be. With some
difficulty I managed to get to the school gate again and waited for twenty minutes while I supposed he signed autographs and fielded
questions inside the auditorium. Eventually, he came out, as he had come in, alone.
I crewed up all my courage and called to him, Mr. Heller.
He almost didnt stop but then he turned around and came over to me.
I just wanted to say how much I enjoyed your book.
He looked down at me in my wheelchair, smiled as if it was the most normal thing in the world and shook my hand. I think that
day may have been very important in the future direction of my life.
1. To the author, Joseph Hellers novel, Catch-22 was
(A) too traditional for most readers.
(B) unusual in its frank portrayal of high school students and their problems.
(C) inspiring and thought-provoking
(D) an important but little-known work
2. The major purpose of the passage is to
(A) profile a famous novelist.
(B) describe an event that may have changed the authors perception of himself.
(C) suggest the sense of disappointment the author felt at encountering his hero.
(D) relate in dramatic form the authors early childhood memories.
3. The author describes Hellers speech (lines 15-21), primarily in order to
(A) show that the students initial skepticism was overcome by their interest in what he was saying.
(B) illustrate the wit and imagery of the novelists ideas.
(C) respond to charges that Hellers work is overrated.
(D) demonstrate that lack of respect that was shown to the novelist because of his speech impediment.
4. The word fielded, in line 23, most nearly means
(A) evaded
(B) asked
(C) delved into
(D) responded to
5. The author most likely remembers his handshake with Heller because
(A) Heller had taken the time to come to visit a high school fan, even though he was a celebrity.
(B) Heller almost didnt stop to shake his hand.
(C) it was a form of recognition from someone who had overcome his own obstacles.
(D) the author had been so unimpressed by Hellers speech at his high school

I. Idioms and Phrases 10%


1.A

2.A

3.C

4.B

5.D

6.C

7.D

8.D

9.A

10.B

6.A

7.C

8.C

9.D

10.C

II. Vocabulary Test 30%


1.A

2.D

3.B

4.B

5.D

11.D

12.D

13.B

14.A

15.C

III. Cloze 20%


1.C

2.D

3.A

4.B

5.B

6.C

7.B

8.B

9.A

10.D

11.B

12.D

13.B

14.C

15.D

16.A

17.B

18.D

19.B

20.D

4.C

5.A

6.A

7.D

8.C

9.B

10.D

IV. Grammar and Rhetorics 20%


1.D

2.B

3.D

-5-


V. Discourse Structure 10%
1.B

2.D

3.E

4.C

5.A

4.D

5.C

VI Reading Comprehension 10%


1.C

2.B

3.A

-6-

97 1 1

Part I:

30%

Each sentence has a missing word. Choose the word that best completes the sentence. There are more words
in the list than you need.
procrastinate

satiate

crafty

temerity

peruse

tentative

tactful

obfuscate

prudent

variable

expedite

versatile

variable

expedite

ecstasy

constraint

vindictive

____________ 1. She is an extremely ____________ actress who can play many different roles equally well.
____________ 2. She could never ________________ his appetite for fishing and boating.
____________ 3. If he were a(n) _______________ person, he would have demanded similar punishment for
them.
____________ 4. The stuntpeople in the film have such ____________ that they drive their cars through flames
and them jump off a cliff.
____________ 5. Her personality is as _______________ as the weather.
____________ 6. We were surprised that someone as __________________ as Jane would do something so
foolhardy.
____________ 7. Jim and Janes house is in complete disrepair because they tend to __________ about
everything that they have to do.
____________8. A ________________ man need not be 200 pounds, 6 feet tall, and muscular.
___________ 9. Although Mary had been told that the arrangement was a(n) _____________ one, she had hoped
that it would be permanent.
___________ 10. The rigid time schedule is a(n) ___________ that will probably prevent me from taking this job.
___________ 11. Mary tried to think of a(n) _____________ way to tell her boyfriend that she was unhappy with
their relationship.
___________ 12. We were all in a state of _________________ when we learned that we had been chosen as
representatives of our school to tour Europe.
___________ 13. Mr. Brown asked if the manufacturing department could ______________ the delivery of the
needed parts because his department had promised delivery to its customer a week ago.
___________ 14. Make sure that you _____________ the contract before you sign it.
___________ 15. Please dont invite Bob to explain the issue because he will do more to ______________ it than
anyone I know.

Part II: Cloze & Reading comprehension: (30%)

I : Alexander Graham Bell invented the telephone and became a very famous man. He also wrote an
article which he presented in 1883. His research paper was about deaf people.
In his article, Dr. Bell explained why there were so many deaf children. He believed that when deaf
adults married each other, they would have deaf children. He thought that this was bad. Bell blamed the
schools for the deaf for causing marriages between deaf people. He did not like the idea of these
intermarriages and tried to think of ways to stop them. He would have preferred that deaf children be
taught at hearing schools. He did not ____1______ schools for the deaf. Bell felt that deaf people would
not mix with hearing people if they went to _____2_____ schools.

Bell was ____3______ about other things, too. He noticed that deaf people socialized with other
deaf people. He felt that socializing with other deaf people was bad. Deaf people should socialize with
hearing people, he thought. Bell tried to start a new law that would make it illegal for deaf people to
marry each other. He ____4______ up his idea when he realized that such a law could not be enforced.
Bell also had strong feelings about the ____5______ of deaf children. He wanted deaf children to
be with hearing children in school. He thought that the deaf children could learn normal behavior from
the hearing children. Bell was against the use of deaf teachers, too, because he thought this added to the
____6______ of a deaf "race" in America.
Bell had another theory which he never proved. He believed that deaf children who signed would
not have good English skills. He could never prove this, and now many people have tried to prove the
opposite! A lot of new research shows that many deaf children whose parents sign (and are deaf) do
_____7_____ schoolwork than deaf children who do not sign at home!
_________1.(A) observe (B) describe (C) support (D) visit (E) study
_________2. (A) larger (B) nicer (C) worse (D) older (E) separate
_________3. (A) selfish (B) upset (C) shy (D) careless (E) unsure
_________4. (A) fixed (B) gave (C) wrote (D) talked (E) printed
_________5. (A) discipline (B) health (C) safety
(D) effort (E) education
_________6. (A)) ability (B) loss (C) problem (D) cost (E) success
_________7. (A) more (B) easier (C) slower (D) better (E) quieter
II. Read the text and answer the questions. Then do Question 8~15

When he was eight, he emigrated to the United States of America, where his uncle tended gardens
in San Francisco. The Chink picked up English and other bad habits. He went to high school and other
dangerous places. He earned American citizenship and other dubious distinctions.
When asked what he wished to do with his life, he answered (although he had learned to
appreciate movies, jukebox music and cheerleaders) that he wanted to grow yams on the side of a
volcano - but as that was impracticable in the city of San Francisco, he became, like uncle, a gardener.
For more than a dozen years he made the grass greener and flowers flowerier on the campus of the
University of California at Berkeley. [...]
By special arrangements with his employers, the Chink attended one class a day at the university.
Over a twelve-year span he completed a good many courses. He never graduated, but it would be a
mistake to assume he did not receive an education.
He was astute enough to warn his relatives, on December 8, 1941, the day after Pearl Harbour,
"The shinto is gonna hit the fan. We'd better get our yellow asses back to some safe volcano and eat
yams till this blows over."
They didn't listen. After all, they were patriotic, property-owning, tax-paying American citizens.
The Chink wasn't anxious to flee, either. He was in love again. Camping on the rim of a different
volcano. So to speak.

On February 20, 1942, came the order. Two weeks later, the Army took steps. In March,
evacuation was in full swing. Some 110,000 people of Japanese ancestry were moved out of their homes
in "strategic" areas of the West Coast and settled in ten "relocation" camps further inland. They could
bring to camp only what they could carry. Left behind were houses, businesses, farms, home furnishings,
personal treasures, liberty. Americans of non-Nip ancestry bought up their farmland at ten cents on the
dollar (The crops failed). Seventy percent of the relocated people had been born and reared in the US.
"Loyal" Japanese were separated from "disloyal." If one would swear allegiance to the American war
effort - and could pass an FBI investigation - one had the choice of remaining in a relocation camp or
finding employment in some non strategic area. The camps were militaristic formations of tarpaper
barracks, supplied with canvas cots and pot-bellied stoves. Six to nine families lived in a barracks.
Partitions between "apartments" were as thin as crackers and did not reach the ceiling. (Even so, there
were an average of twenty-five births per month in most camps.) There was no great rush to leave the
camps: a loyal family that had been relocated on an Arkansas farm had been killed by an irate anti-Jap
mob.
Disloyal Japanese Americans - those who expressed excessive bitterness over the loss of their
property and the disruption of their lives, or who, for various other reasons, were suspected of being
dangerous to national security - were given the pleasure of one another's company at a special camp, the
Tule Lake Segregation Center in Siskiyou County, California. The Chink had been asked if he supported
the American war effort. "Hell no!" he replied! "Ha ha ho ho and hee hee." He waited for the logical
next question, did he support the Japanese war effort, to which he would have given the same negative
response. He was still waiting when the military police shoved him on the train to Tule Lake
(Taken from "Even Cow Girls Get the Blues" by Tom Robins, 1976)
__________8. When the war broke out, the hero was .
(A) a member of the FBI
(B) a recently immigrated Japanese citizen.
(C) an American citizen of Japanese origin.
(D) a Japanese student at Berkeley University.
__________9. Is this sentence true or false?
"As a result of his integration he gave up all Japanese tradition."
____________10. Is this sentence true or false?
He managed to get a degree."
____________11. Is this sentence true or false?
"He was aware of a possible threat and advised his family to leave."
____________ 12. Is this sentence true or false?
"His views (about the potential danger for Japanese citizens) proved to be right."
_____________13. The narrator suggests that the Americans sent the Japanese Americans to special
camps. . ..
(A) For good reasons: they represented a real threat for the country as foreigners
(B) For false reasons: they were suspected of being potential spies in spite of their

belonging to the American nation.


______________14. Is this sentence true or false?
"He was extremely surprised at the decision that was taken against him."
______________15. Is this sentence true or false?
"He condemned the war effort on both sides."
III: Translation:

20%

1: Concision is highly prized in science. The premium on pith is enshrined in perhaps sciences most
important law, known as the law of parsimony, or Ockhams razor. It states, in essence, that when
confronted with two or more explanations for a phenomenon, we assume that the more compact, less
complicated, simpler one must be correct. (please translate this passage to Chinese)

2.

[] , ,
, ()

IV:

Essay: 20%
Please talk about what you think about English language teaching.

97 1 1
Part I:

30%

Part II: Cloze & Reading comprehension: (30%)

1~7: C E B
8 : C 9: False
Part III: Translation
IV:

Essay:

20%

B E C
D
10. False 11: True 12: True 13: B

14: True 15: True

97 1

D1.We are grateful for his


in making a large contribution to our company.
(A)hypothesis (B)wrinkle (C)trauma (D)generosity
C2.British farmers
their cattle to stop the disease from spreading.
(A)distilled (B)coaxed (C)slaughtered (D)vibrated
.
A3.He was sentenced to 10 years for being a cocaine
(A)trafficker (B)garrulity (C)frailty (D)guile
B4.The politician could never speak simply; he was always
.
(A)baleful (B)grandiloquent (C)inherent (D)latent
; throw it away quickly.
D5.This corrupted meat is so
(A)cryptic (B)bombastic (C)laconic (D)malodorous
and preferred to be alone most of the time.
A6.He was not
(A)gregarious (B)momentous (C)ostensible (D)parsimonious
laws of nature.
C7.Scientists are constantly seeking to discover the
(A)obsolete (B)ominous (C)immutable (D)synthetic
me into doing this; I hate force.
D8.Do not
(A)haggle (B)infringe (C)harass (D)coerce
speeches can strike a chord
B9.He is an excellent public speaker and his
in his audiences heart.
(A)ambiguous (B)eloquent (C)eligible (D)impending
drowsiness, so I had better not take the pills
A10.Pills for a cold often
tomorrow morning for fear that I may doze off while I am taking my English
test.
(A)induce (B)taunt (C)hurl (D)perish
error in the sentence On arriving at the station, the
C11.There was a
train had left . The subject of the verb
left should be a person, not the train.
(A)infectious (B)subordinate (C)manifest (D)spontaneous
our thirst with drinking water instead of soft
B12.Doctors advise us to
drinks like cola. Even juice is not desirable because it contains a lot of sugar.
(A)retaliate (B)quench (C)deteriorate (D)incense
of the enemy
A13.We suffered many casualties during the unexpected
troop.
(A)onslaught (B)levy (C) constituency (D)vanity
D14.Doctors advised people to practice good hygiene including hand washing
contaminated surfaces if anyone has become ill.
and

(A)bestowing (B)subduing(C)vying (D)disinfecting


existence.
D15.After his years of adventure, he could not settle down to a
(A)pious (B)tenuous (C)mammoth (D)humdrum

D1.alleviate: (A)transpose
(B)fortify
(C)rectify
(D)exacerbate
B2.caustic: (A)incompatible
(B)innocuous
(C)anesthetic
(D)nonflammable
A3.bolster: (A)decrease support of (B)lose faith in
(C)disconnect
(D)delete
C4.facilitate: (A)hallow
(B)hurdle
(C)hamper
(D) hide

:
1.
2.
3.
4.
5.
6.
7.
8.
9.
10.

:(A)(E)


()
Garbage disposal involves getting rid of the waste food from private homes,
stores, and warehouses. The largest amount of garbage is leftovers from private
homes. (1)D Waste and rotting food attracts all kinds of insects, rodents, and
bacteria, and is a great health menance if it is not removed. (2)C Most cities
today regulate the disposal of garbage. (3)E Others use treated garbage to fill
in land. (4)A Modern incinerators are designed to consume waste materials
and the resulting gases completely. (5)B
(A) A third way of disposal is to set up incinerating plants for burning
garbage.
(B) These devices are considered a vital part of air-pollution control
equipment.
(C) Disease-carrying flies lay their eggs in garbage, and rats feed on it.
(D) The disposal of this waste food creates one of the big problems of
modern life, especially in crowded towns
and cities.
(E) Some have chemical treatment plants which convert garbage to
fertilizers.
()
Massachusetts has been first in many areas. (6)C The state had
outlawed slavery 80 years before Lincoln did so for the whole country. The
nations first subway was built here in 1896. (7)D
(8)A The most famous is also the countrys oldest. Harvard University
was founded in 1636 to train clergy, but you can study a lot more than religion
there today. In the some neighborhood is M.I.T., founded in 1861. (9)E Feel
free to visit. Both of these schools are open to the public. (10)B They will not
give you the go-ahead unless you have high test scores and good grades.
(A) More than anything else, the progressive spirit Massachusetts comes
from its colleges and universities.
(B) Getting in is another story.
(C) The first shots of the American Revolutoin were fired here.
(D) And basketball was first played here in 1896.
(E) The school is respected worldwide for its scientific research.

()
1. She always avoids eye ______ with people she doesnt know well.
(A) contract (B) contact (C) inspection (D) injury
2. I ruined my brothers cell phone. _______, I have to tell him the bad news.
(A) The sooner, the better (B) Later that day (C) Late last night (D) Sooner or later
3. If you are not satisfied with the purchase, you may _____ it for a refund within one week.
(A) return (B) obtain (C) prolong (D) stock
4. By the time you arrive in Bangkok, we _______ in Thailand for three days.
(A) are (B) will be (C) have been (D) will have been
5. The dermatologist used a special solution to _____ the warts on my finger.
(A) remain (B) remove (C) recall (D) recover
6. There were two witnesses, but I wouldnt trust _____ of them.
(A) all (B) either (C) each (D) most
7. My aunt lives in a very _____ area; the nearest large city is one hundred miles away.
(A) urban (B) rural (C) local (D) initial
8. Rita is unwilling to _____ her Barbie dolls for anything. They mean a lot to her.
(A) trace (B) track (C) trade (D) trample
9. It is only when Im alone ______ study hard.
(A) can I (B) so I can (C) so as to (D) that I can
10. No one in the tour group spoke German, so we had to ask the tour guide to _____ for us.
(A) interpret (B) interrupt (C) intervene (D) interview
()
Questions 1-3
In the past, if someone had problems deep in his insides, he had to have a lot of uncomfortable tests to
find the cause of the problem. Now, all you have to do is swallow a camera. Of course, it isnt a common
camera. This camera is about an inch long --- the same size as a large pill. After you swallow it, the camera
will travel through your body the same way that food does.
This new camera pill is a great invention because it can get to parts of the body that other tests cant
reach. As the patient goes about his or her day, the camera travels through their body and takes pictures. The
patient also wears a special belt. At the end of the day, the patient gives the belt to the doctor. The pictures are
put on a computer, and the doctor can see whats wrong.
1. To whom is this camera good news?
(A) Doctors.
(B) Photographers.
(C) People with stomach problems.
(D) A dentists patients.
2. Whats special about the camera?
(A) Its small enough to swallow.
(B) It is light enough to carry around.
(C) It is only available in some hospitals.
(D) It has gone through many tests.
3. How does the doctor get the pictures?
(A) Out of the patients stomach.
(B) The pictures are received by a special belt.

(C) He looks for them on the Internet.


(D) He gets them by opening the camera.
Questions 4-7
There are estimated to be about 2,000 species of fireflies. They are the little bugs that fly through the
air and light up. At night, we see little flashes all over as they try to attract mates. At first the flashes may
look like they are all happening at different times. But what researchers have found is that each male firefly is
trying to be the one that flashes first. They do this because female fireflies are most likely to mate with males
that have flashed the earliest. Because all the males are competing to attract females, they will flash four to
eight times, and then pause for a few seconds before starting again. What ends up happening, though, is that
in their competition to be first, they all end up flashing at the same time!
4. What would be a good title for this article?
(A) How to Attract a Mate
(B) Insect Dating
(C) Competition Among Fireflies
(D) 2,000 Species of Insects
5. Who does the underlined word they refer to?
(A) Female fireflies.
(B) Researchers.
(C) Male fireflies.
(D) Little flashes.
6. According to the article, what do we see at night?
(A) Fireflies getting ready to sleep.
(B) Little flashes all around us.
(C) People trying to catch insects.
(D) Thousands of insects being eaten by birds.
7. Who are the female fireflies most likely to mate wtih?
(A) The male that barely flashes.
(B) The male that flies the fastest.
(C) The male that grows the fastest.
(D) The male that flashes earliest.
Questions 8-10
Dear Mr. Zany,
This letter is to inform you that you have successfully completed the writers workshop. It seems that
all your hard work has paid off. We knew when you came into the course that creative writing was not your
strongest point. But over time you managed to learn the necessary skills needed to write a good story. We
have received your last piece entitled, Typhoon Town, and are pleased to say we enjoyed it very much.
Your writing is lively, entertaining, and you are very skilled at making the readers feel as if they were
actually involved in the story. We hope you will be able to continue with this new-found passion for writing.
You may pick up your certificate in person at our offices or, if you wish, we can mail it to your home address.
Come back and visit us sometime.
Sincerely,
Matt Totti
8. What does paid off in the second sentence mean?
(A) That he made a lot of money.
(B) That he has been working too much.
(C) That the money he spent was worth it.
(D) That he has benefited from his efforts.
9. What can we assume Mr. Zany didnt have much of because he started the writing course?
(A) Writing experience.
(B) Intelligence.
(C) Excitement in his life.
(D) Education.
2

10. What does Mr. Totti hope will happen to Mr. Zany in the future?
(A) That his enthusiasm for writing remains.
(B) That he succeeds in everything he does.
(C) That he becomes a famous writer.
(D) That he writes a story about himself.
()
1.
2.
3. (Wikipedia)
4.
5.

1. BDADB
6. BBCDA

1. CABCC
6. BDDAA


50 ( 2 100 )
Vocabulary and Phrases
1. Studies show that those constantly exposed to second-hand smoke are just as _______ lung disease as smokers are.
(A) susceptible to
(B) concerned about
(C) addicted to
(D) aware of
2. One reason that many people have so much difficulty losing weight is that
factors appear to play an important role in obesity.
(A) metallic (B) metabolic (C) meta-analytic (D) medieval
3. Jane did not realize how difficult it was to
child-rearing
a career.
(A) copewith (B) dealwith (C) strugglewith (D) jugglewith
4. Pass away is a(n)
for die.
(A) euphemism (B) evaporation (C) embryo (D) embodiment
5. The American government
against Iraq by breaking off diplomatic relations.
(A) retailed (B) retrieved (C) retaliated (D) revitalized
6. Inflation will never be
from the economy.
(A) elaborated (B) eradicated (C) enforced (D) reinforced
7. The terrorists
their hostages
terrible torture.
(A) subjectedto (B) subordinatedto (C) submittedto (D) subscribeto
8. The natural resources of this national park are subject to various threats, including exotic ______, pollution, and so on.
(A)spices
(B) life (C) livings (D) species
9. The city is ______ for gang violence. One can read about robberies and murders in the newspaper nearly every day.
(A) notorious (B) famous (C) eminent
(D) unknown
10. David tried to ______ his parents to allow him to travel alone for a week.
(A) prevent
(B) deter
(C) dissuade
(D) convince
11. Unfortunately, the area you are about to move in has a high rate of juvenile ______.
(A) integrity (B) uprightness
(C) delinquency
(D) decency
12. Helen ______ from our company because she was offered a more challenging job at a renowned international corporation.
(A) initiated
(B) resigned
(C) embarked
(D) resided
13. These issues will not be dealt with in this chapter but will be discussed at length in ______ chapters.
(A) suspicious (B) subsequent
(C) subject
(D) substantial
14. He has been ______ of murdering an innocent child and will be sentenced to life in jail.
(A) convicted (B) acquitted
(C) released
(D) emitted
15. This box of _____ cookies from Denmark represents the finest flavors in Scandinavian baking.
(A) musty (B) assorted (C) exported (D) porous

Grammar
16. It ______ absolutely amazing to have seen this place back then, says John Smith, an archaeologist, because these islands were
far richer in biodiversity in those days than they are today..
(A) would be
(B) is
(C) would have been
(D) has been
17. John and Mary
married for thirty years by the end of this year. They are planning to invite their children,
grandchildren, relatives, and friends to a big party to celebrate their wedding anniversary.
(A) have been (B) are
(C) will have been (D) have
18. John read a number of articles assigned by his English teacher, ______ were rather difficult to understand.
(A) most of which (B)most of what (C) most of who (D) most of them
19. Contemporary industrial methods have replaced individual crafts,
cobblers, coopers, and stone carvers
practically vanished.
(A) to make blacksmiths,

(B) make blacksmiths,

(C) made blacksmiths,

4 1

(D) making blacksmiths,

Error Identification In each sentence there are four underlined words. Choose the word(s) marked (A), (B), (C),
or (D) that has/have to be changed in order for the sentence to be correct.
20. Up to the 1970s, there was a predominant assumption that a single best method for teaching English existing.
(A)
(B)
(C)
(D)
21. The Cloze procedure is a problem-solved exercise in which the student has to exploit linguistic clues not only in the linguistic
(A)
(B)
(C)
context but also in the wider context of situation.
(D)
22. Be linguistic and cultural ambassadors, EFL teachers play a unique and important role in helping EFL students master the
(A)
(B) (C)
(D)
English language.
23.

Melt water from Icelands largest glacier thunders into a canyon, drawing tourists to the precipice. Until now most raging rivers
(A)
(B)
(C)
like thisa huge potential source of hydroelectric powerhad remained wild.
(D)

Reading ComprehensionPlease select one best answer to each of the following questions which are based on
the preceding passage.
24.

Throughout the world, engineers have been infamous for their general lack of concern for the environmental impact of their
activities, and, consequently, they have contributed to the decline in the quality of our surroundings.
This means that ______.
(A) engineers should show less concern for the well-being of people and their environment to improve our lives.
(B) engineers should stop driving cars which pollute our environment to improve our lives.
(C) engineers should strive for a greater impact on their environment to improve our lives.
(D) engineers should be aware of and avoid designing sources of pollution to improve our lives.

25.

One drop of rain doesnt make a shower and the performance of a single word or deed does no make a saint.
(A) It is an ill wind that blows no body any good.
(B) One swallow does not make a summer.
(C) It never rains but it pours.
(D) There is no smoke without fire.

26.

In 1551, an English physician named Harvey published a book on the blood circulation. Although Harveys explanation was not
universally accepted at the time, medical men of today credit him with one of the greatest advances in medical history.
This passage does not say anything about Harveys
(A) reputation.
(B)medical training.
(C) field of research.
(D) profession.

27.

When we make a resolution aloud in the company of others, there is the added dimension of commitment. This means, if we
make a firm decision in the presence of our friends, ________.
(A) we have to bear a big burden so heavy that we could possibly give it up.
(B) we must consider it important that we have to promise the fulfillment of our decision.
(C) we will make up some excuse for being unable to carry out our decision.
(D) well probably change our mind to give up our plan.

4 2

28. I would not have said it when she was there if I had thought that it would shock her.
(A) What I said made her shocked, but it was quite without intention.
(B) I did not think she would mind what I said.
(C) I regret that I said it to her face; it did shock her.
(D) To my regret, what I said did not shock her.
29. Any fool can tell the truth, but it requires a man of sense to know how to lie well.
This sentence implies that________.
(A) fools are less dangerous than wise men.
(B) only a sensible man can tell a lie.
(C) to tell beautiful lies needs more wisdom than to tell the truth.
(D) sometimes a wise man is worse than a fool.
30. So far, no traffic accidents have been attributed to the missing street-light poles.
(A) No sooner had the street-light poles been removed than traffic accidents happened in succession.
(B) Owing to traffic accidents, a few street-light poles are pulled up.
(C) A few street-light poles are attributed to traffic accidents. In other words, if there had been no traffic accidents, there would
be no street-light poles.
(D) Although some street-light poles are found missing, traffic accidents are not said to have had anything to do with it.

Cloze Test
Passage (A)
Asthma rates in children are
31
the rise. Some experts say it may be related to our sterile lifestyle and excessive use
of antibiotics. Exposure 32
germs sets the immune system into fighting mode and thats a good thing. Children with
the H. pylori bacterium are half as likely to have asthma
33
those who dont.
31.
32.
33.

(A) in
(A) for
(A) at

(B) at
(B) to
(B) to

(C) to
(C) at
(C) as

(D) on
(D) in
(D) if

Passage (B)
As cultures continue evolving, humans gain an increasing ability to control and modify their environment. The same
intellectual resourcefulness that permitted early humans to discover and use fire and to invent useful tools 34 the agricultural
revolution, the industrial revolution, and the technological revolution. Cultural evolution,
35 the adaptive changes of cultures
in response to changes in the environment over time, is possible only because humans have been 36 with a capacity for
learning and language. Lets not forget,
37 , that cultural evolution is a mixed blessing. With it has come pollutions, many
social ills and injustices (for example, racism), and over-population.
34. (A) prompt
(B) to prompt (C) prompted
(D) prompting
35. (A) and
(B) or
(C) in contrast to (D) accompanied by
36. (A) genetic endow (B) genetic endowed (C) genetically endowment
37. (A) though
(B) in short
(C) for example

(D) genetically endowed


(D) vice versa

4 3

Passage (C)
Prejudice often leads to discrimination. Although we most often think of prejudice and discrimination 38
at the level of
the individual, we often see these phenomena operating at the level of groups, even nations. At this level, they 39 rise to
ethnocentrism, which is the notion that ones own cultural, national, racial, or religious group is superior
40 or more
deserving than others. Apparently, affiliation and prejudice are two sides of the same 41 . That is,
42 the tendency to
identify with and feel close to members of our own group or clan goes the tendency to the suspicious
43
others.
Unfortunately, prejudice seems to be an enduring characteristic of the human species.
38. (A) occur
39. (A) make
40. (A) X
41. (A) knife
42. (A) along with
43. (A) against

(B)occurs
(C) occurred
(B) give
(C) go
(B) than
(C) to
(B) mirror
(C) window
(B) as well as
(C) X
(B) at
(C) of

(D) occurring
(D) push
(D) over
(D) coin
(D) in spite of
(D) on

Passage (D)
Technology has come a long way
44
the time of Thomas Edison, inventor of the electric light bulb, the electric motor
and the phonograph,
45
other things. Edison was also the main
46
for the first commercial central power system in
New York City. Today, nearly a century after Edisons most
47
period of invention, with the
48 of the Computer Age,
many of the functions that Edison brought to life
49
into
50
mobile communication devices.
44. (A) before
(B) since
(C) because of
(D) with respect to
45. (A) accompanied
(B) added
(C) among
(D) with
46. (A) conductor (B) delegate
(C) conspirator
(D) mastermind
47. (A) rabid
(B) seminal
(C) estranged
(D) superlative
48. (A) advent
(B) initiation (C) sophistication (D) prosperity
49. (A) have compacted (B) are included
(C) are being incorporated (D) are making
50. (A) radical
(B) corporeal
(C) surrogate
(D) state-of-the-art

4 4


1.

2.

3.

4.

5.

6.

7.

8.

9.

10.

11.

12.

13.

14.

15.

16.

17.

18.

19.

20.

21.

22.

23.

24.

25.

26.

27.

28.

29.

30.

31.

32.

33.

34.

35.

36.

37.

38.

39.

40.

41.

42.

43.

44.

45.

46.

47.

48.

49.

50.

97

7 100
10:00 11:40 100

2B

8. I received
Asomething
Bnothing
Canything
Deach one

I. Vocabulary & Grammar


their children from the time they
1. Parents hope teachers to
get to school until the time they leave.
Aadmonish
Bsupervise
Ctaunt
Dupstage
2. Live neither in the past nor in the future, but let each days work
your entire energies.
Aabsorb
Bdetach
Ccollocate
Dconfirm
after all the

4. Its always better to


search for a solution afterwards.
Afoster
Bauthorize
Cimmigrate
Danticipate

a problem before it arise than to

5. Though I only caught a


her beauty.
Aappetite
Bglimpse

of her. I was deeply attracted by

to

10. The more one is


to the English-speaking environment,
the better he or she will learn the language.
Akeep on
Bexposed
Ccatch on
Dfill in
11. When filling out a form, you usually have to
you are.
Asurvive
Bindicate
Cinstall
Ddonate
12. Most people consider it
theater.
Abeing rude
Bbe rude
Cabout rude
Drude

what sex

to talk on a cell phone in the

13. The field of molecular genetics is moving at a rapid pace. Our ability
to ________ genes and understand the complex processes involved
in genetics is developing on almost a daily basis.
Amultiply
Bmanifest
Cmanipulate
Dmanufacture
14. We have to
Ahop to it
Bhit it off
Ctake it hard

if we are to catch the bus.

Dface it out

Cminister
Dlegislator
6. My sister has a high
like that.
Aillusion
Bgrievance

beyond what was due to me.

9. They were behind schedule and had to apply for extra


complete their project in time.
Acombination
Bcompetition
Ccountenance
Dmanpower

3. Ashia couldnt believe Lucas didnt want to


meetings they had on this project.
Aswallow the hammer
Bhammer his way
Ccome under the hammer
Dhammer out a deal

of herself. She wont take a job

15. The company went broke because there wasnt enough


products.
Ainterest
Bshelf

for its

Cdemand
Dsale

Copinion
Dimpetus
7. He may be a little careless in his behavior, but his loyalty to his friend
is
.
Aout of order
Bon the right track
Cout of question
Din a stew

16. Tyler was given a final warning about his behavior at school. Mr.
Brown told him that he was walking on
ice.
Athin
Bloose
Cdelicate
Dfragile
17. They went to the country to
Aexpose
Benter
Cexchange
Dexult

1 / 8

noise for quietness.

28. By todays standards, early farmers were imprudent because they


planted the same crop repeatedly, exhausting the soil after a few
harvests.
Astubborn
Btiresome
Cunwise
Dunscientific

18. Howard __________ in a forest __________ in a hotel.


Aprefers camping , staying
Bprefers to camp , than to stay
Cprefers to camping , instead of staying
Dwould rather camp , than stay
19. There is a kind of ___________ education system in that country.
Atests-orienting
Btest-oriented
Ctest-orienting
Dtests-oriented
20. Only after he had spoken __________ he had made a big mistake.
Ahe had realized
Bhe realized
Ccould he realize
Ddid he realize
21. ___________ for the bus, _________ fell out of his pocket.
ARunning, Johns cell phone
BRan, his cell phone
CAs John was running, his cell phone
DWhen he is running, Johns cell phone

29. Human facial expressions differ from those of animals in the degree
to which they can be deliberately controlled and modified.
Aboth
Babsolutely
Cnoticeably
Dintentionally
30. When Jack suffered a return of his illness, he was asked to enter the
hospital.
Arelapse
Brebuff
Creform
Dresort
31. The Mona Lisa is the portrait of a woman with a very enticing smile.
Aluminous
Belusive
Coblivious
Dalluring

22. People who spend too much time online may be ________ to the
Internet.
Aaddicted
Bpredicted
Cexcluded
Dpersuaded

32. Because of the extreme pressure underwater, divers are often slow in
response.
Ashabby
Bsluggish
Cshrewd
Dsinuous

23. The police broke up a gang of criminals who were ________


morphine into the country.
Astruggling
Bstealing
Csmuggling
Dscrapping
24. Our company used to have rows and rows of filing cabinets to store
information on paper before our office systems were ________.
Astandardized
Bcomputerized
Cfertilized
Dindustrialized
25. That vacation locale has several ________ resorts; only certain
people are allowed to use their facilities.
Aexclusive
Bcreative
Csensitive
Ddistinctive

33. Winning a scholarship to Harvard was a kind of achievement you can


be proud of; 200 students had applied for it.
Aworm on your way
Bfeather in your cap
Cnail in your head
Dchip on your shoulder
34. Mary, its ridiculous for us to quarrel. Lets make peace and be
friends again.
Astick to our guns
Blead a dogs life
Cbury the hatchet
Dlose our shirts

26. Every year in Japan, senior high school students face ________
university entrance exams. The exams are extremely difficult and the
preparation period is grueling.
Agive-and-take
Bmake-or-break
Cfar-off
Dinside-out
27. Some radical antiglobalizationists, called localists, believe that
international trade and activity should be greatly ________ and that
power should be returned to the national, regional, and local level.
Asailed
Bdetailed
Cretailed
Dcurtailed

35. The murderer ruined his chances when he returned to the scene of the
crime.
Agot away with the wind
Bcooked his goose
Cpassed the time of day
Dgave it up as a bad job
36. Susan is very shy; she doesnt enjoy being the center of attention.
Akeeping a straight face
Bknown her number
Con the spot
Din the limelight

2 / 8

II. Close
Although earthquakes can cause death and destruction through
__37__ secondary effects as landslides, tsunamis, fires, and fault rupture,
the greatest losses __38__ both lives and property resulted from the
collapse of man-made surface and subsurface structures during the
violent shaking of the ground.
Seismologists routinely gather
considerable quantities of data to explain the characteristics of the
recorded ground motion that take place during earthquakes. Such
knowledge is needed to predict ground motions in future earthquakes so
that earthquake-resistant structures can be designed.

After the UFO sightings and the Roswell incident of 1947, U.S.
Air Force General Nathan Twining undertook a study of the strange
objects that were seemingly ___40_____ through the skies. This study
group was given the code name Project Sign. Project Sign studied 147
cases; 135 cases could be explained, but 12 could not. The group decided
that those 12 sightings were actually of ___41_____ from another planet.
Air Force Chief of Staff General Vandenberg rejected the report. He said
that the evidence gathered did not support such an astounding conclusion.
Vandenbergs rejection ruined ___42_____ at Project Sign, and the case

The most effective way to reduce the destructiveness of


earthquakes is to design and construct buildings and other structures
capable of withstanding strong shaking. When a site is proposed for the
construction of an office building, for example, factors such as the
geometry and frictional __39_ of a nearby fault line, the passage of
seismic waves surrounding the building must be considered.

officers were soon merely collecting and filing reports, and did very little
investigative work. They felt they had done their best and that the brass
had rejected it.

In 1949, Project Sign became Project Grudge. Project Grudge took


a different approach. The researchers opted to try to explain every UFO

37.
Alike
Bsuch
Cso
Dso-called
38.
Aat the thought of
Bin favor
Cin terms of
Dat the risk of
39.
Aproperties
Bevidences
Csources
Dpossibilities

report no matter how fragmentary. In 1952, Project Grudge went public


and was renamed Project Blue Book. However, after 12 years of
investigation, the project found no compelling evidence that UFOs were
either ____43____. or a threat to national security.

40.
Acrashing
Battacking
Czipping
Drotating
41.
Acreatures
Bsmall devices
Cinventions
Dcraft
42.
Amorale
Bpowers of observation
Csalary
Ddevotion
43.
Aextraordinary
Bextraditable
Cextraterrestrial
Dextraneous

3 / 8

III. Reading Comprehension


Dreams and prophesies aside, one of the greatest mysteries
surrounding the Sphinx today is one with more practical implications:
How much longer will this great statue survive? Much of it was already
worn away ages ago, including the nose, parts of the cobra on the
forehead, and the long beard that once extended from the chin. Currently,
not only the natural elements, but also pollution from the nearby city of
Cairo is taking the toll. In addition, past efforts to restore the statue have
sometimes, ironically, done more harm than good. For example, faulty
reconstruction methods using concrete instead of limestone caused
changes in the proportions of the statue. More recent preservation plans
are much more sophisticated, although sometimes still experimental.
However, even if the Great Sphinx were to collapse tomorrow, even if it
should eventually crumble or become covered again with the sands of
time, the end of its physical presence would certainly not mean an end to
the many mysteries that have surrounded it for so long. Indeed, the
"enigma" of the Great Sphinx is likely to remain with us for many
centuries to come.
44. Which of the following is the best title for the passage?
AThe Location of the Sphinx.
BThe Structure of the Sphinx.
CThe Material of the Sphinx.
DThe Mysterious Sphinx.

Flynn limited the possible explanations when he looked carefully at


the test data and discovered that the improvement in scores was only on
certain parts of the IQ test. Test-takers didnt do better on the arithmetic
or vocabulary sections of the test; they did better on sections that
required a special kind of reasoning and problem solving. For example,
one part of the test shows a set of abstract shapes, and test-takers must
look for patterns and connections between them and decide which shape
should be added to the set. According to Flynn, this visual intelligence
improves as the amount of technology in our lives increases. Every time
you play a computer game or figure out how to program a new cell
phone, you are exercising exactly the kind of thinking and problem
solving that helps you do well on one kind of intelligence test. So are
you really smarter than your parents? In one very specific way, you may
be.
46. The Flynn effect is ________.
Aused to measure intelligence
Ban increase in IQ test scores over time
Cunknown in some parts of the world
Dnot connected to our experiences
47. The Flynn effect must be the result of ________.
Aheredity
Bour environment and experiences
Ctaking fewer tests
Dmemorizing information

45. The underlined word "enigma" in the last line probably means
_______.
Amystery
Bpresence
Cdisappearance
Dphysical absence

48. Which sentence from the article gives a main idea?


AScientists have proposed several explanations for the Flynn
effect.
BBecause we take so many tests in our lives, we learn test-taking

-------------------------------------------------------------------------------------Do you think youre smarter than your parents and grandparents?
According to James Flynn, a professor at a university in New Zealand,
you are! Over the course of the last century, people who have taken IQ
tests have gotten increasingly better scores on average, three points
better for every decade that has passed. This improvement is known as
the Flynn effect, and scientists want to know what is behind it.
IQ tests and other, similar tests are designed to measure general
intelligence rather than knowledge. Flynn knew that intelligence is partly
inherited from our parents and partly the result of our environment and
experiences, but the improvement in test scores was happening too
quickly to be explained by heredity. So what was happening in the 20th
century that was helping people achieve higher scores on intelligence
tests?

Scientists have proposed several explanations for the Flynn effect.


Some suggest that the improved test scores simply reflect an increased
exposure to tests in general. Because we take so many tests, we learn
test-taking techniques that help us perform better on any test. Others
have pointed to better nutrition since it results in babies being born larger,
healthier, and with more brain development than in the past. Another
possible explanation is a change in educational styles, with teachers

techniques that help us perform better on any test.


CTest-takers didnt do better on the arithmetic or vocabulary
sections of the test.
DFor example, one part of the test shows a set of abstract shapes,
and test-takers must look for patterns and connections between
them and decide which shape should be added to the set.
49. According to the article, newer educational techniques include
________.
Aexposure to many tests
Bchildren finding things out themselves
Cmemorizing information
Dimproved test scores
50. Which statement would Professor Flynn agree with?
APeople today are more intelligent in every way.
BPeople today have fewer problems to solve.
CPeople today are taking easier tests.
DPeople today have more visual intelligence.

51.

A
B
C
D

encouraging children to learn by discovering things for themselves rather


than just memorizing information. This could prepare people to do the
kind of problem solving that intelligence tests require.
4 / 8

52.Bloom

A
B
C
D
53.

A
B
C
D
54.

A
B
C
D
55.

A
B
C
D
56.
A
B
C
D
57.
A
B
C
D
58.

A
B
C
D
59.

AGagne
BVygotsky
CPiaget
DBruner
60.
A
B
C
D

61.

A
B
C
D
62.
A
B
C
D
63.
A
B
C
D
64.(collge)
A
B
C
D
65.
A
B
C
D
66.

Ausubel
A
B
C
D
67.
A
B
C
D
68.

A
B
C
D
69.

A
B
C
D
70. Herbart

A
B
C
D

5 / 8

71.

A
B
C
D
72.

A
B
C
D
73.
A
B
C
D
74. 5 6

A
B
C
D
75.

A
B
C
D
76. 96 2 26
0960013140C

77.
?
A
B
C
D
78.the third culture
Ascience, technology and society
Bscience and humanity
Cinformation, technology and society
Dscience, technology and humanity
79.magnet school
A
B
C
D

80.pragmatism

A
B
C
D
81.

A
B
C
D
82.PISA
A
B
C
D
83.Peter Drucker 1954

A
B
C
D
84.
Pioneering Program

A
B
C
D
85.

A
B
C
D
86.
AKarl Marx
BAnthony Giddens
CPaul Freire
DMartin Heidegger
87.
Abanking pedagogy
B
C
D
88.A. Maslow

A
B
C
D
89.
A
B
C
D

6 / 8

90.

A
B
C
D

91.2008

B
C
D 3

92.

A
B
C
D
93. 4

A
B
C
D
94.

A
B
C
D
95.

A
B
C
D
96.

()
()
()
()
A
B
C
D
97.

?
A
B
C
D

98.

?
A

D
99.
?
A
B
C
D
100.

A
B
C
D

7 / 8

97 <>
1

10

11

12

13

14

15

16

17

18

19

20

21

22

23

24

25

26

27

28

29

30

31

32

33

34

35

36

37

38

39

40

41

42

43

44

45

46

47

48

49

50

8 / 8

97

I. Vocabulary and Phrases: Choose the most appropriate answer. 30 points


1. The critics comments were unnecessary and
added them.

, no one could understand why he

(A) gracious (B) gratuitous (C) gregarious


2. The prosecutors were looking for the
hold-up.
(A) accessory

(D) grotesque

who had provided the car used in the

(B) accomplice (C) defendant (D) defector

3. By the time the police arrived at the bank to question the employee suspected of
embezzlement, he had 3 with one million US dollars.
(A) absconded

(B) besieged (C) deposed (D) exhumed

4. After much coaxing, the belligerent child

cleaning his bedroom.

(A) adverted to (B) assented to (C) asserted to

(D) averted to

5. By placing mirrors along the hall way, the school creates the

of spaciousness.

(A) allusion (B) collusion (C) delusion (D) illusion


6. Because all the students wear blue blazers and red ties, they are
(A) homogeneous

in that respect.

(B) homogenized (C) homogenous (D) homologous

7. In order to raise money quickly, the company decided to


not important to its profits.

itself

assets

(A) divest . . . from (B) diversify . . . into (C) divest . . . of (D) divest . . . with
8. Unfortunately these favorable influences will
next few years.

or even disappear within the

(A) abate (B)defray (C) recur (D) vanish


9. He is the chief 9 of his aunts will. After taxes he is left with an inheritance
worth close to one billion dollars.
(A) beneficiary (B) contestant
10. The teacher took a

10

(C) executor (D) pensioner

pleasure in the achievements of her students.

(A) uncanny (B) unethical (C) vacuous (D) vicarious


11. His

11

behavior was marked by a simple inability to handle a knife and fork.

(A) abrasive (B) convivial (C) quixotic


1

(D) uncouth

12. The manager of the museum put the sign 12 on the door so that the crowd
would move on, expecting to see another exhibit.
(A) Advent (B) Egress

(C) Morass

(D) Ratio

13. The
13 of the candidate expressed itself in his extreme and unrealistic
isolationism.
(A) acrophobia

(B) claustrophobia (C) homophobia (D) xenophobia

14. The conspirators met

14

in the night to exchange information.

(A) amenably (B) blatantly (C) clandestinely (D) discretely


15. The teacher handed out a sample letter as a
(A) facsimile

(B) hyperbole

(C) metaphor

15

of the correct form.

(D) paradigm

II. Cloze: Choose the most appropriate answer. 40 points

Humankind lingers unregenerately in Platos cave, still reveling, its age-old habit,
in mere images of the truth. But being educated by photographs is not like being
educated by older, more artisanal images._ 16 , there are a great many more images
around, claiming our attention. The inventory started in 1839 and since then just about
everything has been photographed, 17
so it seems. This very 18 of the
photographing eye changes the terms of confinement in the cave, our world. In teaching
us a new 19 code, photographs alter and enlarge our notions of what is worth
looking at and what we have a right to observe. They are a grammar and, even more
importantly, an 20 of seeing. Finally, the most grandiose result of the photographic
enterprise is to give us the sense that we can hold the whole world in our headsas an
anthology of images.

16. (A)On the contrary (B)For one thing (C)For the time being (D)At the same time
17. (A) and (B) but

(C) for (D) or

18. (A) affiliation (B) conflagration (C) insatiability (D) propinquity


19. (A) dressing (B) gustatory (C) olfactory (D) visual
20. (A) aesthetics (B) ethics (C) ideology (D) orifice

It depends on how far the process had gone, of transmuting life into truth.
2

21

the completeness of the distillation, so will the purity and imperishableness of the
product be. 22 none is quite perfect. As no air-pump can by any means make a
perfect vacuum, so neither can any artist entirely 23
the conventional, the local,
the perishable from his book, or write a book of pure thought that shall be as efficient,
in all respects, to a remote posterity, as to contemporaries, or rather to the 24
age.
Each age, it is found, must write its own books;
25
, each generation for the next
succeeding. The books of an older period will not fit this.

21. (A) Because of (B) In proportion to (C) In spite of


22. (A) And (B) Thus (C) So
23. (A) exclude (B) exhume
24. (A) first

(B) last

25. (A) for example

(D) Thanks to

(D) But

(C) include (D) inter

(C) second (D) third


(B) however (C) or rather

(D) to boot

Most of the luxuries, and many of the so called comforts of life, are not only not
26
, but positive hindrances to the elevation of mankind. With respect to luxuries and
comforts, the wisest have ever lived a more simple and
27 life than the
poor. The ancient philosophers, Chinese, Hindoo, Persian, and Greek, were a class than
which none has been poorer in outward riches, non so rich in inward. We know not
much about them. It is remarkable that we know so much of them 28
we do. The
same is true of the more modern reformers and benefactors of their race. None can be
an impartial or wise observer of human life but from the
29 ground of what we
should call voluntary poverty. There are nowadays professors of philosophy, but not
philosophers. To be a philosopher is not merely to have subtle thoughts, nor even to
found a school, but so to love wisdom 30
live according to its dictates, a life of
simplicity, independence, magnanimity, and trust. It is to solve some of the problems of
life, not only theoretically, but practically.

26. (A) adequate (B) germane (C) indispensable (D) ubiquitous


27. (A) corpulent (B) meager (C) noxious
28. (A) as (B) like (C) until
29. (A) foresight
30. (A) as to

(D) transitory

(D) when

(B) hindsight (C) vintage (D) vantage

(B) in order to (C) not to (D) with a view to

The experience of change and loss exerts an incalculable influence on the


3

development of Western culture. Western


31 and Western religion derive from
that experience, especially as it led to repeated attempts to distinguish between
appearance and reality. Broadly speaking, the world we experience was said to be the
world of 32 , the domain of unreality, deception, loss, transience, and deathto
be contrasted with an ultimate, changeless reality which was either deeper within or
entirely
33 the world of experience. The
34 or transcendent reality was
also said to be the source of absolute, as distinct from relative, truth, and 35 of
eternal life. Some of the greatest literature in the West derives from the tension
between the desire for that ultimate reality to exist, and thereby redeem loss, and the
conviction that, in reality, it does not.

31. (A) astrology (B) geometry (C) hermeneutics (D) metaphysics


32. (A) appearance (B) essence (C) opacity (D) transparency
33. (A) below (B) beyond (C) within (D) without
34. (A) ephemeral (B) immanent

(C) sentient (D) transitory

35. (A) also (B) one (C) even (D) some

III. Reading Comprehension: Choose the most appropriate answer. 30%

At Christies May 5 New York auction of impressionist and early modern art, the
financial meltdown thats been gripping the globe for the better of a year was nowhere in
evidence. The worlds wealthy snapped up Monets, Rodins, and Giacomettis at
record-setting prices. American buyers were in the minority, as a swarm of Europeans
flew in to take advantage of tumbling dollars. Christies CEO Edward Dolman says
wealthy Russians accounted for another big chunk of the $277 million takethe houses
third highest ever for modernist art. Billionaire collector Nicolas Berggruen, the French
founder of Berggruen Holdings, says his bids were outgunned each time, with some of
the better-known worksincluding Rodins Evefetching two or three times
Christies pre-auction estimates. Ive been waiting for the prices to come down ever
since the financial crisis started, but its not happening, he says, Its bizarre. Logically
it should all come falling down, but it isnt.

36. Which of the following statements is true?


(A) Rodins Eve is among the auction.
(B) The auction is about post-impressionist arts.
(C) Nicolas Berggruen gets what he wants.
4

(D) The auction is influenced by the financial meltdown.


37. What is the passage mainly about?
(A) The worlds superwealthy are not affected by the financial meltdown.
(B) Christies auction in New York is very successful.
(C) Russians are the biggest consumers of art.
(D) The prices of artistic works go down because of financial meltdown.
38. Which of the following statements is NOT true?
(A) Logically, the prices should go down with the financial meltdown.
(B) American buyers are the minority in the auction.
(C) European buyers come to take advantage of the tumbling dollars.
(D) Nicolas Berggruen bids the highest price for Rodins Eve.
39. Whose work is NOT in Christies New York auction?
(A) Rodin (B) Monet (C) Giocometti David

In 1989 France celebrated the Bicentennial of the Revolution which overthrew


the ancient rgime and established the First Republic. Culminating in a spectacular
display on the Place de la Concorde on 14 July, the celebrations were an affirmation of
Frances revolutionary origins and its republican tradition, a tradition, which, through
the first two-thirds of the nineteenth century, was eclipsed by Bonapartism but which,
with the hiatus of the Occupation, had run uninterrupted from 1871. The events in Paris
on 14 July 1989 were important as a statement about Frances political and social
identity at the end of the twentieth century, but they also adopted a cultural format of
extreme theatricality in which high cultural and popular cultural traditions merged: the
evenings climax consisted of the American soprano Jessye Norman singing the
Marseillaise whilst circling the obelisk at the center of the Place de la Concorde
entwined in a huge tricolore flag. Not for the first time did France choose to celebrate
its historical origins with a cultural event of high theatricality which also adopted the
format of the carnival. At the same time, the Bicentennial celebrations were by no
means the result of spontaneous or piecemeal enthusiasm. On the contrary, they were
the culmination of years of planning at the highest levels of the State, overseen and
orchestrated by the Minister of Culture, Jack Lang. As such, they were confirmations of
the central role of the State in the operation of late twentieth-century, as early
twentieth-century, France, and of the importance it accords to culture as a means of
affirming national identity.

40. When did France establish its First Republic?


(A) 1789 (B) 1848 (C) 1852 (D) 1871
5

41. Where did the climax of the Bicentennial celebrations take place?
(A) La Defense (B) Arc de Triumph (C) Hotel de Ville (D) Place de la Concorde

42. Which of the following statements is true?


(A) French Republican tradition had run uninterrupted since 1848.
(B) The celebration was orchestrated by the Minister of Justice.
(C) Two-thirds of the 19th Century France was dominated by Bonapartism.
(D) Carnival theatricality appeared for the first time in this event.
43. Which of the following statements is NOT true?
(A) The celebration was an important statement about Frances national identity.
(B) An American soprano sang Frances National anthem at the celebration.
(C) The Bicentennial celebration was the result of a spontaneous and piecemeal
enthusiasm.
(D) Jack Lang supervised the Bicentennial celebration himself.

Trafficking has been identified by the international Organization for Migration as


the most menacing form of irregular migration due to its ever increasing scale and
complexity involving, as it does, arms, drugs and prostitution. The UN Office on
Drugs and Crime describes it as the worlds fastest-growing international organized
crime. But it remains a topic riddled with contradictions, anomalies, and differences of
definition, with deep divisions over how to deal with it among both national and
international organizations. Reliable figures are impossible to come by. Officials at the
International Labour Organization say only that they believe that between 700,000 and
two million women and children are trafficked across an international border
somewhere in the world every year, feeding an industry with profits estimated at
somewhere between $12 billion and $ 17 billion per year. According to the United
Nations, there are currently 127 source countries that provides large numbers of
prostitutes, mainly in Asia and Eastern Europe, and 137 destination countries.

44. According to the United Nations, besides Asia, where do prostitutes in the
international market mainly come from?
(A) Africa (B) South America (C) East Europe (D) Middle East

45. Which of the following statements is true?


(A) Precise information about trafficking of women and children is impossible to get.
(B) Consensus has been reached as to how to deal with trafficking.
(C) There are more source countries than destination countries of women
trafficking.
(D) Trafficking can be easily defined.
46. Which of the following is NOT a problem involved in the issue of trafficking?
(A) anomaly (B) contradiction (C) willpower (D) definition

Decoding Advertising sets out not simply to criticize advertisements on the


grounds of dishonesty and exploitation, but to examine in detail, through over a
hundred illustrations, their undoubted attractiveness and appeal. The overt economic
function of this appeal is to make us buy things. Its ideological function, however, is to
involve us as individuals in perpetuating the ideas which endorse the very economic
basis of our society. If it is economic conditions which make ideology necessary, it is
ideology which makes those conditions seem necessary.
If society is to be changed, this vicious circle of necessity and ideas must be
broken. This book is an attempt to undo one link in the chain which we ourselves help
to forge, in our acceptance not only of the images and values of advertising but of the
transparent forms and structures in which they are embodied. It provides, not an
answer, but a set of tools which we can use to alter our own perceptions of one of
societys subtlest and most complex forms of propaganda.

47. Which description is NOT true about advertising?


(A) It is dishonest and exploitive.
(B) It is indispensable to economy.
(C) It is ideologically free.
(D) It is attractive and appealing.
48. Which of the following statements is true?
7

(A) The consumers are not responsible for the ideology behind advertising.
(B) Advertising is a subtle and complex form of propaganda.
(C) The book provides answers as to how the society should be changed.
(D) Advertising has nothing to do with how we see the world.

49. In your opinion, what is Decoding Advertising mainly about?


(A) How TV commercials are produced.
(B) Advertising is loaded with Marxist ideology.
(C) How to analyze the ideology behind advertising.
(D) Ideology is responsible for the vicious circle in society.
50. In this passage, what does the vicious circle in the second paragraph refer to?
(A) The interdependence between economy and ideology
(B) The interdependence between product and advertising
(C) The interdependence between consumption and advertising
(D) The interdependence between advertising and ideology

97

10 11 12 13 14 15 16 17 18 19 20

B B A B D A C A A D D B D C D B D C D B
21 22 23 24 25 26 27 28 29 30 31 32 33 34 35 36 37 38 39 40

B D A C C C B A D A D A B B C A A D D A
41 42 43 44 45 46 47 48 49 50

D C C C A C C B C A

97

I. Reading Comprehension:(10%)
Imagine if you ate half your weight in food at one sitting, then your metabolism would really
have to go into overdrive. That's the case with the Burmese python, which like other carnivorous
snakes can gobble up to half its weight in rats or other animals at a time. Scientists have long
known that snakes greatly increase their metabolism over the two to three weeks it takes to digest
such a meal. Now, researchers at the University of California, Irvine, show how the python does
it. After a rat-feast, it rapidly enlarges its heart. Dr. James Hicks and colleagues dissected and
studied python hearts at various stages - fasting, 48 hours after eating, and after digesting. They
found that after eating, the python's ventricle (there's only one, snakes having three-chambered
hearts) increased in mass by 40 percent. The findings were reported in the journal Nature.
According to Dr. Hick, the heart of a python gets bigger not because the cells are filling with
fluid nor because it builds more cells; instead, it takes the existing cells and packs them with more
contractile protein. That protein, cardiac myosin, allows the muscle cells that make up the heart
to lengthen and enlarge, a condition called hypertrophy. So the snake can pump more blood with
each heartbeat and get more oxygen to tissues to use with the nutrients being digested. The
process is reversible; four weeks after eating, the snakes' hearts had returned to normal. The
researchers found that during digestion there was increased activity by the gene responsible for
expression of cardiac myosin. "There's this incredible ramping up of the genetic machinery," Dr.
Hicks said. But other kinds of myosin - that found in muscles attached to the skeletal system, for
example - showed no change. "There isn't some sort of global response throughout the animal."
Dr. Hicks said that understanding the snake's remarkable ability might be useful in studying the
general process of cardiac hypertrophy. Currently, researchers typically use pigs or rats,
exercising them on treadmills for weeks so that their hearts eventually enlarge. "Here you have
an animal that you feed and two days later you have a 40 percent increase in heart size," he said.
--Adapted from the New York Times March 8, 2005
1. According to the passage, how does a Burmese python increase its metabolism to digest the
prey?
(A) It dissects and studies the prey to increase the metabolism.
(B) It gobbles up to half its weight in other animals one at a time.
(C) It quickly enlarges its heart in order to accelerate the metabolism.
(D) It strengthens its muscles attached to the skeletal system to speed up the metabolism.
2. What is the reason that contributes to the hypertrophy during a pythons digestion?
(A) There are more cells filling with fluid.
(B) There are more new cells built in the heart. .
(C) The cells are packed with more contractile protein.
(D) The gene responsible for expression of cardiac myosin decreases activity.
1

3. Which type of animals is mentioned in studying the process of cardiac hypertrophy?


(A) Pigs.
(B) Rats.
(C) Snakes.
(D) All of the above.
4. What is the best title of the passage?
(A) How People Digest Food.
(B) How Proteins Work in Animals.
(C) How A Python Captures Its Prey.
(D) How A Python Enlarges Its Heart.

II. Test Design:


Part A: Design a cloze test with 5 test items in the form of multiple choice questions
based on the article provided.0
One recent report said that many elementary and junior high school teachers in Taipei suffer
from depression. The news is causing much concern in educational circles and among parents of
small children.
A survey conducted by the Taipei City Teacher's Association between March and May
found that 13 percent of the city's junior high and elementary school teachers have the propensity
to suffer from depression, while 28 percent of those polled had latent depression.
In related news, many elementary and junior high school teachers around the age of 50 are
anxious to retire, even though they can teach for many years if they want to. This might suggest
that teachers experience a lack of satisfaction from their jobs. Most educators enjoy good pay
and long holidays, but the demands of their jobs are stressful and exhausting. Whatever the
reason, Taiwan is facing the possibility of a shortage of qualified teachers if teachers retire earlier.
The two reports speak about two serious problems facing public education. It goes without
saying that the system needs to prevent depression and attrition, the two problems raised in the
reports, or the education of Taiwan's youths will undoubtedly be jeopardized.
Part :Design a reading comprehension test with 5 test items in the form of
multiple choice questions based on the article provided.0
Are you preparing for a big test? If so, you may want to go play some basketball in between
hitting the books. Doctors are starting to find more and more information that suggests a
connection between exercise and brain development. Judy Cameron, a scientist at Oregon Health
and Science University, studies brain development. According to her research, it seems that
exercise can make blood vessels, including those in the brain, stronger and more fully developed.
Dr. Cameron claims this allows people who exercise to concentrate better. As she says, While we
2

already know that exercise is good for the heart, exercise can literally cause physical changes in
the brain.
The effects of exercise on brain development can even be seen in babies. Babies who do
activities that require a lot of movement and physical activity show greater brain development than
babies who are less physically active. With babies, even a little movement can show big results.
Margaret Barnes, a pediatrician, believes in the importance of exercise. She thinks that many
learning disabilities that children have in elementary school or high school can be traced back to a
lack of movement as babies. Babies need movement that stimulates their five senses. They need
to establish a connection between motion and memory. In this way, as they get older, children will
begin to associate physical activity with higher learning, says Margaret.
Older people can beef up their brains as well. Cornell University studied a group of seniors
ranging in age from seventy to seventy-nine. Their study showed a short-term memory increase of
up to 40 percent after exercising just three hours a week. The exercise does not have to be very
difficult, but it does have to increase the heart rate. Also, just like the motion for infants, exercise
for older people should involve some complexity. Learning some new skills or motions, such as
with yoga or tai-chi, helps to open up memory paths in the brain that may not have been used for a
long time.
For most people, any type of physical activity that increases the heart rate is helpful. The
main goal is to increase the brains flow of blood. And your brain can benefit from as little as two
to three hours of exercise a week.

IV. Discourse Structure & Cloze Test : ( 15% )


A.
The way a snake is shaped lets it do special things. ___1___ It can push its way below the
ground. ___2___ Such abilities account for the usefulness of a robot designed like a snake. A
snakebot would be able to do these things, too. ___3___ Wheeled robots easily get stuck or fall
over. A snakebot would not have this problem. ___4___ Since they can carry tools, snakebots
would be able to work in space, as well. ___5___
(A) They could, for example, help repair the International Space Station.
(B) It could go almost anywhere and so would be useful for exploring the
different environments of other planets.
(C) This would make it much more effective than a regular robot with wheels.
(D) A snake can get into very small spaces, like cracks in rocks.
(E) A snake can also climb up different kinds of objects, like high rocks and trees.
B.
Back in England, Colonel Blashford-Snell was offered a 360-kilogram grand piano as a gift
from the Millennium Gloucester Hotel London. ___6___ First, Colonel Blashford-Snell and his
3

team flew the piano as far as they could into the jungle by helicopter. ___7___ With the help of a
few people from the tribe, ___8___ Then they loaded the piano into a dugout and started up the
river. ___9___ With help from more tribe members, ___10___. The piano had arrived at its new
home in the Amazon!
(A) they put the piano in a sledge and pulled the sledge by ropes several miles through the jungle
to a river.
(B) they were able to climb to the top of the mountain carrying the piano.
(C) But getting the piano from the landing site to the village was no easy feat.
(D) Next, he only needed to find a way to get the piano to the village deep in the Amazon jungle.
(E) Finally, they arrived at the base of the mountain where the tribe lives.
C.
___11___ For the statues 500th birthday, the Galleria dellAccademia, the
museum in Italy where David stands today, planned to restore the statues
appearance.
___12___ He was just dirty. And that should not have surprised
anyone because for more
than 300 years the statue stood outdoors in the main
square in Florence, Piazza Signoria.
___13___ Therefore, critics of Davids
restoration claimed that cleaning the statue would
alter its intended appearance
by the artist. Ignoring these cries of protest, the Galleria
dellAccademia went
ahead with the restoration. After nine months of careful cleaning, the
restored David was reveled to the public. And how was the statue different? For one thing,
Davids hair was no longer blackened with grime. ___14___ As one art expert remarked, Only
someone with expert knowledge and long familiarity with the skin of the statue will be aware
that certain unattractive irregularities are no longer there. ___15___
(A) David is a perfect case in point of just such a touch-up project.
(B) Also, the stains on his hands were gone, and the dirty streaks on his left shin
had been erased.
(C) When he carved the sculpture, Michelangelo planned for it to stand outside and
naturally accumulate the marks of weathering.
(D) Even critics of the project agreed that the touch-ups to David were very
moderate.
(E) There was really nothing wrong with David.

V. Guided Translation ( 20% )


1. .
She
1
watches a movie
2
3
moved to
4
2.
Local
7

6
for

to this overwhelming metal construction in their


that it might collapse and crush their homes beneath it.
4

3. , .
10
grateful to their parents and
11
parents' love
They are by
9
for
12
.
4. , ,.
For years, the finished product remained a world-record
13
,
14
15
16
of any other building.
5. ,, .
Industry can also
17
water by using better
19
20
instead of fresh water.
6. Jordan

18

The man made Jordan extremely mad, so he


21
hit him.
7. David

and reusing

22

Not finding any chair in the room, David had to be


23
the entire day.
8.
There is no time to

26

27

and

24

25

about anything. You must make the decision

now.
9.
The students were getting restless and noisy, so the teacher raised her voice to
28
herself
29
.
10.
30
11.

31

32

, thats a tough task.

Heaven helps
33
34
35
12.

36

This time youll have to solve the problem on your own. There is
38
39
40
your parents.

37

VI. (10%)
Starbucks stores serve a variety of freshly brewed coffees, which change on a weekly
___31___ in order to provide customers with an easy way to ___32___ different kinds of coffees
and blends. Most of its revenue, ___33___, does not come from coffee, but from blended products
that ___34___ coffee or other flavors with large amounts of milk, sugar, and granulated ice. As a
general rule, Starbucks does not offer promotional prices on its products, which tend to be higher
than ___35___ of competitors.

Starbucks' whole-bean coffee is roasted in one of their roasting plants which are ___36___
Kent, York, Carson Valley and Amsterdam. These whole beans are ___37___ for purchase at all
Starbucks store locations and in many grocery stores.
Starbucks is also known for the ___38___ slang of its menu, substituting "tall," "grande," and
"venti" for the more traditional "small," "medium," and "large." Additionally, most coffee drinks
can be customized in some way, ___39___, using skim milk instead of cream for a "nonfat" option,
or ___40___ regular and decaffeinated coffee to make a "half-caf.".
(A) for example
(F) available

VII.

(B) those
(G) novel

(C) located in
(H) sample

(D) basis
(E) combine
(I) however (J) mixing

(25%) :

Read the following sentence, and write a short essay of about 100 to 120 words to express
your opinions.

I believe that being able to understand the difficulties and nuances


involved in learning a second language, as well as the underlying
mentality of the students, is what qualifies me to teach English.

97
I. Reading Comprehension:
1. C
2. C
3. D
4.D
II. Test Design:
A.()
B. ()
IV. Discourse Structure & Cloze Test:
A. 1. D
2. E
3.C
4. B
B. 6. D
7. C
8.A
9.E
C. 11.A
12.E
13.C
14.B

5.A
10.B
15.D

V. Guided Translation:
1. never, without, being, tears
2. residents, objected, district, fear
3. no means, take, granted
4. breaker, twice, the, height
5. conserve, plumbing, treated, wastewater
6. lost, control
7. on, his, feet
8. think, twice
9. make, heard
10. To, be, brief
11. those, who, help, themselves
12. no,use, depending, on
VI.
1. D
6. C

2.H
7.F

3.I
8.G

4.E
9.A

5.B
10.J

VII. ()


. Vocabulary and idioms:
(C) 1.Henry led a____ childhood, living on the streets after his parents passed away.
(A)paramount (B)blithe (C)tumultuous (D)tenacious
(A) 2.Having access to little food and water, the refugees are in dire____.
(A)straits (B)flops (C)gape (D)swamp
(D) 3.At the night market, the vendors____ their wares at the top of their lungs.
(A)consume (B)snatch (C)entice (D)peddle
(C) 4.My belief in God has never____, even during the hardest times of my life.
(A)bounced (B)squirmed (C)faltered (D)interfered
(B) 5.The boy______ the fight by pushing his classmate down the stairs.
(A)exchanged (B)instigated (C)snatched (D)escorted
(C) 6.Tom ____ tired to embarrass me in front of the girl I liked, but it didnt work.
(A)potentially (B)accidentally (C)deliberately (D)harmoniously
(C) 7.I know youre on a diet, but a slice of bread is not enough to ____you for a
whole day.
(A)benefit (B)conceal (C)sustain (D)despise
(D) 8.This movie is a successful ____of one of last years best-sellers, which is a
biography of Cleopatra.
(A)collision (B)illustration (C)distraction (D)adaptation
(B) 9.You can read all about it in my ____autobiography.
(A)resounding (B)forthcoming (C)enlightening (D)outstanding
(C) 10.My brother ____his watch trying to fix it, but he couldnt find the problem.
Whats worse, he had trouble reassembling it.
(A)activated (B)assigned (C)dismantled (D)dangled
(A) 11.The High Speed Rail train can reach speeds ____300 miles per hour and
allows people to save a lot of time.
(A)in excess of (B)in proportion to (C)in competition with (D)in relation to
(A) 12.He gets paid ___,so he doesnt have to pay any taxes. Hes afraid of getting
caught, though .
(A)under the table (B)on the spot (C)in the limelight (D)in the black
(B) 13.Jim is tough, and if you take advantage of him, he will surely ___you in one
way or another.
(A)run away with (B)get even with (C)keep pace with (D)make up with
(D) 14.To live up to her mothers expectations, Kelly has made up her mind to win the
dance competition____.
(A)for good (B)by no means (C)by chance (D)at all costs
1

(C) 15.Three days ____the commencement ceremony, Jack finally made up his mind
to tell Carrie he liked her.
(A)aside from (B)above all (C)prior to (D)far from
.Cloze Test:
(Part A)
Keeping ornamental fish as pets is one of the most popular hobbies in the world.
Millions of people keep beautiful fish for their amazing shapes and colors, as well as
for the peaceful aura they bring to an outdoor garden or a living room.
The breeding of fish specially for use as pets has its origins in ancient China. Around
2,000 years ago, farmers would raise silver crucian carp for food in ponds and rice
paddies. The farmers noticed that some carp__1__ bright golden ad orange scales.
They began to selectively breed those mutated carp to enhance the bright colors, and a
hobby was born.
In the 17th century, Chinese goldfish made their way to Japan and Europe, and
the Japanese quickly mastered the art of fish breeding. From the early 19th century,
they started breeding the common carp, creating beautiful nishikigoi, better known as
koi. Europeans,__2__, experimented with keeping other varieties of fish in glass
containers. The hobby became popular among wealthy Europeans in the 1850s after
special equipment was developed. Articles written by fishkeeping enthusiasts
appeared, __3__ an influential article entitled The sea in a Glass.
At first, only the very wealthy__4__ afford equipment, and only local varieties of fish
were easily available. Later, advancements__5__electricity, water filtration systems,
plastic bags, silicone, and air travel greatly reduced the price of equipment and
increased the varieties of fish available. Now, beautiful fish from all corners of the
world can be found in your local pet shop.
(B) 1.(A)disclosed (B)displayed (C)disguised (D)distributed
(D) 2.(A)by the way (B)in the long run (C)as a result (D)on the other hand
(B) 3.(A)regarding (B)including (C)considering (D)providing
(C) 4.(A)might (B)would (C)could (D)ought
(A) 5.(A)such as (B)up to (C)no less than (D)as many as
(Part B)
In the 19th century, the education of the rich and __6__ including a grand tour of
Europes cultural sites. When Thomas Cook organized the first package tour, things
started to change for ordinary people. By 1939, about one million people travel
abroad for holidays each year. However, tourism __7__ until 1970s. Since then, it has
been industrialized: scenery, cultures and religions have become consumer goods and
__8__ in travel booklets. The effects of tourism since the 1960s __9__ amazing. Lots
2

of visitors surged to the worlds most treasured sites, __10__ the Taj Mahal,
Stonehenge, or the national parks of Kenya. Many places that were once too remote to
visit are now part of package tours.
(D) 6.(A)privilege (B)privileges (C)privileging (D) privileged
(C) 7.(A)boomed (B)booms (C)didnt boom (D)has boomed
(A) 8.(A)been displayed (B)be displayed (C)displayed (D)have displayed
(B) 9.(A)was (B)have been (C)is (D)had been
(B) 10.(A)such like (B)such as (C)looks like (D)as like
(Part C)
Teachers hitting students or resorting to corporal punishment are now likely to
get walloped. __11__ teachers who take excessive punishment, the Ministry of
Education announced a new policy clearly defining what is acceptable __12__
disciplining wayward students. The policy is the Observations on the law governing
educators teaching and punishing of students as defined and implemented by
schools draft, a set of rules explicitly defining __13__ a teacher can address
misbehaving students.
National Teachers Association and the Education Ministry formulated the policy
after they had consulted with specialists. Directly or indirectly inflicting physical
punishment on students, threatening, humiliating or intimidating them, are causes
__14__ dismissal, official reprimand, demotion, transferal, suspension, or civil or
criminal proceedings. The policy prohibits teachers from rifling through confiscating
students belongings without reasonable cause, like on suspicion__15__students
possess weapons, drugs, or any contraband.
__16__, the policy defies educators the authority to search a students person.
Verbal reprimands and demands for apology are well within teachers bounds of
authority. Teachers can also __17__ students to hand in a written self-criticism or
reflect silently.
The ministry has distributed the new regulations to all schools and required them
to observe them. __18__, schools in the countryside disagree on the rules, for
punishment there is always harsher. Rural parents require schools to discipline their
children more harshly, __19__urban parents tends to espouse more progressive
attitudes. Being a work in progress, the policy will be __20__ to be revised if its
implementation exposes difficulties.
(D) 11.(A)Feeding up with (B)Not fed up with (C)Having fed up with
(D)Having been fed up with
(A) 12.(A)concerning (B)concerned (C)to be concerned (D)in concerning
(B) 13.(A)what (B)how (C)that (D)if
(B) 14.(A)at (B)of (C)in (D)on
3

(C) 15.(A)if (B)which (C)that (D)what


(A) 16.(A)Besides (B)However (C)Likewise (D)Therefore
(B) 17.(A)amend (B)demand (C)diminish (D)separate
(A) 18.(A)However (B)Furthermore (C)As a result (D)Instead
(D) 19.(A)as (B)for (C)if (D)while
(C) 20.(A)barely (B)finally (C)likely (D)nearly
.Discourse Structure:
(Part A)
Open the pages of any fashion magazine and images of thin models and
underweight actresses can be seen. __1(E)__ While the majority of women outside of
the fashion and entertainment industries are not nearly as thin, this doesnt stop
millions of women from trying to fit this image. This goal can be extremely unhealthy
and even dangerous when it develops into a condition known as anorexia.
Anorexia nervosa, as it is called in medical terms, is a type of eating disorder.
People with the disorder have a distorted vision of their bodies. __2(A)__
Unnecessary dieting is a problem that is common enough, but for some, the desire to
lose weight becomes an obsessive fear of gaining any weigh. In most cases,
individuals with anorexia use voluntary starvation, vomiting, excessive exercise, or
other weight control measures like diet pills to control their weight. __3(D)__
Although anorexia nervosa occurs in a small number of men, ninety-eight
percent of anorexics are women. __4(B)__
While it may seem normal that looks inspired by thin stars are what many young girls
strive for, anorexia has proven to be a serious danger. One of many examples is from
2006, when a 172-centimeter-tall Brazilian model died as a result of the
disorder.__5(C)__ This condition is a sad reflection of cultures in which advertising
and popular entertainment demand that only one body type be attractive and
acceptable.
(A) Sufferers may falsely believe that they are overweight and begin to diet to
change their figures.
(B) Most come from affluent countries, where the condition appears in one in 100
young girls and is more common among wealthier portions of society.
(C) At her death, the 21 year old weighed a shocking 39 kilograms.
(D) In severe cases, anorexics even restrict their food intake to levels that can barely
sustain life.
(E) Tall and skinny, the women on these pages have become representative of beauty
in many parts of the world.

(Part B)
When climbing the worlds tallest peaks, mountaineers must worry about many
dangers, including falls, rockslides, and animal attacks. ___6(D)__This condition can
strike anyone who ventures into high altitudes, and it can be deadly.
High above sea level, the air is thin and the air pressure is low. These effects increase
as elevation increases. Such changes can begin to have a great effect on the human
body above approximately 2,400 meters. __7(E)__ The results are usually a headache,
tiredness, and dizziness. Normally, one or two days at this altitude are enough to
reduce these symptoms and feel comfortable. __8(A)__
However, in some cases, more dangerous forms of altitude sickness may occur
that can seriously damage the lungs or the brain. __9(C)__ There are several ways
mountain climbers can avoid putting themselves too much at risk for serious altitude
sickness. One method is never to climb higher if they are experiencing symptoms of
altitude sickness. Serious mountain climbers should always take their time reaching
the peaks to allow their bodies to adjust. __10(B)__ After doing this a couple of times,
they spend the night at a higher altitude. This encourages the body to produce more
red blood cells, which is part of its adjustment process.
Mountain climbing is thrilling, and it attracts many adventurers. By being aware
of altitude sickness and taking steps to avoid it, anyone can start to explore mountains
with minimal discomfort and worry.
(A)These first days of discomfort pose no long-term threat to health.
(B)A common technique climbers use to go higher in extreme conditions is to climb
up to a higher destination and then come back down.
(C)These conditions can be fatal if they are not treated rapidly.
(D)However, there is one invisible danger in climbing tall peaks that often gets
overlooked: altitude sickness.
(E)At this height, the air is too thin to provide sufficient oxygen for many parts of
the body.
.Reading Comprehension:
(Reading 1)
China is no doubt one the most remarkable places in the world today, particularly
in terms of its architectural marvels. Its buildings, many of them epic in size and
design, range from the ancient to the high-tech and modern. National Geographic
Channels China Special Month is about the stories of the modern-day architectural
projects in China that are producing amazing new buildings for the world to enjoy.
Beijing Water Cube is one of the programs in this series, and it is very much
about the modern as opposed to the ancient. The Beijing Water Cube is a building for
5

the Beijing 2008 Olympics. The building, which will be the National Aquatics Center,
is indeed in the shape of a cube, but its appearance is far from run-of-the-mill. It is
made of a steel honeycomb-like frame covered in a unique skin that is modeled after
soap bubbles. Simply put, the effect is that the Water Cube looks like an enormous
cube-shaped bundle of bubbles.
The bubbles are made of a plastic called ETFE, which is also used to protect
spaceships from cosmic radiation. One of the advantages of ETFE is that it traps solar
energy in the winter and reflects it in the summer, helping to control the buildings
temperature. 3,500 bubbles had to be cut individually and sized in order to create the
honeycomb-like structure. The bubbles are not identical or symmetrical, but seem to
be organized randomly, with different shapes and sizes nestled together. Despite its
random appearance, however, the soap-bubble structure used in the design has a
geometry thats perfect for a high-tech building. Soap bubbles actually always cling
together in regular patterns, and the fragile-looking skin of the buildings bubbles
the plastic covering is only 1/5 of a millimeter thick!is entirely safe.
(C)1.Which of the following is true about the Beijing Water Cube?
(A)It is made with a huge number of steel cubes.
(B)It is meant to show Chinese architecture, old and new.
(C)The plastic is put in place around a steel frame.
(D)It was finished during the last Olympics and will be used in the 2008
Olympics.
(B)2.Regarding ETFE, which of the following can NOT be inferred from the article?
(A)The plastic is extremely thin.
(B)The plastic feels like soap when you touch it.
(C)An astronaut might find ETFE useful.
(D)The plastic helps the building stay cool on hot summer days.
(C)3.According to the article, soap bubbles are a good model for a building
because____.
(A)they are cheap and easy to get
(B)they are transparent
(C)their structure is not as delicate as it seems
(D)they blend in with the environment
(A)4.Which of the following could be another title for the article?
(A)An Amazing New Building for the Olympics
(B)The Discovery of ETFE
(C)Soap Bubbles Can Be Used for Many Things
(D)The Water Cube Replaces All Older Chinese Buildings

(Reading 2)
Throughout the East, convenient and tasty ramen is a popular dish. Even in
Western countries, ramen is commonly sold in stores and stocked in pantries. But
above all, ramen reigns supreme in Japan, which is considered its birthplace.
Ramen is usually used as a very general term to describe any dish based on two
things: soup stock and noodles. The soup stock used for making ramen was not
always a common item in Japan. Prior to the Meiji era, which lasted from 1868 to
1912, the typical Japanese diet consisted mainly of seafood and vegetables. Because
of this, the pork and chicken that were essential for making ramen soup stock were
neither easy to come by nor eagerly accepted. It wasnt until after the introduction of
Western foods, in which meat products played a key role, that ramens popularity
increased.
Even after World War II, it was necessary to dine out to enjoy ramen, whether it
was at one of the many Chinese-influenced restaurants opened by former soldiers, or
at a local noodle vendor. However, a man named Momufuku Ando changed the
history of ramen forever in 1958, when he introduced instant noodles. This new
convenient form of the food allowed ramen to be eaten anywhere by anyone at any
time. Ramen vending machines are now fun novelties adored by tourists and locals
alike.
After ramen became a cultural icon, the Shin-Yokohama Ramen Museum in
Yokohama was opened to honor the delicious dish. Visitors learn about the history of
ramen and can even try their hand at inventing new flavors. It certainly seems that
when it comes to ramen, well never have our fill.
(B)5.Why wasnt ramen greatly popular before the Meiji era?
(A)Japanese women were concerned with dieting.
(B)It didnt match the general eating habits of people at that time.
(C)It was not available in Japan until after World War II.
(D)Westerners had not yet introduced ramen to Japan.
(D)6.Which two major events in the history of ramen are mentioned in the article?
(A)The important of chickens and the arrival of Westerners.
(B)The death of Momofuku Ando and the end of the Meiji era.
(C)World War II and the introduction of vending machines.
(D)The introduction of Western food and instant noodles.
(A)7.Which best defines the term cultural icon in the last paragraph?
(A)An object with special status that has had a strong effect on people.
(B)A historical event that changed a nation forever.
(C)An item that people can not live without in modern times.
(D)The food most often eaten in a country.
7

(Reading 3)
The right genetic mix might lead to a lifetime of happiness, a new British study
suggests. The genes that have to do with ones personality seem to be linked to ones
level of happiness. Researchers for the study found that people who dont worry
excessively, are sociable, and are conscientious tend to be happier, according to the
report in the March issue of Psychological Science.
It turns out that if we want to understand happiness, we will need to understand
personality, said Tim Bates, a researcher for the study. An important implication is
that personality traits of being outgoing, calm, and reliable provide a resource, which
we called affective reserve, that drives future happiness, he said
While happiness has a genetic component, about 50 percent of the difference in
happiness between people results from relationships, physical health, and careers,
Bates noted. However, those lucky enough to have the right gene mix appear to have
an extra supply of happiness that they can use when times are tough, the researchers
reported.
If people want to raise their own levels of well-being, our best advice is that
they practice the kinds of behavior that characterize calm, conscientious extroverts,
Bates said. He recommends that people who want to increase their happiness make
efforts to be social, even if its only with a few people.
One expert commented that the study contributes to the understanding of what
happiness is, because it shows that happiness and personality share some of the same
genes.
(A)8.What has the new study shown?
(A)Happiness may be partly explained by genetic makeup.
(B)Happiness is entirely caused by biological structure.
(C)Our genes make us happy to be around certain personalities.
(D)Happiness has little to do with genetics.
(B)9.Which of the following can be inferred from the article?
(A)People with certain genetic structures are always happier than others.
(B)Certain genetic structures may make it easier to attain happiness.
(C)Being happy can change our genetic makeup.
(D)Genes explain everything we want to know about happiness.
(C)10.What does the researcher mean by affective reserve in the second paragraph?
(A)The ability to change ones personality.
(B)The total group of traits of someones personality.
(C)Personality traits that can help one be happy at certain times.
(D)The amount of happiness in ones life.
8

.Writing: *:*

1.
2.
3.

4.
Write your responses to the editorial and growing educational problems in Taiwan.
Directions:
1.Read the editorial below and express how you feel about the students in Taiwan.
2.If you were the homeroom teacher of this 13-year-old girl student, who suffered a
stroke because she stayed up preparing her schoolwork, tell us how you would
feel about this tragic incident
3.Share with us what you will do with the other students in your class.
4.Read the sentences in bald face and response to them.
Saturday, June 21, 2008

Children in Taiwan pressured too hard


The China Post news staff
The news is unbelievable, but its true: a 13-year-old girl fell to the floor during
class one recent day, and at the hospital was diagnosed as having suffered a stroke.
Doctors who treated her said the child was the youngest stroke patient they had ever
seen.
At first, the doctors suspected her stroke was due to an inherited condition.
They decided later that it had resulted from fatigue. The students teachers said she
was diligent and highly motivated and often stayed up late doing homework.
The doctors said that living a stressful life could lead to many physical
problems and result finally in a serious malady. Unfortunately, many Taiwan
residents live tension-filled lives, including countless young students.
Our education system itself is responsible for this situation. The education
system requires that students at various levels study exceedingly hard day in, day
out. Curricula at almost all schools are designed to prepare students for
entrance exams they must take to advance to the next level of schooling.
As a consequence, most of the children and teenagers are expected to do
their utmost to excel scholastically. At some schools, corporal punishment is
imposed on students who do poorly on tests or exams. At others, students are
placed in different classes according to their abilities and those in the classes for
9

weaker students are usually despised.


Its little wonder then that most young people give almost all their time to
schoolwork. Weak eyesight is a common problem among the islands younger
generation. The number of children who commit suicide for fear of being scolded
or criticized for not working hard enough has been growing from year to year.
Efforts made by the government to reform the education system have not been very
successful.
In 1994, then-President Lee Teng-hui ordered the formation of an educational
reform committee and appointed Nobel Prize winner Lee Yuan-tse as its convener to
overhaul the islands education system. The committee proposed a variety of
schemes to make the system more effective. Among them were the lifting of
restrictions on the establishment of new universities. Another was the decision to
abrogate the Joint University Entrance Examination.
Although well-intentioned, the projects were mostly mapped out without
looking deep into the roots of the islands educational problems. The members of the
committee also lacked a good understanding of the values unique to Taiwanese
society.
Critics say the reforms created more problems than they solved. Today,
most of the islands children, including those in elementary school, still face loads
of homework every day and frequently burn the midnight oil to live up to the
expectations of adults. They have little time for recreation and sports, even on
weekends and during long holidays.
Education officials should show more concern about the physical and mental
well-being of children. It is essential that authorities at each elementary and high
school attach as much emphasis to students state of health as to their grades.
Parents should refrain from pushing their children too hard to get good grades
at school. Having healthy, energetic children is more of a blessing than having whiz
kids with delicate bodies.
Write your responses to the editorial and growing educational problems in
Taiwan.

10

97
Part I:
I. Vocabulary and Idioms :
1. Attractive as the commercials are, I wonder how many celebrities actually
use the products they ______.
(A) smuggle (B) endorse (C) demolish (D) undermine
2. Over fifty percent of college students complain about the lack of
employment prospects. To solve the ________, the government must
revise its policy for higher education.
(A) reconciliation (B) resonance (C) censorship (D) predicament
3. In an astonishing discovery that could rewrite the history of human
_______,scientists say they have the skeleton of a new hobbit-sized human species.
(A) resolution (B) revolution (C) evolution (D) dissolution
4. The locals were told to _________ from the earthquake-stricken village to
escape from aftershocks.
(A) alleviate (B) evacuate (C) deteriorate (D) accelerate
5. We do not favor giving up anything that is _________ by the Constitution.
(A) simulated (B) stimulated (C) quarantined (D) guaranteed
6. The doctor says that if the tumor in the upper arm is malignant and there is no
________, the old lady will die by the end of the year.
(A) amputation (B) deputation (C) computation (D) infatuation
7. _______ fats are found in butter and meat, so too much intake of butter or
meat may lead to heart attack.
(A) Unprecedented(B) Saturated (C) Dissipated (D) Galvanized
8. There is a radical change in Davids personality since he has been addicted to
painkillers. He is becoming aggressive and threatening in his _________.
(A) demeanor (B) hyperventilation (C) sovereignty (D) menagerie
9. A lie detector detects _____ changes in the temperature of a persons skin.
(A) minute (B) monetary (C) dormant (D) discretionary
10. With the price of crude oil and the staples keeping surging steeply up, a
specter of globally economical inflation and ______ is looming ahead.
(A) reservoir (B) conscience (C) alliteration (D) recession
11. Falling in the wake of rising, the share price fell to the ground immediately after
hitting the ceiling, leaving a host of equity investors holding their breath, _____, and
shocked.
(A) paralyzing (B) stagnating (C) deliquescing (D) inveighing

12. Driving while _____ could very possibly pose fatal havoc to the traffic on the road,
and would oftentimes be clamped down by police operations at
any possible time.
(A) incompatible(B) invulnerable (C) intimidated (D) intoxicated
13. Tammy said she would ____ the classified ads every day until she found the job
for which she was qualified.
(A) scrutinize (B) vacillate (C) stipulate (D) apprehend
14. Even the cutting-edge planes still depend on _____ groundbased radar
system, plus thousands of people who watch blips on screen and issue
verbal instructions for takeoffs, landings and course changes.
(A) fraternal (B) elaborate (C) immaculate (D) squeamish
15. Many people consider stability and social reform to be ____ exclusive since one
seems to hold back the advancement of the other.
(A) substantially (B) unanimously (C) extravagantly (D) mutually
16. A new stipulation is promulgated recently that the promotion of all the
personnel serving in various agencies affiliated to the governments is set to
____ their English ability.
(A) appeal to (B) hinge on (C) bend over (D) brace up for
II. Cloze Test:
(A) Men and women are different in many aspects, one of which is
genderlect, the way a person speaks __17__ his or her gender identity. For
example, it is stereotypically accepted that women gossip, often discussing
personal and domestic issues __18__ men communicate at a bare minimum
level only to convey important topics. Deborath Tannen describes the different
conversational styles in her new book, You Just Dont Understand. For men,
conversations are negotiations in which they try to achieve and maintain the upper
hand and protect themselves from others attempt to put them down. They struggle to
preserve independence and avoid failure in this __19__ society. Women, __20__, see
conversations as negotiations for closeness in which they seek confirmation and
support and protect themselves from others attempt to put them away. They struggle
to preserve intimacy and avoid isolation in this network of connections. __21__ is
mens and womens difference in conversational styles that it often leads to
misunderstanding. Understanding the others way of talking is a giant leap
across the communication gap between men and women.
17. (A) based on (B) on account of (C) due to (D) in spite of
18. (A) so that (B) however (C) as well as (D) whereas
19. (A) monarchical (B) hierarchical (C) chronic (D) chronicle

20. (A) for another (B) besides (C) whats more (D) on the other hand
21. (A) What (B) It (C) Such (D) Whatever
(B) Many airlines in the US state that passengers "needing an extra seat to maintain
comfort" must pay for an adjacent seat. Their thought is that since they can't sell that
seat to another person, the passenger occupying part of that seat needs to __22___.
Some overweight members of the community
disagree with that policy, however, suggesting that obesity is a disease and
citing __23__ against the disabled. Whether overweight people should pay
more for seat in a plane will continue to be a hot issue. Proponents say
travelers worldwide are being forced to bear the brunt of increases in not only flight
prices due to the __24__ dollar and depletion of fuel reserves, but they are also being
forced to pay extra charges for checked baggage. It should be the same for severely
overweight and obese people. Not only ___25___ the price of fuel, but it has also had
an environmental impact. An estimated 3.8 million extra tons of carbon dioxide is
released into the air every year. Opponents argue that it is humiliating to ask
passengers to stand on a scale. Does the same standard __26__ people who are taller
than average? How about pregnant women? A person would not like to sit next to an
overweight person because they feel cramped; it is also unpleasant to sit next to
someone with body odor, bad breath, screaming children. Somehow, these people
aren't required to pay extra money, and someone who battles overweight tendencies
should not be required to, either.
22. (A) get the money worth (B) pay its way
(C) make a charge (D) foot the bill
23. (A) suspension (B) reverence (C) discrimination (D) extinction
24. (A) inflated (B) dropping (C) rising (D) higher
25. (A) the extra weight increases (B) to increase
(C) overweight people increase (D) does the extra weight increase
26. (A) succumb to (B) subscribe to (C) stick to (D) apply to
(C) After an epic battle with Senator Hillary Rodham Clinton in a primary
campaign that inspired millions of voters from every corner of America,
Senator Barack Obama __27__ the Democratic presidential nomination on
Tuesday evening.
Despite failure in the 16-month primary campaign, Hillary still __28__ Mr.
Obamas accomplishments. The tedious primary campaign, __29__ tensions over race
and gender, broke records on several fronts: the number of voters who participated,
__30__ and the sheer length of the fight. It also provided unexpected twists to the end

as Mr. Obama __31__ prevailed over Mr. Clinton. __32__, the victory for Mr. Obama,
the son of a black Kenyan father and a white Kansan mother, broke racial barriers and
represented a remarkable rise for an Afro-American.
27.(A) exclaimed (B) acclaimed (C) declaimed (D) claimed
28. (A) paid homage to (B) held hostility to
(C) was in retaliation against (D) sought retribution for
29. (A) confused by (B) infused by
(C) refused by (D) profused by
30. (A) the number of funds raised and spent
(B) the raising and spending amount of money
(C) the amount of money raised and spent
(D) the quality of money raised and spent
31. (A) ultimately (B) riotously (C) nebulously (D) heuristically
32. (A) In the course (B) In a sense
(C) To put it differently (D) Theoretically speaking
(D) More than half of the world population will live in cities by 2008.
Developing countries should not try to put the brakes on urban growth, but
___33__ implement social policies to benefit from the phenomenon, the United
Nations said.
Some 3.3 billion people will live in urban areas by next year. And that figure will
continue to grow rapidly, ___34__ the urban population set to rise to 4.9 billion
inhabitants in 2030, or six out of 10 people among an eight billion-strong population.
Most of the growth will be in developing countries. As a consequence, by 2030,
81 percent of the worlds urban population will be in developing countries, nearly 70
percent of them __35__ in Africa and Asia.
Urban explosion would be all the more difficult to control because poor countries
will __36__ a large part of future urban growth. However, if it is thoughtfully
anticipated, urbanization can play a positive role in economic development. Cities
concentrate poverty but they also represent the best hope of heading off it.
33. (A) instead (B) that (C) for (D) by
34. (A) as (B) with (C) among (D) while
35. (A) live (B) lives (C) will live (D) living
36. (A) map out (B) put forth (C) make up (D) hold back
III. Reading Comprehension :
(A) For millennia people have exploited and often destroy the riches of the
land. Now they covet the wealth of the oceans, which cover nearly three-quarters of

the earth. In the millions of oceans that touch a hundred


nations live four out of five living things on earth. In the seabed, minerals and oil have
been proved to exist in lavish supply. The oceans are a source of
pure water and food protein, of drugs and building materials, and they are even
possibly a human habitat, a key to survival for the doubling population on land. But
the scramble for mineral and oil, for new underwater empires, could heighten
international tensions and set a new and wider stage for world conflicts. People may
yet learn to use a tiny fraction of this wealth. Unless international law soon
determines how it shall be shared, that fraction alone could set off a new age of
colonial war. We must decide how to divide this great wealth equitably among nations.
But wealth is not the only thing at
stake. We must learn how to protect the oceans from the menace of pollution.
In
those pioneer years of the Ocean Age, the damage done sometimes
seems to exceed the benefit gained. Beaches have been soaked in oily slime.
Insecticides, seeping into the oceans, have killed fish and waterfowl. The
future disposal of increasing amounts of atomic waste is an unresolved problem. Part
with negligence and part with lax regulations, we are contaminating our last resource
of life. How to exploit the wealth and these possibilities has become a world problem.
37. The oceans have become peoples coveted wealth because ____________.
(A) there are more resources in the oceans than on land
(B) every nation has equal access to the oceans
(C) the resources on land are being depleted
(D) they can help reduce the population
38. What is the culprit behind the pollution in the oceans?
(A) Peoples careless destructive habits and soft rules.
(B) The underestimated resources in the ocean.
(C) Too much exploitation of the oceans.
(D) The international conflicts over the dominance of the oceans.
39. What can be inferred from this passage?
(A) Three-quarters of petroleum comes from the ocean.
(B) So far people have succeeded in exploring most of the deep oceans.
(C) The exploitation of oceans brings humans more benefits than expected.
(D) The uneven distribution of ocean resources may lead to wars.
(B) The orangutan is literally a man of the woods. A great ape spends most of its
time moving about in trees and less on the ground. They forage for food primarily by
moving about in the canopy---the tree-tops, but males sometimes descend to the
ground to travel longer distances in search of food. Females, however, virtually never

leave the trees. Orangutans lives are nomadic---they travel from place to place
depending on food availability. They have a varied diet of fruit, bark, leaves, and
insects such as ants, termites, and bees. They have a solitary lifestyle and dont live in
group like other primates. The current number of orangutans left in the wild is
estimated at between 20,000 and 27,000. They are endangered species and are the
only big ape found in Asia.
Like other primates, orangutans have complex brains and are highly
intelligent. One orangutan, Nonja, a 19-year-old female inhabitant of the
Schonbrunn Zoo in Vienna, Austria, has gained fame as an accomplished
painter. Nonja has been extremely successful with her paintings for many
years. At a recent exhibition of her paintings, twenty-seven of her
twenty-nine pictures were sold, boosting the zoos funds by 103,000Austrain
schillings. While earlier works were exclusively watercolors, now she even
works on canvas. She uses strong, non-toxic colors. Her favorite colors are
brown and violet. Generally, Nonja sustains her artistic activity for up to an
hour. Recently, an Italian television station invited Nonja to Rome to take
part in a talk show. The zoos management decided to spare the female
orangutan from such a burden and said, Nonja doesnt like stress, and only
paints when it amuses her.
40. How are orangutans different from other primates?
(A) They are on the verge of extinction.
(B) They have larger brains and are smarter.
(C) They are nomadic, moving around all the time.
(D) They are not social animals.
41. Why does Nonja like to paint?
(A) She can make money for her zoo.
(B) Her dream is to have an exhibition at the gallery.
(C) She can save the trouble to find food.
(D) She gets pleasure from painting.
42. What can be inferred from this passage?
(A) Nonja will enjoy the talk show.
(B) Nonja spends most of her time painting.
(C) Wild orangutans seldom settle in one place.
(D) There is adequate food on the trees for wild orangutans.
(C) Cloning is the process of making an organism genetically identical to
another organism through nonsexual means executed in a laboratory. People
have been cloning vegetation for thousands of years, but just recently scientists have

been able to clone plants using a technique called tissue culture propagation.
Cloning didnt get widespread attention until 1997, when a sheep was
successfully cloned at the Roslin Institute in Edinburgh, Scotland. Donned Dolly by
the scientists, the white-faced sheep was cloned and impregnated into a black-faced
ewe who had a normal gestation period. Since the
successful cloning of Dolly, many other science institutions have gone on to
clone other animals such as monkeys, cows, pigs, and mice.
Consequently, there has been much controversy surrounding the issue of
cloning, this controversy sparking fierce debates among politicians, scientists,
theologians and the general public about the moral issue. To have more nutritious and
attractive vegetables, we mass-produce organisms with desired qualities by cloning,
but applying the same principal to humans has given rise to many a proverbial
eyebrow among the public.
Some governments are attempting to enact legislation to ban cloning experiments
outright as some individuals are stating that humans do not hold the divine right to
create. Regardless, cloning is a technological tool that could have definite advantages
if done with responsibility.
43. What does the author think of cloning?
(A) The author fully objects to cloning.
(B) The author fully backs up cloning.
(C) The author thinks society should approach cloning with caution.
(D) The author thinks everything should be cloned.
44. How long have people been cloning for?
(A) Since 1997. (B) Since only very recently.
(C) For a hundred years. (D) For thousands of years.
45. Which of the following statements about cloning is right?
(A) Cloning is the process of giving birth to a sheep in the laboratory.
(B) The world didnt pay much attention to cloning until 1997, when
Dolly was successfully cloned.
(C) Man is the lord of all creatures, and thereby he has the right to create
them.
(D) All walks of life welcome the skill of cloning with open arms since it
mass-produces organisms and supplies adequate organs for people.
(D) Human vision, like that of other primates, has evolved in an arboreal
environment. In the dense, complex world of a tropical forest, it is more
important to see well than to develop an acute sense of smell. In the course of
evolution, members of the primate line have acquired large eyes while the

snout has shrunk to give the eye an unimpeded view. Of mammals, only
humans and some primates enjoy color vision. The red flag is black to the
bull. Horses live in a monochrome world. Light visible to human eyes,
however, occupies only a very narrow band in the whole electromagnetic
spectrum. Ultraviolet rays are invisible to humans, though ants and
honeybees are sensitive to them. Humans have no direct perception of
infrared rays, unlike the rattlesnake, which has receptors tuned into
wavelengths longer than 0.7 micron. The world would look eerily different if
human eyes were sensitive to infrared radiation. Then, instead of the darkness of night,
we would be able to move easily in a strange, shadowy world where objects glowed
with varying degree of intensity. But human eyes excel in other ways. They are, in
fact, remarkably discerning in color graduation. The color sensitivity of normal
human vision is rarely surpassed even by sophisticated technical devices.
46. What does the passage mainly discuss?
(A) Ultraviolet rays. (B) Human vision.
(C) Sight and smell. (D) The environment of primates.
47. The word monochrome is closest in meaning to ________.
(A) monotonous (B) ultraviolet (C) one-dimension (D) one-color
48. Which of the following statements is NOT true?
(A) Human vision is the complex product of a tropical forest.
(B) The ability of humans to distinguish color difference is excellent.
(C) If humans had infrared vision, they could move more easily at night.
(D) Rattlesnakes are capable of detecting wavelengths of light longer than
0.7 micron.

97
Keys: I. 1-5 BDCBD 6-10 ABAAD 11-16 BDABDB
II. (A) ADBDC (B) DCBDD (C) DABCAB (D) ABDC
III. (A) CAD (B) DDC (C) CDB (D) BDA

97

IThe following is a piece of writing from one of the students in the school. If your students are at the
same level as this one, what will you do to teach them writing? First of all, please comment on this
writing sample. You can discuss the language ability of this student as well (5%). Next, based on this
piece of writing, please provide a syllabus of 10 weeks (you can use a list or a table) and further
explain the reasons for designing those objectives/topics/activities in your syllabus (15%).
My Favorite Advertisement
I remember the first time I saw this ad was three years ago. My friend send me an email about this
successful advertisement of ARISTON. I couldnt forget this commercial because its unusual washing
machine that I have never seen. When teacher wanted me to find out my favorite commercial, I
imminently thought of this great commercial.
Without any words we still could understand it wants to tell audience what. Ariston didnt put
emphasis on its capability , instead they create a wonderland just like under the ocean.
By the way, no matter the machine is good or not, it is absolutely my favorite advertisement.
IIPlease read the following article and then write a summary of about 180~220 words (20%). Next,
design a cloze test with 10 multiple-choice items based on your summary (10%).
What makes ordinary people heroes? Can we all aspire to be exemplars of the greatness of human
spirit? Or are some people born to be heroesand others not?
Science provides some clues. Studies now show where empathy and altruism may originate. For
instance, in the relatively new field of mirror neurons, scientists are exploring how infants learn empathy
through mimicking the behaviors of their mothers or caregivers. The neurons fire when the baby observes
in other emotions such as love or distress. The neuron mirrors the emotion or the behavior as though the
observer were performing the action.
Adam Anderson, the research chair in cognitive neuroscience at the University of Toronto, says that
when a person perceives someone else is in trouble, the same part of the brain that processes fear
responses processes sensory information for its emotional significance. It then decides whether to relay
this danger signal to the motor cortex. The critical question is, Why do heroes activate their motor
cortex to act while others stand by and watch? asks Anderson.
How a human is nurtured may be the answer to that question, says Samuel Oliner, author of Do Unto
Others: Extraordinary Acts of Ordinary People and Altruistic Personality: Rescuers of Jews in Nazi
Europe. Oliner has conducted studies seeking to understand why some September 11 firefighters and
1

some Germans during the Second World War risked their lives to help othersin the case of the latter, to
take in Jewish children, including Oliner himself, during the Holocaust.
In interviews with rescuers and bystanders, Oliner found that the rescuers tended to come from
nurturing homes where there were outward expressions of love and commitment to the family. Also,
respect for other people, including those from other cultural backgrounds, was reinforced either through
religious beliefs or a universal sense of morality.
Many rescuers we talked to said simply, It was the right thing to do, says Oliner.
While almost everyone is born with the capacity for empathy, says Stephen G. Post, author of Why
Good Things Happen to Good People, studies have shown that when parental attachment during
childhood is inconsistent or absent because of abuse or neglect, developing compassion and empathy
towards others is more difficult.
III 2
(1) Vocabulary Test
1. A tsunami is a natural phenomenon consisting of a series of waves generated when water in a lake or
the sea is rapidly displaced on a massive scale.
Ais made up of Bcontaining Cincluding
Ddeveloping from
2. Millions mourned the death of Pope John Paul, warmly recalling him on Sunday as an emissary of
peace and hope.
Asymbol Brepresentative Cpriest
Dmartyr
3. The novelists personal letters were published posthumously.
Afollowing death Bduring lifetime Cbefore death
4. The proposition met with strenuous objection.
Aindolent Bvigorous
Cbland

Dafter birth

Dfutile

5. The contract was annulled because one of the signers was too young.
Arenewed
Benacted
Ccancelled
Dvalidated
as a faithful and kind friend.
6. He will be remembered by many with
Aconnection
Baffection Csusceptibility Dextinction
7. As he has
Atorn

our patience, well not wait for him any longer.


Bwasted
Cexhausted

8. The parents are to


Acoordinate

the students on a tour of the campus.


Bcooperate
Cassociate

Dconsumed
Daccompany

9. The dentist charges half price for treatment and then ______ the cost from the government.
Aregains
Bretrieves
Crecoups
Drecovers
10. My grandmother walked long distances to the nearest store, ______ back home with her bags of
groceries.
Atrudging
Bsauntering
Ccreeping Dstomping
11. Michelles aunt was very ______ and would be shocked and feel offended if you mentioned
sex-related matters to her face.
Aantiquated
Bstraitlaced
Cquaint
Dold-time
2

12. The fire hydrant blew a ______ of water into the air as soon as a car crashed into it.
Adribble
Bgust
Cshaft
Djet
13. Do you believe that he is the author of a historical romance that has sold ______ the million
Aby
Baround
Cup to
Dfor
(2) Cloze Test
Why is the world producing so much garbage? Garbage is an unavoidable result of consumption. And to
__14__ the high standard of living that has been reached in many industrialized countries, we must continue to
consume, as the consumption keeps the wheel of industry turning. We use the rapidly disappearing natural
resources of the earth to feed the machines of industry, which __15__ produce endless quantities of consumer
goods. What we havent been able to do, however, is to find a successful way to __16__ all the by-products of
industry and the products themselves when they are used up, worn out, or __17__ . One of the biggest causes of the
garbage problem is the worlds exploding population. More and more people means more and more waste. Before
the industrial revolution, when the world population was very small, the problem of garbage did not exist.
Whatever waste was produced then could be absorbed by the earth without __18__ natures balance, but nature can
no longer cope with the amount of garbage that is currently being produced.
14. AcontainB entertainCmaintainDordain
15. Aby contrastBin turn Cmore or lessDto the contrary
16. Atake offBconsist of Cdispose ofDput off
17. AoutdatedBoutfitted CoutreachedDoutnumbered
18. AcorruptingBdisrupting CretainingDsustaining

(3) Sentence Choice


Researchers have found a clear relationship between the number of ideas produced and the quality of the ideas.
__19__ There are two reasons for this. The first is a matter of simple probability. Creative ideas are statistically
uncommon. As Alfred North Whitehead explains, The probability is that nine hundred and ninety-nine of our
ideas will come to nothing, either because they are worthless in themselves or because we shall not know how to
elicit their value. __20__ The thousandth idea may be the one that will change the world. The second reason is
that initial ideas are usually poorer in quality than later ideas. Just as water must run from a faucet for a while to be
clear and free of particles, so thought must flow before it becomes creative. Early ideas are not usually true ideas.
__21__ One very plausible hypothesis is that familiar and safe responses lie closest to the surface of our
consciousness and therefore are naturally thought of first. __22__
19.AIn any case, success in creative thinking depends on continuing the flow of ideas long enough to purge the
common, habitual ones and produce the unusual and imaginative.
BNot infrequently, the idea that provides a key to resolving an issue will seem, at first consideration,
irrelevant.
CThe more ideas produced, the better the chances of having one or more good ones.
DExactly why this is so is not known.
20.ABut we had better entertain them all, however skeptically.
BThe more ideas produced, the better the chances of having one or more good one
3

CNot infrequently, the idea that provides a key to resolving an issue will seem, at first consideration,
irrelevant.
DIn any case, success in creative thinking depends on continuing the flow of ideas long enough to purge the
common, habitual ones and produce the unusual and imaginative.
21.AIn any case, success in creative thinking depends on continuing the flow of ideas long enough to purge the
common, habitual ones and produce the unusual and imaginative.
BProducing a large quantity of ideas is one of the ways to produce creative solutions.
CBut we had better entertain them all, however skeptically.
D Exactly why this is so is not known.
22.AThe more ideas produced, the better the chances of having one or more good ones.
BBut we had better entertain them all, however skeptically.
CNot infrequently, the idea that provides a key to resolving an issue will seem, at first consideration,
irrelevant.
DIn any case, success in creative thinking depends on continuing the flow of ideas long enough to purge the
common, habitual ones and produce the unusual and imaginative.
(4) Reading Test

THE REAL THING?


One of the most successful commercial products ever launched is said to have come about as the
result of a mistake. In 1896, Jacob's Pharmacy in Atlanta, Georgia, was selling a nerve tonic known as
'French Wine Cola - Ideal Nerve Tonic'. By accidentally adding fizzy water instead of still water to the
recipe, a pharmacist called John S. Pemberton invented what has today become the most popular soft
drink in the world: Coca-Cola. Along with its closest rival - Pepsi - which appeared on the market three
years later, Coke has enjoyed phenomenal success worldwide, particularly in the past fifty years. Indeed,
old Coke bottles and 'limited edition' cans can often fetch considerable sums from collectors, and there
are even stores which deal exclusively in Coke products and memorabilia.
What could possibly account for the amazing success of Coca-Cola? How has this combination of
carbonated water, sugar, acid and flavourings come to symbolize the American way of life for most of the
world? After all, even the manufacturers could hardly describe Coke as a healthy product since it contains
relatively high amounts of sugar (admittedly not the case with Diet Coke which contains artificial
sweeteners instead of sugar) and phosphoric acid, both of which are known to damage teeth.
One explanation may be found in the name. The original recipe included a flavouring from the coca
plant and probably included small amounts of cocaine (an addictive substance), but since the early part of
this century all traces of cocaine have been removed. However, Coke (like all cola drinks) also includes a
flavouring from the cola tree; cola extract contains caffeine, which is a stimulant, and the Coca-Cola
company adds extra caffeine for good measure. While caffeine is not thought to be an addictive
substance in itself, there is considerable evidence that over a period of time the consumption of caffeine
has to be increased in order for its stimulating effect to be maintained, and so sales of Coke perhaps
benefit as a result.
4

A more likely reason for the enduring popularity of Coke may, however, be found in the company's
enviable marketing strategies. Over the years it has come up with some of the most memorable
commercials, tunes, slogans and sponsorship in the world of advertising, variously emphasizing
international harmony, youthfulness and a carefree lifestyle. Few other companies (arguably including
Pepsi) have been able to match such marketing ploys so consistently or effectively. As suggested earlier,
the influences of American culture are evident just about everywhere, and Coca-Cola has somehow come
to represent a vision of the United States that much of the rest of the world dreams about and aspires to.
Perhaps drinking Coke brings people that little bit closer to the dream.
23. In paragraph 1, 'cans can often fetch considerable sums' means the same as:
(A) Coke is quite expensive in some parts of the world
(B) collectors consider carefully how much they are paying for a can of Coke
(C) some collectors will only drink Coke in exclusive stores
(D) certain Coke cans are worth a lot of money as collectable items
24. In the second sentence of paragraph 2, the writer seems surprised that:
(A) something so normal could have such importance
(B) all the ingredients can be found in most countries
(C) most of the ingredients are not very healthy
(D) Coke must be so easy to produce
25. The writer uses 'for good measure' in paragraph 3 to emphasize the fact that:
(A) there is a lot of caffeine in Coke
(B) the amount of caffeine in Coke is carefully measured
(C) the extra caffeine improves the taste of Coke
(D) the extra caffeine balances the amount found naturally in the cola extract


I. Choose the best possible answer.
(B) 1. Id rather that you ____ to the summer camp.
A. join
B. go
C. going
(A) 2. Curiosity got the ____ of meI had to find out what it was.
A. better B. good C. best
(B) 3. I cant finish my meal. Can I get a ____ for this
A. sack
B. doggy bag
C. kitty bag
(C) 4. Its been a long time. Lets ____.
A. hang out sometimes
B. meet up sometimes
(C) 5. A meal NO.2. OK, for here or ____

C. hang out sometime

A. going
B. take to go
C. to go
(B) 6 Oh, noMy computer ____ again
A. turned off
B. crashed
C. hacked
(C) 7. I suggest that he ____ more credits this semester.
A. takes B. took C. take
(A) 8. Dont stopYoud better ____.
A. keep working on it B. keep working for it C. keeping on
(B) 9. They saw a burglar ____ into the house last night.
A. broke
B. break
C. broken
(B) 10. She avoids ____ him all the time.
A. see
B. seeing
C. to see
II. Choose the answer that best interprets the underlined words.
(C) 11. Get out of hereI dont buy that.
A. I dont like you
B. I dont like you to be around.
C. You must be kidding.
(B) 12. Im not pulling your leg.
A. helping you
B. making fun of you
C. curing your leg
(A) 13.He got cold feet on that matter.
A. intimidated
B. encouraged
C. interested

(B) 14. We see each other once in a blue moon.


A. often
B. rarely
C. all the time
(B) 15. Sorry for the hitch. It worked out well, though.
A. misunderstanding
B problem
C. injury
(A) 16.Im not asking you. Im telling you
A. commanding
B. sharing a secret with
C. talking about something with
(A) 17. I believe you are right, but thats a different story.
A. thing of different nature
B. a different opinion
C. a different news
(A) 18. He always bad-mouths his neighbors.
A. uses profanity on someone
B. talks bad behind others
C. talks someone into not doing something
(B) 19. Its your call now.
A. the right person
B. a decision
C. calling for a meeting
(A) 20. Someone has to do this, and he is the one.
A. the right person
B. the person who is to blame
C. the only person who can do it

III. Choose the best answer to complete the sentences.


(C) 21. HeyBe carefulYou ____ me.
A. kill
B. killed
C. could have killed


(C) 22. Sorry, I was not home when you came by. You ____.
A. would have called
B. could have called
C. should have called
(C) 23. Being a teacher, ____.
A. it is hard to conduct the class well
B. who knows what the students will do to you
C. you have to be prepared for the problems ahead of you
(C) 24. ____, you are obliged to carry it out.
A. Assigning to the project
B. Having assigned to the project
C. Being assigned to the project
(A) 25. The textbook ____ the price is the lowest will be used in class next semester.
A. of witch
B. of that
C. whose
(A) 26. Blessed are those ____ show kindness to others.
A. who
B. who that
C. that who
(C) 27. She ____ her parents before she reported to the police about the accident.
A called
B. has called
C. had called
(C) 28. I brought little something for you, Barbara. Oh, you ____.
A. dont have to
B. must not
C. shouldnt have
(C) 29. Here is a token for you to remember me ____.
A. of
B. upon
C. by
(A) 30. To boost the economy, we are fighting a war ____ all fronts.
A. on
B. with
C. against

97
I.

Vocabulary &Phrases: choose the best answer for each question


D1. Do you need a map to
your way through the city
(A) desolate (B) fluster (C) prohibit (D) navigate
A2. In the
at this company, some department managers are more
senior than others.
(A) hierarchy (B)slogan (C) recess (D) league
B3. Thomas doesn't believe in ghosts. He wants to see

_proof that they

exist.
(A) intimate (B) tangible (C) frugal (D)voluntary
him for eating all the cookies she had just baked.
C4. Phil's mother
(A) exterminated (B) erupted (C) admonished (D) defended
A5. After losing every game today, Mark finally
that he was a
terrible tennis player.
(A) conceded (B) deferred (C) col1ided (D) avenged
B6. Laura ______ as the winner of her states beauty contest.
(A) arrested (B) emerged (C) propelled (D) scratched
C7. This weekend, I am ______all the junk in my garage and throwing it away.
(A) sheltering (B) persuading (C) tackling (D) inhaling
A8. Since fossil fuels pollute, scientists are developing ________that are
environmentally friendly.
(A) substitutes (B) illusions (C) flirts (D) defects
D9. The ________ plan to build a power plant has caused some local residents to
protest.
(A) synthetic (B) perceptive (C) indifferent (D) controversial
D10. Wearing seat belts in cars is ________ in some countries.
(A) awkward (B) abstract (C) candid (D) compulsory
C11. The Smiths ________ a private plane to fly them to their honeymoon
destination.
(A) transcended (B) sustained (C) charted (D)revoked
A12. After the earthquake, we could feel ________ from aftershocks hundreds of
miles away.
(A) vibrations (B) fibers (C)domains (D) challenges
B13. Jim is tough, and if you take advantage of him, he will surely ________ you
in one way or another.
(A) run away with (B) get even with (C) keep pace with (D) make up with
A14. ________ the journalists question, the actress claimed she didnt have

anything to say about her personal life.


(A) In reply to (B) In pursuit of (C) On the verge of (D) In proportion to
C15. The High Speed Rail Train can reach speeds ________ 300 miles per hour
and allows people to save a lot of time.
(A) in respect of (B) in terms of (C) in excess of (D) in competition with
II. Grammar: choose the best answer for each question
C1. The nurse approached the child, ________ to make him relaxed.
(A) and trying (B) and tries (C) trying (D) tried
D2. By the time we meet again, summer________.
(A) has arrived (B) had arrived (C) is arriving (D) will have arrived
D3. Since then Australia ________ from the rest of the world by vast oceans.
(A) is isolated (B) had isolated (C) has isolated (D) has been isolated
C4. If I had know the fact, I ________ action right away.
(A) would take (B) have taken (C) would have taken (D) had taken
A5. To be creative, you must dare ________, and courageously express your own
outlook.
(A) to rebel (B) rebel (C) be rebelled (D) rebelling
B6. There are programs on television that ________ how to do things.
(A) explains (B) explain (C) explaining (D) explained
A7. England was probably the first country in Europe ________ ordinary people
used umbrellas against the rain.
(A) where (B) when (C) which (D) what
B8. ________ men and women are able to respect and accept their differences,
love has a chance to blossom.
(A) As soon as (B) As long as (C) As well as (D) As far as
D9. ________carefully, this letter is very beautiful.
(A) Type (B) Typing (C) To type (D) Typed
A10. The government recommends that giving out plastic bags at convenience
stores________.
(A) be quitted (B) quit (C) has been quitted (D) is quitting
III. Cloze Test
Some words in the following passage are deleted. Each deleted part is numbered.
Please choose the best answer for each missing part from the word list given below.
Write down only the Alphabetic Letter representing the word for each answer.
(A) after all (B) In spite of (C) essential (D) above all (E) irrelevant (F) based on
(G) that
(H) which (I) owing to (J) regarded as
Throughout the East, convenient and tasty ramen is a popular dish. Even in Western
countries, ramen is commonly sold in stores and stocked in pantries. But

1.D ,

ramen reigns supreme in Japan, which is considered its birthplace.


Ramen is usually used as very general term to describe any dish
2.F
two
things: soup stock and noodles. The soup stock used for making ramen was not
always a common item in Japan. Prior to the Meiji era, which lasted from 1868 to
1912, the typical Japanese diet consisted mainly of seafood and vegetables. 3.I
this, the pork and chicken that were
4. C for making ramen soup stock were
neither were neither easy to come by nor eagerly accepted. It wasnt until after the
introduction of

Western foods, in which meat products played a key role,

5. G

the popularity of ramen increased.


IV. Discourse Structure
Some sentences in the following passage are missing. Please choose the best
answer for the missing part from the sentences given below. Write down only the
Alphabetic Letter representing the sentence for each answer.
Having a baby can inspire new parents to try the latest method for ensuring their
childs health.
1. E
Cord blood is collected immediately after a baby is born. The blood is valuable
because it is rich in stem cells that can be used in treating diseases like childhood
cancers and immune system disorders.
2.A
Stem cells, on the other hand,
can easily turn into useful red blood cells, white blood cells, and platelets, which
restore the immune system and help patients become healthy again.
3 D. The
potential for future uses of stem cells, such as the creation of entire organs, is another
reason people are considering storing their babys cord blood.
4. B To begin with, storing cord blood is quite expensive. Second, the effort
might be wasted since it's highly unlikely hat a stem cell transplant will ever be
needed. And, third, if such a transplant were necessary, its doubtful that the childs
own cord blood would be better than that from another donor.
5. C For these
reasons, doctors discourage private cord blood storage, but support the donation of
cord blood to public blood banks. If a transplant were necessary, a likely match could
be found in public bank.
Storing a childs cord blood could possibly save his or her life someday. However,
it could also turn out to be a waste of resources and money. Despite the drawbacks,
many parents are choosing to spend money on cord blood storage, just in case.
(A) Treatment of such diseases normally involves radiation that can be harmful to the
patient.
(B) However, there are drawbacks.
(C) Furthermore, even if a childs disease can be treated with cord blood, it may be

likely that the childs own cord blood stem cells carry the same defect that caused the
disease.
(D) Although stem cells can also be taken from other sources, cord blood stem cells
are especially valuable because they are easily accepted into a patients body while
other types of stem cells are often rejected.
(E) Recently, that method has been storing the blood from newborn babies umbilical
cords.
V. Reading Comprehension
By definition, the documentary is dramatic, but non-fictional presentation of
information that is, for the most part, not staged specially for the cameras benefit. In
fact, the documentary makers mission is not to change reality, but to embrace the
substance of it and to convey that with passion and impact. This is not to say there is
no room for artistic license in the making of a documentary. Documentary filmmakers
employ a wide range of cinematographic and dramatic techniques to make their
subjects more palatable for their audience. These might include reenactments of
events by actors, or by people who were actually involved in the event themselves.
The subject matter could be entertaining in nature, such as travel programs, or purely
informative, such as a study of alcoholism, poverty, or racial prejudice.
Non-fiction cinema existed long before the documentary from was first invented
and given a name in the 1920s. Before the advent of the television journalism, it was
used to report the news. The term documentary, derived from the French term
documentaries, is said to have been first coined by English filmmaker John Grierson
in 1922, while he was reviewing Robert Flahertys Nanook of the North, a story of
Eskimo survival in the Far North. This film was a surprising success. It drew large
crowds even though its distributors had first believed the public would not be
interested in a factual film. Eskimo life was portrayed in a way that went beyond the
fragmented presentation of previous efforts.
As documentaries became more popular, filmmakers saw in them a way to present
important social issues to their audiences. The1930s saw the debut of political
documentaries. The Plow That Broke the Plains (1936) depicted a contemporary crisis
in American farming brought about by soil erosion. Then came World War II, and the
demand for documentaries soared as the people thirsted for news about the war.
Documentaries such as the Why We Fight series encourage men to enlist and
motivated them to fight for their country. With the end of the war camp a sharp
decline in the number of documentaries made, until the changing social climate and
the growth of television during the 1950s and 60s created a need for documentaries
with social content. One such example was Harvest of Shame (1959), which showed
the plight of migrant worker in America and became a model for the work of future

documentary filmmakers.
Generally speaking, documentaries do not attract the huge audiences for whom
mainstream movies have such appeal. For that reason, they are often dependent t on
the approval and good will of television networks for exposure. To viewers
accustomed to fast-paced fantasy liberally sprinkled with high-tech special effects,
they quite often seem slow, make heavier demands upon ones concentration, and are
not thought of as being entertaining. Documentaries remain a vitally important source
of factual information. They can convey this information and inform the public in
ways that the print media cannot match.
B1.According to the first paragraph of the passage, documentaries_________.
(A) do not portray events in a factual way.
(B) present events in a way that will impress the viewer.
(C) are part fact and part fiction.
(D) are primarily intended to entertain.
A2.Which of the following statements is NOT supported by the essay?
(A) The documentary film had its origin in movie fiction.
(B) Early documentaries were a popular form of visual entertainment.
(C) Documentaries today depend on television companies for access to the
public.
(D) The earliest American documentaries were news programs.
C3.What is the main purpose of the essay?
(A) To show the importance of the documentary in film history.
(B) To relate the difficulty of making documentaries.
(C) To discuss the historical development of the documentary.
(D) To show the documentarys role in influencing social opinions.
C4.The word plight in paragraph 3 of the passage is closest in meaning to
_______.
(A) corruption
(B) engagement
(C) predicament
(D) purpose
A5.The passage discusses all of the following reasons for the limited success of
the documentary EXCEPT _________.
(A) the difficulty of dealing with a complex subject in a short film.
(B) competition from more exciting poplar films.
(C) the desire of the public for more relaxing and entertaining films.
(D) the television networks lack of enthusiasm for non-fiction programming.

97
______

_______

________

( 7 75 1-50 1 1 1/3 51-60


1 2 1/4 61-75 1 2
1/3 )
()
1. Marsha wants to look slim. However, every time she goes home, her mom
will _____ tell her that shes too thin and give her more food to eat.
(A) extensively (B) subconsciously (C) eminently (D) invariably
2. In _____ of her taking drugs, the actress said before the press that she had
suffered from great pressure of gaining fame.
(A) reverence (B) dominance (C) equivalent (D) justification
3. With the spiraling _____ in house prices, few young couples can afford to
buy homes.
(A) surge (B) roar (C) plea (D) lodge
4. Winning at the Olympics and meeting the president were the most important
____ in the tennis players life.
(A) destinies (B) episodes (C) analyses (D) fantasies
5. Mr. Hanks decision was based upon nothing more _____ than his prejudice
against women.
(A) substantial (B) harmonious (C) illiterate (D) identical
6. Next month we are going to _____ new editing procedures so that we can
bring out every issue more efficiently.
(A) rotate (B) precede (C) integrate (D) upgrade
7. The writer made a(n) _____ appeal for human rights in her book, but
unfortunately it was never published in Chinese.
(A) formidable (B) decent (C) amiable (D) eloquent

8. The love affair between the Italy tycoon and the actress from Brazil caused a
_____ storm of press coverage.
(A) comparable (B) sensational (C) sentimental (D) confidential
9. The young scientist showed great _____, saying that his success was as
much the result of good luck as his own talent.
(A) dignity (B) intensity (C) humidity (D) modesty
10. The problem with Jane Smith is that she tends to take criticism too _____
and gets angry very easily.
(A) personally (B) eventually (C) intimately (D) certainly
11. The old castle was creepy but _____. We were curious and wanted to go
inside.
(A) intangible (B) insightful (C) intriguing (D) ambiguous
12. The radical reforms of the national health system will be _____ in one year,
which will benefit more than 200,000 low-income people.
(A) implemented (B) illuminated (C) implicated (D) intervened
13. The much-reported fairytale romance between the actor and the actress is
just a(n) _____ stunt to promote their upcoming movie.
(A) exploitation (B) publicity (C) solidarity (D) assumption
14. My friend Jason is a mountain climber, so the books he buys are _____
about mountain climbing.
(A) predominantly
(B) sarcastically
(C) simultaneously
(D) tentatively
15. Suharto, the controversial former Indonesian president, was _____ due to
failure of several internal organs this January, and he later passed away.
(A) undermined (B) stimulated (B) massacred (D) hospitalized
16. The _____ of that Hong Kong entertainers obscene photos on the Internet is
illegal and should be banned.
(A) currency (B) courtesy (C) concession (D) circulation
17. Many sophomores in that college _____ to the literary periodical as
complementary reading material.
(A) subscribe (B) describe (C) prescribe (D) inscribe
18. With his excellent social skills, Steven has been _____ as a great
communicator by all his colleagues.
1

(A) diagnosed (B) exploited (C) perceived (D) concerned


19. The telephone has changed beyond _____ in recent years. In both form and
function, it has become totally different from what it was before.
(A) recognition (B) possession (C) prevention (D) appreciation
20. Kevin _____ together all of his savings to buy an engagement ring before he
proposed to Nina.
(A) snarled (B) scraped (C) preyed (D) perched
()
As an international filmmaker and director, Steven Spielberg was no A
student when he was young. Rather than read, the kid really preferred shooting
homemade movies. In his sophomore year of high school, he __21__. But when
he returned, he was mistakenly placed in a learning0disabled class. Only when
the family moved to another town __22__ in a more suitable high school,
where he eventually graduated.
After being denied __23__ into a traditional filmmaking school, Steven
Spielberg enrolled in English at California State College at Long Beach. Then
in 1965, his life took a complete turn. __24__ Universal Studio, he met Chuck
Silvers, an executive in the editorial department, who brought Steven into a
brand new universe.
Ten years later, 28-year-old Spielberg directed Jaws, which __25__ $470
million. Dozens of films and awards have followed, with which this no A
student in school has proved himself remarkable in the movie industry.
21. (A) showed up (B) dropped out (C) ended up (D) passed out
22. (A) was he put (B) did he put (C) he was put (D) he put
23. (A) application (B) permission (C) entrance (D) approval
24. (A) To visit (B) Visiting (C) Visit (D) Visited
25. (A) called in (B) gave in (C) turned in (D) took in
Paul Klee is one of the most original artists of the 20th century. A
Swiss-born painter and graphic artist, whose humorous works are rich in
allusions to dreams, music, and poetry, Paul Klee (1879-1940) __26__ difficult
to classify. He follows no movement, and there is no single term that describes

his style. Primitive art, surrealism, cubism, and childrens art all seem blended
into his small-scale, delicate paintings, watercolors, and drawings.
Klees early works are, __27__, etchings and pen-and-ink drawings. These
combine satirical, grotesque, and surreal elements and reveal the influence of
Francisco de Goya and James Ensor, __28__ Klee admired. A short trip to
Tunisia in 1914 proved to be a __29__ in Klees art. Up to this time, he had
worked mainly in black and white. In Tunisia, Klee discovered color. The
brilliant red sun there seemed to make all colors brighter and deeper. He was
__30__ overwhelmed by the intense light there that he wrote, Color has taken
possession of me; no longer do I have to chase after it; I know that it has hold
of me forever. Color then became the basis of his art.
26. (A) being (B) is (C) is being (D) to be
27. (A) at most (B) most importantly
(C) for the most part (D) most of all
28. (A) neither of whom (B) neither of them
(C) both of whom
(D) both of them
29. (A) fleeting moment (B) grace period
(D) quality time
(D) turning point
30. (A) even (B) so (C) too (D) very
A microbiologist, a doctor, an actor, and a computer engineer what do
these people have __31__? How about a passion for music, a love of
performing and a tendency to take home top awards? These people from
__32__ backgrounds are some of the members of Taiwanese vocal group
Semiscon, making their debut performance in North America at the Taiwanese
Cultural Festival.
Semiscons ten members officially formed the group in 2002, and have
been singing a cappella relying mainly __33__ the human voice for
accompaniment. Just one short year after its birth, Semiscon won the hearts of
judges and the audience at the 3rd Annual Taiwanese A Cappella Competition
with its eclectic mix of jazz, musical theater and Asian and Western pop music,
__34__ the top prize in addition to awards such as Best On-Stage Presentation
Award.
2

In July 2004, Semiscon traveled to Graz, Austria to attend as spectators,


the ten vocalists were thrilled when the group __35__ taking home the most
prestigious and coveted Ward Swingle Award.
31. (A) in vain (B) in common (C) in store (D) in return
32. (A) abundant (B) extraordinary (C) unique (D) various
33. (A) in (B) at
(C) on (D) with
34. (A) to receive (B) receiving (C) received (D) having receiving
35. (A) ended up
(B) turned out
(C) made it into (D) had a hard time
When it comes to biting humans, it is the female mosquitoes who do the
dirty work. There are some 2,500 varieties of these mini-vampires. Only a few
__36__ human blood. Still, a human often comes in handy in egg __37__. If
you are scratching, itching and a little too red, just remember that youre
helping someones mother.
The mosquito has made its presence felt. However, __38__ 1890 that
scientists suspected the bugs role in the spread of many diseases. Through
malaria, yellow fever, and other insect-borne diseases, our little winged foes
have claimed the lives of various popes and world leaders.
Think of the mosquito as a protector of the weak. When spears and guns
wont stop a would-be band of plunderers, mosquito-borne illnesses will often
do the trick. In Africa and Asia, youll find that explorers contracted malaria
and other diseases that the locals were __39__, says Professor Andrew
Spielman, one of the worlds foremost experts on mosquitoes and the diseases
they carry. Besides, many historians believe Africans were able to rebel on the
Amistad slave ship because the crew __40__ by yellow fever.
36. (A) lay off (B) cal for (C) feast on (D) come by
37. (A) production (B) replication (C) contention (D) innovation
38. (A) not until (B) it was until (C) until (D) it was not until
39. (A) alive with (B) immune to (C) engaged in (D) hooked on
40. (A) struck
(B) are stricken
(C) had stricken (D) had been stricken

To all who say that circumstances in their lives have made personal success
impossible, Dr. Carson, director of pediatric neurosurgery at the Johns Hopkins
Medical Institutions, __41__ proof that perseverance, prayer, and devotion to
acquiring knowledge can overcome any obstacles in ones way.
Coming from a broken home, Ben Carson was just another kid trying to
survive. His possibility for a decent future didnt look good. __42__, through
his mothers guidance and his own hard work, he went from the bottom of his
class to the top in one year and became a real and smart reader, quenching his
__43__ for knowledge. He graduated with honors.
Dr. Carson tells his audiences that the keys to a life of accomplishment
__44__ ones ability to discover his or her potential for excellence, the
acquisition of knowledge to develop it, and a willingness to help others.
Education, he says, is liberation. He introduces young people to the abundance
and lifestyles that exist in __45__, far beyond the narrow worlds of sports and
entertainment, so mistakenly glorified in todays celebrity culture of TV and
movies. He calls upon us all to strive for excellence and recognize our innate
abilities.
41. (A) living (B) live (C) to live (D) alive
42. (A) That is (B) Accordingly (C) Nonetheless (D) In contrast
43. (A) seek (B) thirst (C) starvation (D) acquisition
44. (A) lay down (B) lie in (C) inform of (D) persist in
45. (A) literary circles
(B) spiritual freedom
(C) dream weaving (D) intellectual pursuit
In order to stop teenagers from smoking too early, the British government
decides to raise the legal minimum age __46__ which tobacco can be bought in
English and Wales from 16 to 18 years from October. The move, announced by
Public Health Minister Caroline Flint, will follow the introduction of a ban on
smoking in public and work places later this year. The ban __47__ in Wales in
April and England three months later. According to the statistics, about 9% of
11 to 15-year-olds smoke and the minister hopes the move will reduce this
figure. However, experts warned other measures were also needs.
The government argues that raising the legal age to 18 will make it easier for
3

retailers to __48__ under-age smokers. Minister Flint also believes that


bringing the legal age for the purchase of tobacco into line with __49__ of
alcohol will reinforce the dangers of smoking to young people.
Ms Flint said, Smoking is dangerous at any age, but the earlier young
people start, the more likely they are to become life-long smokers and to die
early. Someone who starts smoking aged 15 is __50__ doe of cancer than
someone who starts in their late 20s.
46. (A) at (B) in (C) with (D) by
47. (A) brings into effect (B) comes into effect
(C) knows its place
(D) takes its place
48. (A) locate (B) spot (C) neglect (D) resemble
49. (A) this (B) that (C) it (D) those
50. (A) three times likely to
(B) three times more likely to
(C) more likely three times to (D) likely three times to
()
After college, many students head off to graduate school or enter the
workforce. For those that go on to get their Masters degree, this isnt
necessarily the end of the educational road. __51__ Getting a PhD is a long
process that doesnt follow the standard formula of a four-year-undergraduate
degree. A PhD program has less structure and a set number of credits wont
guarantee a degree at the end of the program. The process for acquiring a PD
__52__ They must acquire the first-hand knowledge that cant be attained
through books. A candidate must learn from doing, seeing, and being a part of
their area of study. They end goal of a PhD program isnt just to finish a thesis.
__53__ The thesis is proof that the candidate is an expert in his or her field. It is
the final paper to end all papers. Papers written in high school and college are
often research based, but are often just taking old information and rewriting it
in ones own words. __54__ Questions must be answered in a thesis and new
information must be given that never been tackled before.
The PhD program is not for the easily distracted. __55__ For those do make
it through, the end result of being called doctor is well worth the trials and

tribulations.
(A) The process is long and the dropout rate is high.
(B) A PhD thesis pushes this to the side and instead aims to explore new
frontiers.
(C) To expand their knowledge in their chosen field even more, studying
for a doctoral degree, or a PhD, is an option.
(D) The ultimate goal is to become a knowledgeable and dedicated
researcher in ones chosen field.
(E) Like an apprentice in a field where an apprenticeship is necessary, a
PhD candidate must work within their field during their period of
study.
You wake up in the morning with a runny nose and a cough. Your head us
pounding and your body aches. What would you do in the hope of making
yourself feel better? __56__ Sometimes they can seem strange to someone who
didnt grow up with the same beliefs. Many traditional cures center on the use
of certain herbs, fruits, or vegetables, and the people who use them swear by
them. __57__
The belief that vitamin C can cure and even prevent the common cold is
prevalent throughout the U.S. and Canada, as well as other Western countries.
__58__ So why are people in the West so convinced?
Although the idea that Vitamin C can improve the immune system has been
around for a long time, the idea was popularized by an American Nobel
Prize-winning scientist named Linus Pauling. In 1970, Pauling published a
book titled Vitamin C and Common Cold. __59__ However, controlled
experiments done by scientific agencies have failed to yield any proof that a
large dose of vitamin C will make you feel any better. Either way, at least
taking extra vitamin C is harmless, and definitely an improvement over popular
American remedies of the past. __60__
(A) For example, ask any American what you should eat when you have a cold,
and youll likely hear that you should try taking vitamin C.
(B) One popular method around 1900 had Americans putting whole onions into
dirty socks and then wearing them around their necks!
4

(C) Americans have since popped vitamin C supplements and eaten vitamin
C-rich fruits like oranges and tangerines.
(D) Every society has different ideas about how to cure the common cold.
(E) However, as widespread as this belief may be, there is little scientific
evidence to back up the claims that vitamin C can do anything to make you
feel better.
()
Eczema is a chronic condition where skin becomes reddened, dry, and

swollen. Unlike simple dermatitis, an isolation of the skin, the causes of eczema
are found deep inside the body. Half of all eczema patients also suffer from hay
fever, while 20 percent have asthma all are diseases associated with immune
system dysfunction. Therefore, we can safely claim that immune system
dysfunction plays a pivotal role in eczema. When the body has an imbalance in
the production of the chemicals that control inflammation, some illnesses, such
as eczema, can occur.
It may seem obvious, but the first step in treating eczema or day skin is
prevention dont let the skin dry out. The moister you keep the skin, the more
under control the eczema will be.
Here are some suggestions that should help lesson the severity of eczema
outbreaks:
Avoid irritants, particularly fragrances and dyes. Foods that tend to cause
allergic reactions, such as dairy products, eggs, and nuts, may make eczema
flare-ups worse.
Use a non-irritating detergent. Liquids are preferable to powders because
they rinse easily. Run clothes through the rinse cycle twice and wash new
clothes before wearing them.
Dont use soap. Most soaps tend to dry the skin. Non-soap cleansers are
usually gentler and allow the skin to retain more moisture.
Moisturize. In general, the thicker the moisturizer, the better it is. Go for one
with an ointment base because lotions are mostly water with just a little oil

thrown in. Low-cost products such as petroleum jelly and even solid
vegetable shortening such as Crisco work well, too.
Avoid hot water for hand washing, bathing, or showering because it dries
skin. Limit baths and showers to 5-10 minutes. Gently pat skin dry.
61. What is eczema?
(A) A skin disease where the infected area appears red, dry, and inflamed.
(B) Simple dermatitis.
(C) An isolated irritation of the skin.
(D) A disease found inside the body.
62. According to the article, what is the main cause of eczema?
(A) Hay fever.
(B) Asthma.
(C) Chemical infection.
(D) Immune system dysfunction.
63. What is the best way to treat eczema?
(A) Keep the skin moisturized all the time.
(B) Lessen the severity of eczema outbreaks.
(C) Use detergent and do not use soap.
(D) Do not use water when taking a shower.

64. How come eczema patients should avoid dairy products?


(A) Because they are poisonous.
(B) Because they can cause pimples.
(C) Because they can cause allergic reactions.
(D) Because they may upset ones stomach.
According to a joint report by the U.S. Department of Defense and the U.S.
Department of Transportation, GPS, the global positioning satellite system,
consists of 24 satellites that became fully operational in April 1995. Space,
control, and user are the three major points of reference for GPS. The space
reference is the 24 satellites that operate in six orbital planes; the control
reference s the five monitoring stations that track the satellites and gather
information; and the user reference is the antenna and receiver-processor that
provide positioning, velocity, and precise timing to the user.
GPS, developed by the U.S. Department of Defense, is primarily a
5

navigation system. It provides two levels of service: standard positioning and


precise positioning. The standard system is a positioning and timing service
available for use by the general public. The precise system is a more accurate
one available only for military use.
These days, more and more drivers are turning to GPS for directions to get
from point A to point B. since its initial development, the range of uses for GPS
systems has broadened to include finding fishing holes, receiving television
transmissions, and mapping geological changes.
65. Which of the following is the best title for this article?
(A) GPS.
(B) What is GPS?
(C) The Functions of GPS.
(D) The Invention of GPS.
66. What was the primary purpose of GPS when it initially came about?
(A) Military navigation.
(B) Driver navigation.
(C) Flight navigation.
(D) Receiving television transmissions.
67. What are the three major references for GPS?
(A) U.S. Department of Defense, U.S. Department of Transportation, and
the general public.
(B) Militarism, commercialism, and transportation.
(C) Satellites, airplanes, and automobiles.
(D) Space, control, and user.
68. Who uses the precise positioning GPS?
(A) The military.
(B) Car drivers.
(C) Fishermen.
(D) The general public.
Nearly 20 years later, Harrison Ford swings back into action in the fourth
Indiana Jones movie, The Kingdom of the Crystal Skull.
He wears a stylish fedora, wields a fearsome whip, and is terrified of snakes.
Who could this be? Ask any American of a certain age, and the answer will
probably come easily: Indiana Jones.
With his 1981 debut in Raiders of the Lost Ark, Indiana Jones, played by
Harrison Ford, captured the imagination of a generation. Part daring adventurer
and part learned archaeologist, the "Indy" character has been the hero of three
films created by the star team George Lucas and Steven Spielberg. With Lucas

producing and Spielberg directing, the first movie was an instant success,
winning four Oscars and ranking among the highest grossing movies ever
released up to that time. Ford starred in two more Indiana Jones movies, The
Temple of Doom (1984) and The Last Crusade (1989), both produced and
directed by Lucas and Spielberg.
Following the success of the movies,
Indiana Jones has also turned up as the protagonist of television series, video
games, novels, and comic books.
Dr. Henry "Indiana" Jones, Jr. divides his time between teaching as a
professor of archaeology and trekking around the world in pursuit of ancient
relics. The first three movies are set in the 1930s. In the first and third, Indy is
pitted against German Nazis in a battle to uncover mysterious relics believed to
hold great power. In The Temple of Doom, Indy uncovers a gruesome cult in
India and rescues children held by the cult as slaves.
Lucas and Spielberg have teamed up again to produce the fourth installment,
The Kingdom of the Crystal Skull. This time, Harrison Ford stars as an older
Indy, who races against Soviet agents for the fabled Mayan Crystal Skull. The
movie is set for world release on May 22.
69. Based on the above article, what best describes Indiana Jones's personality?
(A) He is romantic, strong, and talented.
(B) He is smart, adventurous, and just.
(C) He likes to read books and is afraid of danger.
(D) He is much more adventurous than intelligent.
70. Which name was Indiana Jones unlikely to have been called during his
lifetime?
(A) Indy.
(B) Henry.
(C) Harrison. (D) Professor Jones.
71. What is probably least significant in Indiana Jones's popularity?
(A) He wears a hat with a fancy name.
(B) His movies are filled with adventure and foreign settings.
(C) Extremely talented teams created the movies.
(D) Indiana Jones is courageous, and he wants to help people.

When you close your eyes and imagine a trip to Africa, what do you see?
Perhaps you picture nature parks filled with wild and fascinating animals, great
peaks rising out of wide deserts, or beautiful lakes. Or perhaps your interest is
piqued by early human art and artifacts, or one of Africas famous stone towns.
Whatever your interest may be, you can find samples of all Africas wonders in
Tanzania.
Tanzania has considerable tracts of wildlife habitat, including the famous
Ngongoro Crater and Serengeti Plain, home to over 70 large mammals and
many smaller species. The 500-mile overland migration of two million animals,
such as wildebeests, gazelles and zebras, is the largest and most famous in the
world. Lions, leopards, rhinoceroses, hippopotamuses, elephants, and other
amazing animals all call Tanzanias plains home, and tourists can visit them on
preserves across the country. Tanzania is also home to Mount Kilimanjaro,
Africas tallest mountain, and the deepest and largest lakes on the continent.
In addition to the fascinating environment in Tanzania, the country is also
home to some early rock art and impressive historical remains. Almost two
hundred sites are found in Tanzania, and some date back to 3500 BC. The
paintings are mainly red and white, though sometimes black or brown paint can
be found. Most of the paintings depict people or animals and are often located
on giant rock faces overlooking valleys. Visitors can also connect with
Tanzanian history by visiting the famous sites of Kilwa Kisiwani and the stone
town Zanzibar. Both of the sites, located on islands off of Tanzanias coast, are
the homes of former capital cities whose influence reached throughout Africa.
Both of the sites are considered World Heritages Sites by the United Nations
and are home to unique and exciting architecture.
72. Another way to say tracts in the second paragraph is _____.
(A) animal footprints
(B) nature trails
(C) areas of open land (D) examples
73. Which of the following can NOT be found in Tanzania?
(A) Wildlife preserves.
(B) Early human rock art.
(C) Interesting architecture. (D) The worlds largest lake.
74. Which of the following is an example of Tanzanias wildlife?
(A) Wildebeest. (B) Kilimanjaro. (C) Zanzibar. (D) Serengeti.

75. What incredible event can visitors to Tanzania witness?


(A) The migration of millions of animals.
(B) Villagers creating rock art.
(C) The building of impressive stone cities.
(D) Mating rituals of African tigers.

97
:
1-20
21-50
51-60
61-75

CDABA CDBDA CABAD DACAB


BACBD BCCDB BDCBA CADBD ACBBD ABBBB
CEDBA DAECB
ADAC BDDA BCA CDAA

97
( )
:
(
)1. I prefer to ___________ our dispute right now.
(A) medicate (B) merge(C)metaphor(D)martial
(
)2. It takes___________ to make a good plan.
(A) peculiar (B) prescience (C)passive(D)peer
(
)3.He felt deep___________ for his crime.
(A)resolution(B) remorse (C)revile(D)rhetoric
(
)4. Feeling insulted by his ___________, she swept.
(A) sheen (B)slender(C)spate(D) slight
(
)5. Their marriage is full of___________.
(A) bloat (B)bate(C) bliss (D)beget
(
)6.The communitys __________ efforts to build a new library led many to conclude that there was little local support for civic
projects. (A) ambiguous
(B) cardinal
(C) abortive
(D)combustible
(
)7. I was disappointed when I learned that the impressive buildings at the theme park were merely empty _________.
(A) facades
(B) exertions
(C) expedients
(D) dross
(
)8. I planned to _______ all of my possessions in a massive garage sale before embarking on a two-year voyage to circumnavigate
the globe. (A) lacerate
(B) mete
(C) liquidate
(D) invert
(
)9. The violinist accused his unappreciative critics of being _____________.
(A) pious
(B) philistine
(C) pithy
(D) motley
(
)10.A _________ understanding cannot be enforced as a contract; the terms of a contract must be explicit.
(A) verbose
(B) tacit
(C) trenchant
(D) ribald
:
11.(
) In many monasteries, the church is constructed on a grand scale, which often having little relationship to the number
(A)
(B)
(C)
of potential worshipers.
(D)
12.(
)The common fox is reddish in color, but a few of they are sliver.
(A)
(B)
(C) (D)
13.(
)It is not seldom revealed that the first American presidents were not only the young nations most famous citizens but
(A)
(B)
(C)
also its wealthy.
(D)
14.(
)The freezing point of water is approximately zero degrees centigrade, depending on both altitude and air dense.
(A)
(B)
(C)
(D)
15.(
)It was not until 1912 that Arizona became a last of the United States 48 continental states.
(A)
(B)
(C)
(D)
:
Medical researchers in the United States say they have recently found that the sounds and sights of nature can help control pain when
patients go through unpleasant procedures_____16._____.
Lung examination is an unpleasant procedure. ___ 17._____. While patients receive some medications to prevent pain, they remain
conscious throughout the procedure. Researchers at John Hopkins University, in addition to giving patients pain-relieving medicine,
played the sounds of gurgling streams during the three-hour examination. _____18______. Compared to another group of 39 similar

patients, who had only the usual painkillers, the trial group was much more likely to have good control of their pain.
____19._____ . For example, many women use breathing and visualization techniques to distract them from the pain while giving birth
to babies, and meditation methods are widely practiced in coping with anxiety and stress.
____20______ . This suggests that a small investment in very simple environmental improvements could produce remarkable benefits.
(A)
(B)
(C)
(D)
(E)

In carrying out this procedure, tubes should be inserted through the nose or mouth to see the lungs.
The researchers also displayed on walls large pictures of a forest with a river running through it and mountains in the distance.
Earlier research has also found that planting trees near patients windows can speed up recovery rates.
They used recordings of gurgling streams and pictures of beautiful scenery to distract the patients during and after treatment.
The benefits of distraction techniques had long been recognized before doctors discovered them.

Campus Violence
Being teased, threatened and hit is part of a normal day for many students. These children and teens are sadly the __ (21)____ of
bullies. In recent years, victims have started to take action and stand up for themselves. They are fighting back and trying to stop the
harassment and abuse that they ___ (22)_____. Some victims have ___ (23)_____ started to try and protect themselves by carrying
weapons to school. Unfortunately, several shootings in the United States have been the ___ (24)_____of victims of school bullies striking
back at their bullies.
The typical victim is usually shy and sensitive. He or she may be overweight, physically small, or different from other students. They
are targets because they dont fit in with other students. On the flip side, bullies ____ (25)____ for power and are unable to feel
the pain of another person or empathize. ___ (26)_____ show that one in four kids who bully will end up with a criminal record before he
or she reaches the age of 30. In the end, bullies need help as much as the victims and it is up to parents to get this job done.
(
(
(
(
(
(

)21. (A) supporters (B) victims (C) mentors (D) winners


)22. (A) accept (B) receive (C) greet (D) embrace
)23. (A)quick (B)even (C)reluctant (D)ever
)24. (A) pride (B) source (C) results (D) reputation
)25. (A) long (B) short (C) wide (D )deep
)26. (A) Statistics (B) Aeronautics (C) Dynamics (D) Economics
The escalating drug crisis is beginning to take its ___27__ on many Americans. And now ___28___ numbers of well-intentioned
officials and other opinion leaders are saying that the best way to fight drugs is to legalize them. But what theyre really admitting is
that theyre willing to abandon a war we ____29____.
Just the thought of legalization brings up more problems and concerns ____30_____ already exists. Advocates of legalization
should be reminded, for example, that its not as simple as opening up a ___31___ of friendly neighborhood pharmacies.
(
) 27. (A) number
(B) toll
(C) tally
(D) count
(
) 28. (A) growing
(B) planting
(C) jumping
(D) lengthening
(
) 29. (A) have even begun not to fight
(B) even not have begun to fight
(C) have not even begun to fight
(D) not have even begun to fight
(
) 30. (A) but
(B) that
(C) when
(D) than
(
) 31. (A) train
(B) fleet
(C) chain
(D) cycle

The Penghu Islands


The first part of Taiwan to be settled by Chinese was the Penghu Islands Chain. In the early days, Chinese people found it hard to
travel by boat across the Taiwan Strait. Although Taiwan is only separated form the mainland by about 200 kilometers, navigation was
hazardous. Seafarers had to deal with shallows and strong currents. Besides these, there were also threats of bad winter weather and
summer typhoons.
2

Despite these hardships, Chinese started setting the Penghu Islands sometimes in the Sung Dynasty. Fishermen from Fujian Province
were lured to the islands by the abundance of fish. They started a settlement by 1171. Over a hundred years later, the Chinese
government during the Yuan Dynasty appointed an official to the islands. This was the first time that the islands were officially
considered a part of China. For the next two hundred years, Fujianese fishermen continued to migrate to the islands. They brought with
them agriculture, pigs, goats, and pottery. In Penghu, they established a city temples and a local government.
Chinas interest in the Penghu Islands did not last long. However, sometimes after the founding of the Ming Dynasty in 1368, the
local government of Penghu was recalled and all settlers were ordered to return to the mainland. Most fishermen did not leave the islands,
despite the governments order. In the absence of government authority, the Penghu Islands became a safe place for pirates to make a
base. At that time, pirates were a big problem for the Ming government. Japanese and Chinese pirates ravaged the coast of China. In
order to wipe out pirates for good, Ming sailors chased the pirates off of Penghu. These pirates later settled on Taiwan.
Because of pirates, the Chinese government could no longer ignore the Penghu Islands. The process of settlement, which had started
several hundred years earlier, could not be stopped.
(
)32. Which of the following did not make it hard to sail to Taiwan?
(A)Shallows and strong currents.
(B) Chinese settlements on Penghu.
(C) Winter storms.
(D)Summer typhoons.
(
)33. What brought Fujian fishermen to Penghu?
(A)Abundant fish.
(B) The Ming government
(C) Strong currents.
(D)Japanese sailors.
(

)34. Who went to the Penghu Islands after the government left?
(A)Fishermen.
(B) Pirates.
(C) Foreigners.
(D)Ming officials.
)35. Why couldnt the Chinese government ignores the islands?
(A) There werent enough fish.
(B) They were too beautiful.
(C) Pirates used them to raid China.
(D)The government wanted Taiwan.

Since the Hawaiian Islands have never been connected to other land masses, the great variety of plants in Hawaii must be a result of
the long-distance dispersal of seeds, a process that requires both a method of transport and equivalence between the ecology of the source
area and that of the recipient area.
There is some dispute about the method of transport involved. Some biologists argue that the ocean and air currents are
responsible for the transport of plant seeds to Hawaii. Yet the results of floatation experiments and the low temperatures of air currents
cast doubt on these hypothesis. More probable is bird transport, either externally, by accidental attachment of the seeds to feathers, or
internally, by the swallowing of fruit and subsequent excretions of the seeds. While it is likely that fewer varieties of plant seeds have
reached Hawaii externally than internally, more varieties are known to be adapted to external than to internal transport.
(

)36.The author of the passage is primarily concerned with


(A) discussing different approaches biologists have taken to testing theories about the distribution of plants in Hawaii
(B) discussing different theories about the transport of plant seeds to Hawaii
(C) discussing the extent to which air currents are responsible for the dispersal of plant seeds to Hawaii
3

(D) resolving a dispute about the adaptability of plant seeds to bird transport
) 37.The author mentions the results of floatation experiments on plant seeds most probably in order to
(A) refute the claim that Hawaiian flora evolved independently from flora in other parts of the world
(B) suggest that the long-distance dispersal of seeds is a process that requires long periods of time
(C) challenge the claim that ocean currents are responsible for the transport of plant seeds to Hawaii
(D) lend credibility to the thesis that air currents provide a method of transport for plant seeds to Hawaii
) 38.It can be inferred from information in the passage that the existence in alpine regions of Hawaii of a plant species that also
grows in the southwestern United States would justify which of the following conclusions?
(A) The ecology of the southwestern United States is similar to the ecology of alpine regions of Hawaii.
(B) There are ocean currents that flow from the southwestern United States to Hawaii.
(C) The plant species discovered in Hawaii is especially well adapted to transport over long distance.
(D) The plant species discovered in Hawaii reached there by attaching to the feathers of birds migrating from the
southwestern United States.
) 39. The passage supplies information for answering which of the following questions?
(A) Why does successful long-distance dispersal of plant seeds require equivalence between the ecology of the source and
recipient area?
(B) Why are more varieties of plant seeds adapted to external rather than to internal bird transport?
(C) What varieties of plant seeds are birds that fly long distance most likely to swallow?
(D) What is a reason for accepting the long-distance dispersal of plant seeds as an explanation for the origin of Hawaiian
flora?
) 40. Which of the following words is best close to the underlined word excretions?
(A) exertion
(B) exfoliation
(C) emunctories
(D ) execute

Essay: Write a well-organized essay of about 200-300 words on the following topic: ( 20%)

The Changing Roles of English Teachers in the 21 st century

97

:
1

10

:
11

12

13

14

15

:
16

17

18

19

20

21

22

23

24

25

26

27

28

29

30

31
C

32

33

34

35

36

37

38

39

40

Essay: Write a well-organized essay of about 200-300 words on the following topic: ( 20%) ********

The Changing Roles of English Teachers in the 21 st century

97
PART I: Multiple choices
A. Choose the most appropriate vocabulary which has the meaning similar to the
underlined words. (30%)
1.

____
2.
____
3.
____
4.
____

5.
____

6.
____
7.
____
8.
____
9.
____

10.
____

Bricks and cinder blocks are made from the fused mineral residue of coal.
(A) wastes
(B) origins
(C) powder
(D) remainder
Cacti are indigenous to arid regions.
(A) recurring
(B) restricted
(C) harmful
(D) native
Exhilarating effects may result from the caffeine content in coffee.
(A) hypnotic
(B) elating
(C) intricate
(D) prodigious
Thinking people disdain TV commercials that use pretty girls instead of informative data to
sell their products
(A) scorn
(B) conflict
(C) change
(D) deride
Through magic the prehistoric man sustained his sense of security in his struggle for
subsistence.
(A) hostility
(B) posterity
(C) existence
(D) decency
English physicist John Tyndall wrote of superstition as incongruous with intelligence.
(A) shocking
(B) inconsistent
(C) mingled
(D) recurring
Arrowroot is an edible, easily digested starch extracted from certain plants.
(A) an eatable
(B) a fine
(C) a cuisine
(D) wholesome
Aspirin is efficacious in bringing down body temperature.
(A) forceful
(B) effective
(C) dependable
(D) futile
In botany, a succulent plant is one that, like cactus and other desert plants, is provided with
thick, porous tissues that can store up water.
(A) delicious
(B) fruitful
(C) cynical
(D) juicy
Screw shells are beautiful examples of shells with many whorls that expand very gradually
towards the aperture.
(A) beginning
(B) opening
1

(C) mid point


(D) bottom
11.
An art critic is not expected to deal with anything so mundane as the price of a work of art.
(A) practical
____
(B) versatile
(C) worldly
(D) meaningful
12.
An introductory course in economics helps one approach social problems from a pragmatic
and dispassionate point of view.
____
(A) inordinate
(B) ingenious
(C) practical
(D) coherent
13.
The ubiquitous coconut palm on the Pacific islands still keeps to itself the mystery of its
geographical origins.
____
(A) omnipresent
(B) versatile
(C) unique
(D) extravagant
14.
Stars are now forming throughout our Milky Way, as well as in a myriad of other galaxies
throughout the Universe.
____
(A) great number
(B) trillion
(C) small number
(D) few
15.
Mostly nocturnal, the bittern eats frogs, fish, and insects.
(A) migratory
____
(B) omnivorous
(C) living in marshes
(D) active at night

B. Choose the best topic sentence for the paragraph based on supporting details.
(10%)
1.
_____

2.
_____

________________________. For one thing, you should always remove your shoes when
you enter a Japanese home, and you should leave them pointing toward the door. Another
suggestion is to make sure that you bring a gift for your Japanese hosts, and to be sure to
wrap it. A third recommendation is to be appreciative of things in a Japanese house, but
not too appreciative. Finally, remember that when you Sit down to eat, you do not have to
accept every kind of food that you are offered, but you are expected to finish whatever you
do put on your plate.
A) Visiting a Japanese home is very enjoyable.
B) Taking a gift is very important when you visit a Japanese home.
C) There are a number of things to keep in mind when you visit a Japanese home.
D) When you visit a Japanese home, be sure not to eat too much.
__________________________. One reason is that when commercial flights began, all
pilots were male. Men were hired because they had flight experience obtained in combat.
Women, not having been in combat, had no flight experience. A second reason is simply
prejudice: The powers in the airline industry presumably believed the stereotype that there
are certain jobs that women cannot do as well as men. A third reason is inertia and the
status quoflying has mostly been a male-dominated profession since it began, and it
takes time to change things. Eventually we will see more and more female commercial
airline pilots, but for the present, old ideas die hard.

A) Why there are so few women commercial pilots today.


B) There are three principal reasons why there are so few women commercial pilots
today.
C) Women pilots in aviation.
D) Men are still prejudiced about women's capabilities.
3.

_____

4.
_____

5.
_____

_________________________. The outside looked like an ordinary building, except


that the windows were painted black. When you opened the door and walked in, a bell
rang like an alarm. As you waited and your eyes became accustomed to the dim light, you
saw that the jumble of cages contained only poisonous creatures: spiders, snakes, and
dark things you couldn't quite make out. Then you heard a voice like a serpent's hiss,
saying "y-e-s-s-s-s-s-s?" in a terrifying welcome.
A) In my hometown there was a zoo that was both fascinating and scary.
B) A zoo in my hometown is Important for animals.
C) The zoo building in my hometown is designed in a special style.
D) A zoo in my hometown is very scary and dangerous.
__________________________. The authoritarian leader holds all authority and
responsibility, with communication usually moving from top to bottom. This leader assigns
workers to specific tasks and expects orderly, precise results. The leaders at United Parcel
Service employ authoritarian leadership. At the other extreme is the laissez-faire leader,
who gives authority to employees. With the laissez-faire style, subordinates are allowed to
work as they choose with a minimum of interference. Communication flows horizontally
among group members. Leaders at Apple Computer are known to employ a laissez-faire
leadership style in order to give employees as much freedom as possible to develop new
products. The democratic leader holds final responsibility but also delegates authority to
others, who participate in determining work assignments. In this leadership style,
communication is active both upward and downward. Employee commitment is high
because Of participation in the decision-making process. Managers for both Wal-Mart and
Saturn have used the democratic leadership style to encourage employees to become
more than just rank and file workers.
A) The authoritarian leader is the most popular type of the three types of leaders.
B) There are more than three types of leaders.
C) To be leaders is not easy because there are too many types.
D) Several styles of leadership have been identified: authoritarian, laissez-faire, and
democratic.
_________________________. No one will read about her in a book about heroes, but in
her small circle of friends, no one doubts her heroism. Certainly my younger brother
doesn't. He is the special beneficiary of her heroism. He was in an accident, and the
doctor, when he was five years old, told us that he would never walk again. My mother
listened respectfully but she didn't believe him. She had optimism. She went to another
doctor and then another. Finally she found one who prescribed exercises. She worked with
my brother for three years. Day after dismal day, she persevered. It wasn't just her
working with him that helped my brother. It was her courage in the face of failure. My
brother worked with her. They both were courageous. We other family members weren't.
To us, my brother and my heroic mother who has endless courage were acting like a
couple of blind fools. We thought my mother especially, the leader, was in prolonged
denial. But in three years my brother was walking. He won't be an athlete; nevertheless, he
gets around. We're proud of him, but we know--and he knows--that without Mother he
would never have walked. Of course, she's not a miracle worker. Most of the time, doctors
are right, some injured people can never walk. But the ones like my brother, who
somewhere have that hidden ability, need that special someone like my mother. She's
more than ordinary. She's a hero.
3

A)
B)
C)
D)

She's more than ordinary.


She's a hero.
Of course, she's not a miracle worker.
My mother is the best example of a hero I can think of.

PART II: English Writing


A. Topic sentence. (20%)
Directions: Look at each of the following sets of details (3~4 sentences in a paragraph) and write an
appropriate topic sentence for the paragraph. .
1
Topic sentence
Supporting
details

y
y
y

For one thing, there's almost always a traffic jam I get stuck in, and I'm
often late to work.
Also, there's not always a parking place when I do get to work.
Worst of all, I'm spending more money on gas and car maintenance than
I would if I took public transportation.

2
Topic sentence
Supporting
details

y
y
y

One is that I often fall asleep when watching the TV screen, no matter
how interesting the video is.
Another is that watching movies is basically a social experience, and I'm
usually alone when I watch videos.
The main reason is that the TV screen, no matter how large it is,
diminishes the impact that you get when watching a movie on the big
screen.

3
Topic sentence
Supporting
details

y
y
y

Nothing spontaneous usually happens on a guided tour, but I've had lots
of spontaneous things happen when I've charted my own vacation
course.
Tour guides present you with what they think is interesting, whereas when
you are in charge of your own vacation, you do what you think is
interesting.
Unplanned vacations can often be cheaper than guided tours.

4
Topic sentence
Supporting
details

y
y
y
y

Cats don't bark and wake up the neighbors or bite the mailman.
Dogs have to be walked at least two times a day, but cats handle their
own exercise.
Cats eat a lot less than dogs.
You can't leave your dog at home when you take a vacation, but you can
leave your cat if a friend or neighbor will come and feed it.

B. Assessment & testing (10%)


Directions: Based on the reading provided, create 2 comprehension questions with 4 choices. An
example is given.
If Your Brain Has a Buy Button, What Pushes It?
Knowing what brand you are buying can influence your preferences by commandeering brain
circuits involved with memory, decision making and self-image, researchers have found.
Circulating in draft form over the last year, the study has been widely discussed by
neuroscientists and advertisers, as well as people who worry about the power of commercials in
determining consumer behavior.
At issue is whether marketers can exploit advances in brain science to make more effective
commercials. Is there a buy button in the brain?
Some corporations have teamed up with neuroscientists to find out. Recent experiments in
so-called neuromarketing have explored reactions to movie trailers, choices about automobiles, the
appeal of a pretty face and gut reactions to political campaign advertising, as well as the power of
brand loyalty.
But the trend also has critics. For example, Commercial Alert, a consumer group that is highly
critical of neuromarketing and has called it Orwellian, said that such studies were dangerous.
What would happen in this country if corporate marketers and political consultants could
literally peer inside our brains and chart the neural activity that leads to our selections in the
supermarket and voting booth? Mr. Ruskin wrote. What if they then could trigger this neural activity
by various means, so as to modify our behavior to serve their own ends?
Defenders of the studies counter that Mr. Ruskin and others who express fears about the
studies are overreacting and do not understand the research.
Companies, however, see the chance to find out what their customers really think as a great
opportunity.
Corporate executives are the first to admit they do not really know how advertising works. They
spend $117 billion a year on advertisements, but most people do not remember what product is
featured in a given commercial. Four out of five new products flop. There is no conclusive evidence
that advertising ever causes sales to go up. Worst of all, consumers tend to behave like finicky cats,
making it difficult to fathom what they want.
Neuromarketing relies on a brain scanning device called functional magnetic resonance
imaging or f.M.R.I., a machine that tracks blood flow as people perform mental tasks. Specific
regions light up, showing increased blood flow when they recognize a face, hear a song, make a
decision, perceive a reward, pay attention or sense deception.
In the studies, the machines are being used to shed light on brain mechanisms that play a
central role in consumer behavior: circuits that underlie reward, decision making, motivation,
emotions and the senses of self.
. Why are people interested in neuromarketing based on the passage?
A) Because it is an opportunity to find out what customers really think.
B) Because most of corporate people do not really know how advertising works.
C) Because it can help know if advertising ever causes sales to go up.
D) Because it can help customers make a decision.

1.

A)
B)
C)
D)
2.

A)
B)
C)
D)
5

C. Write a summary & response (15%)


Directions: Write a reaction (one-paragraph summary and one-paragraph response) based on the
reading above in B If Your Brain Has a Buy Button, What Pushes It?
[Summary]

[Response]

D. Write an essay to explain how you would integrate English writing with reading in
your high school English class (100~200 words). (15%)

Part I (A)

Part I (B)

1.

2.

3.

4.

5.

6.

7.

8.

9.

10.

11.

12.

13.

14.

15.

Potrebbero piacerti anche